board vitals part II

Lakukan tugas rumah & ujian kamu dengan baik sekarang menggunakan Quizwiz!

Which of the following are risk factors for periprocedural bleeding? (Select all that apply) A. Hypertension B. History of stroke C. Abnormal renal or hepatic function D. Female gender E. Age under 50 years

C - Abnormal renal or hepatic function,B - History of stroke,A - Hypertension. Correct answer: (A) (B) (C) Hypertension, history of stroke and abnormal renal or hepatic function. Explanation: The HASBLED classification system of risk factors for periprocedural bleeding lists seven risk factors: hypertension; abnormal liver/renal function; stroke; bleeding history; labile international normalized ratio (INR); elderly (over 65 years); and drugs/alcohol concomitantly.

Which of the following is the most common location of forefoot pain? A. First metatarsophalangeal joint. B. Plantar surface between the second and third metatarsal diaphysis. C. Lisfranc ligament D. Plantar fascia E. Collapse of the transverse arch

Answer: a Explanation: The first metatarsophalangeal joint is the most common location of forefoot pain. Metatarsalgia is the second most common area of forefoot pain, located on the plantar surface below the second and third metatarsal diaphysis. Metatarsalgia is the result of partial or complete collapse of the transverse arch formed by the metatarsal heads. The Lisfranc ligament is one of four ligaments in the Lisfranc joint, a common location for midfoot pain. Plantar fasciitis results in rear foot pain, frequently at the insertion of the plantar fascia into the medial aspect of the calcaneus.

What anatomic structure is targeted by the x-ray beam in a lateral oblique projection of the foot (medial oblique view)? A. fourth metatarsal-cuboid articulation B. fifth metatarsal-cuboid articulation C. cuboid D. calcaneocuboid articulation

Correct answer: (A) Fourth metatarsal-cuboid articulation. Explanation: The lateral oblique projections or medial oblique view is a very common projection used in podiatry. The phalanges, sesamoid bones, metatarsals and tarsal bones are well visualized. The film is positioned flat and the lateral aspect of the foot is parallel and along the edge of the cassette nearest the tube. The X-ray tube is angled 40o with the central ray targeted at the fourth metatarsal-cuboid articulation.

Which of the following is the exception regarding holding Aspirin 7-10 days before surgery? A. Minor dermatologic procedures B. Procedures at high-risk for cardiovascular events C. Surgery within 6 weeks of placement of a bare metal stent D. Bunionectomy with osteotomy E. Surgery within 6 months of placement of drug-eluting stent

Correct answer: (A) Minor dermatologic procedures. Explanation: Because it takes 7-10 days for the entire platelet pool to be replaced, aspirin therapy should be held for 7-10 days in most cases with a few exceptions, as noted above.

Generally, postoperatively, warfarin, low molecular weight heparin, or unfractionated heparin should be resumed: A. Immediately after surgery B. 24 hours after surgery C. 48 hours after surgery D. 72 hours after surgery E. One week after surgery

Correct answer: (B) 24 hours after surgery. Explanation: In most cases, these anticoagulants are resumed within 24 hours after surgery. Only in cases of high-bleeding-risk surgery are low molecular weight heparin or unfractionated heparin delayed for 48-72 hours.

A 41 year old female patient is having a neuroma removed from her right foot today. She is in the pre-op holding area filling out her paperwork. Her only significant medical history is diabetes that is well controlled with diet and most recent HbA1c 6.0. What ASA classification status would this patient be? A. ASA PS 1 B. ASA PS 2 C. ASA PS 3 D. ASA PS 4

Correct answer: (B) ASA PS 2. Explanation: The American Society of Anesthesiologists (ASA) Physical Status classification system is the main system used by anesthesiologists to rate the degree of "sickness" a patient has before surgery. ASA 1 is a normal healthy patient excluding the very old and very young. ASA 2 is a patient with mild systemic disease (well controlled DM), pregnancy, mild obesity. ASA 3 is a patient with severe systemic disease. ASA 4 is a patient with severe systemic disease that is a constant threat to life. ASA 5 is a patient not expected to survive without the operation and ASA 6 is a brain dead patient whose organs are being harvested for donor purposes.

The intermediate chains of the local anesthetic molecule are either made up of esters or amides. Out of the statements listed below, which is a true statement regarding the difference between an ester intermediate chain and an amide intermediate chain? A. Esters are broken down by enzymatic degradation in the liver. B. Amides have a longer duration. C. Esters have a longer half-life. D. Amides are hydrolyzed by pseudocholinesterase.

Correct answer: (B) Amides have a longer duration. Explanation: Local anesthetic molecules can have either an amide intermediate chain or an ester intermediate chain. Depending on the presence of an ester or an amide, the metabolism of the local anesthetic will vary. The ester type anesthetics are hydrolyzed in the plasma by pseudocholinesterase. Additionally the half-life of esters is shorter than that of amides. This correlates with duration with amides being the longer acting of the two.The amide anesthetics are metabolized more slowly in the liver.

Which of the following is true of fentanyl? A. Has a long duration of action B. Has rapid onset C. Is 20% of the potency of morphine D. Is 50% of the potency of heroin E. It is a naturally-occurring opioid

Correct answer: (B) Has rapid onset. Explanation: Fentanyl is a potent, synthetic opioid analgesic with a rapid onset and short duration of action. It is approximately 80 to 100 times more potent than morphine and roughly 40 to 50 times more potent than pharmaceutical grade heroin. During anaesthesia it is often used along with a hypnotic agent like propofol.

What is the greatest limitation of the V-Y plasty? A. It is useful for contractures and burns. B. It allows 20% increase in distance. C. It is easy to perform. D. Thickness of the flap is consistent with adjacent skin. E. There is match of skin color, texture, and hair growth.

Correct answer: (B) It allows 20% increase in distance. Explanation: The V-Y plasty only allows a 20% increase in the distance from the tip as it moves to its new location. If more lengthening is required, other procedures such as the Z-plasty are more appropriate. The procedure does have the advantages of ease of performance, usefulness in contractures and burns, consistency of thickness, and good match of skin color, texture, and hair growth.

Which biopsy technique is the best for obtaining a full-thickness skin specimen? A. Scissor biopsy B. Punch biopsy C. Shave biopsy D. Saucerization biopsy E. Incisional biopsy

Correct answer: (B) Punch biopsy. Explanation: Scissor, shave, and saucerization biopsies are indicated for more superficial skin samples, with the saucerization technique, the deepest of the three, only reaching superficial reticular dermis. The incisional biopsy can provide a deeper specimen but is primarily used to partially remove a small section of involved skin within a large area of involvement. The punch biopsy is a simple and effective technique which provides a full-thickness specimen (epidermis, dermis, subcutaneous tissue).

In reference to MR imaging, what is the term for the time it takes for the transverse magnetization to decay? A. T1 B. T2 C. Proton Density D. Dephasing

Correct answer: (B) T2. Explanation: The time constant T2 describes how quickly the transverse magnetization decays over time. This reflects the tendency of different protons, which are initially aligned together after a 90 degree pulse, to precess quickly out-of-sync with each other.This process of falling out-of-sync with each other at different rates is called dephasing. Additionally, proton density is an important property of tissue. Another important property is how soon the protons realign with the static magnetic field, B0, following a radio frequency excitation pulse. This describes the time constant called T1.

Which statement is true regarding rheumatoid degenerative changes of the subtalar joint? A. They occur in over 75% of rheumatoid patients. B. They occur in 29-42% of rheumatoid patients. C. They're characterized by laxity of the subtalar joint. D. They have no effect on gait. E. They can lead to a pes cavus deformity.

Correct answer: (B) They occur in 29-42% of rheumatoid patients. Explanation: Degenerative changes of the subtalar joint occur in 29 -42% of rheumatoid patients. (A) These changes occur in only 29-42% of patients. (C) They are characterized by stiffness, not laxity of the joint. (D) They can lead to gait disturbances, especially ataxia. (E) They can result in progressive joint collapse leading to a pes valgoplanus deformity.

A pediatric patient is brought in following severe injury to the right leg. Upon obtaining xrays of the affected limb, a fracture is noted along the epiphyseal plate and exiting through the metaphysis with positive Thurston-Holland's sign. This is a Salter Harris: A. Type I B. Type II C. Type III D. Type IV E. Type V

Correct answer: (B) Type II. Explanation: The Thurston-Holland's sign is a hallmark finding of Type II Salter-Harris epiphyseal fractures. As the fracture along the epiphyseal plate exits through the metaphysis, it creates a triangular fragment with this classic appearance. (A) Salter Harris Type I fracture is separation of the piphyseal plate. (C) Salter Harris Type III is an intra-articular fracture through the epiphyseal plate out the epiphysis into the joint. (D) Salter Harris Type IV is an intra-articular fracture extending from the joint through the epiphysis and the epiphyseal plate then exits out the metaphysis. (E) Salter Harris Type V is a crush injury with compression fracture at the epiphyseal plate.

A patient is placed in a below knee cast for a suspected acute charcot event. X-rays are obtained after cast application. What is the appropriate adjustment to ensure adequate visualization of the osseous structures through the dry fiberglass cast? A. 3 x normal mAs OR +15kVp B. 2 x normal mAs OR +10kVp C. 1.5 x normal mAs D. No adjustments are necessary for dry fiberglass cast material

Correct answer: (C) 1.5 x normal mAs. Explanation: Due to the additional cast material, adjustments must be made for cast radiography. For a dry fiberglass cast, the x-ray should be taken at 1.5 times the normal mAs. (A) Wet plaster casts require that x-rays be taken 3 times the normal mAs or an additional 15 kVp. (B) Dry plaster casts or wet fiberglass casts require 2 times the normal mAs or an additional 10 kVp. (D) If the proper adjustments are not made for cast radiography, the image of the underlying structures may not be adequate or beneficial.

Which of the following radiation protection quantities are defined as the measure of the amount of energy that is absorbed in matter when radiation passes through it? A. Effective Dose Equivalent B. Exposure C. Absorbed Dose D. Dose Equivalent

Correct answer: (C) Absorbed Dose. Explanation: Exposure, Absorbed Dose, Dose Equivalent and Effective Dose Equivalent are four quantities measuring radiation protection. Exposure is defined by the measure of the amount of ionization that is produced when radiation passes through matter. Dose Equivalent is the measure of the biological damage caused by radiation. Effective Dose Equivalent is the measure of biological damage caused by radiation to a certain part of the body that is exposed. Absorbed dose is a measure of the amount of energy that is absorbed in matter when radiation passes through it.

A phenol and alcohol matrix ablation is usually immediately followed by rinsing with: A. Acetic acid B. Water C. Alcohol (isopropyl or ethyl) D. Dextrose

Correct answer: (C) Alcohol (isopropyl or ethyl). Explanation: Phenol is soluble in alcohol, so it will accordingly wash away the excess phenol from the nail area. A sodium hydroxide matrix ablation is usually followed by a vinegar (acetic acid) lavage.

Patients presenting with primary headaches disorders that include migraines, cluster, pseudotumor cerebri, exertional headaches on prophylactic medications. What should be their perioperative regimen? A. Discontinue all prophylactic medications. B. Discontinue all prophylactic medications 24 hours prior to surgery. C. Continue their prophylactic medications. D. Avoid all stress during the perioperative period.

Correct answer: (C) Continue their prophylactic medications. Explanation: Headache disorders are heterogeneous group of diseases, broadly categorize as primary and secondary. Primary disorders include migraines, cluster, pseudo tumor cerebri, hemicranias continuum, and exertional headaches. Secondary headaches are ominous processes such as intracranial masses, giant cell arteritis, and hydrocephalus. Prophylactic medications should be continued if possible for primary headache disorders. (A) Prophylactic medications should be continued throughout. (B) If possible prophylactic medications should be continued if possible. (D) Medications should be continued either way.

A patient presents with the following on clinical exam: an overpowering of the extensor digitorum longus during the swing phase of gait, with dorsiflexion at the lesser MPJ reaching 90 degrees, which is abnormal. You notice a bowstring appearance of the extensor digitorum longus during heel off. The patient also complains of pain plantarly around the metatarsal heads. You diagnose lesser digital deformity, what causes this particular deformity? A. Flexor stabilization B. Flexor substitution C. Extensor substitution D. Extensor stabilization

Correct answer: (C) Extensor substitution. Explanation: Can be seen in pes cavus, or neuromuscular disease, the EDL is able to gain mechanical advantage over the stabilizing effects of the lumbricales, resulting in a swing phase deformity via extensor substitution. The normal 30 degrees of dorsiflexion at the MTPJ during the swing phase is able to reach as much as 90 degrees. Equinus may cause extensor contractions as well to develop. When the forefoot is plantarflexed in relation to the rearfoot, the extensor tendons must fire earlier and more intensely during heel off to prevent tripping over the equinus forefoot. A short Achilles tendon may also be responsible for claw toe development.

The following ECG finding is most consistent with? A. Hypercalcemia B. Hypokalemia C. Hypocalcemia D. Hyperkalemia E. Hypomagnasemia

Correct answer: (C) Hypocalcemia. Explanation: Characteristics of hypocalcemia include: • Narrowing of the QRS complex • Reduced PR interval • T wave flattening and inversion • Prolongation of the QT-interval • Prominent U-wave • Prolonged ST and ST-depression

Brachymetatarsia is associated with all of the below EXCEPT: A. Turner's Syndrome B. Down's Syndrome C. Kelly's Syndrome D. Larsen's Syndrome

Correct answer: (C) Kelly's Syndrome. Explanation: Kelly's Syndrome is not associated with Brachymetatarsia. (A) (B) (D) are all associated with brachymetatarsia. A congenital hypoplasia of one or more metatarsals, with the shortening of the fourth metatarsal is the most common.

What tendon balancing procedure often accompanies a transmetatarsal amputation? A. Hibbs transfer B. Jones Tenosuspension C. Split Tibialis Tendon Transfer D. Peroneus Brevis Transfer

Correct answer: (C) Split Tibialis Tendon Transfer. Explanation: With the transmetatarsal amputation the Tibialis Anterior and Tibialis Posterior tendon overpower and create a varus deformity. The Split Tibialis Tendon Transfer transforms the pull of the tibialis anterior from an investor to a pure dorsiflexor. Now there is an even tug-o- war between the Tibialis Posterior and the Peroneus Brevis. (A) The Hibbs transfer is not practically and would be ineffective against a varus deformity. (B) A Jones tensosuspension is impossible as the majority of the first ray must be amputated. (D) Transferring the Peroneus Brevis would be inadvisable as it is the sole remaining evertor after the amputation.

The first ray begins the stance phase of gait in a maximally dorsiflexed position due to the pull of this muscle. A. The peroneus longus B. The posterior tibialis C. The anterior tibialis D. The extensor hallucis longus E. The extensor digitorium longus

Correct answer: (C) The anterior tibialis. Explanation: The first ray begins the stance phase in a maximally dorsiflexed position due to the pull of the anterior tibial muscle which acts after heel-strike to decelerate plantarflexion of the ankle. After the anterior tibial relaxes throughout contact, the first ray plantarflexes toward the supporting surface.

Which of the following is a requirement of radiation safety guidelines for fluoroscopy? A. The protective curtain must be a minimum 0.50 lead equivalent. B. A three-minute timer with an audible alarm is required. C. The exposure switch must be a dead-man type. D. All of the above are required.

Correct answer: (C) The exposure switch must be a dead-man type. Explanation: During a fluoroscopic examination, the fluoroscopy exposure switch must be a dead man type, which refers to a switch that will immediately terminate the exposure when the foot is removed from the fluoro pedal. The fluoroscopy timer must sound an audible alarm or immediately interrupt the exposure after 5 minutes. The protective curtain and Bucky slot shield should both be a minimum of 0.25 mm lead equivalent.

A 62 year-old female with a history of methicillin-resistant staphylococcus aureus presents for preoperative evaluation. What prophylactic antibiotic would be ordered for elective forefoot surgery? A. Cefazolin B. Clindamycin C. Vancomycin D. Amoxicillin

Correct answer: (C) Vancomycin. Explanation: Those patients with a history of previous MRSA infection, resistant MRSA, or a prolonged preoperative hospital stay are best prophylaxed with vancomycin. Cefazolin is the most common agent used in routine prophylaxis. It is inexpensive and effective against Staphylococcus and Streptococcus species, however it is not effective against MRSA. Clindamycin is most effective at penetrating glycocalyx around prosthetic implants. Amoxicillin is an oral antibiotic that does not have MRSA coverage.

Consider the following local anesthetics. Which local anesthetic below would require the least amount of anesthetic to be effective? A. A lipid sol 28 protein bind% 96 pka 8.1 B. B lipid sol 1 protein bind% 78 pka 7.6 C. C lipid sol 3 protein bind% 64 pka 7.9 D. D lipid sol 80 protein bind% 80 pka 8.6

Correct answer: (D) Explanation: The physical and chemical properties that influence local anesthetic activity are lipid solubility, protein binding, and the physical-chemical parameter (pKa). Lipid solubility determines the potency of the local anesthetic. This is logical if one thinks in term the mechanism of action of local anesthetics. The more lipid soluble the anesthetic molecule is, the more easily it can penetrate the perineurium and cellular membrane to gain entry into the nerve cell and cause depolarization. That in turn would mean that a lesser dose would be need to provide a deeper degree of anesthesia. An example of this phenomenon is the addition of a butyl group onto mepivicaine. This leads to the formation of bupivicaine which is 28 more times lipid soluble and 4 times more potent than mepivicaine.

All of the following are examples of chemical methods of hemostasis except? A. Phenol B. Epinephrine C. Thrombin D. Gentian violet

Correct answer: (D) Gentian violet. Explanation: Gentian violet has antibacterial, antifungal, and antihelminthic properties and was formerly important as a topical antiseptic. Phenol is an example of a chemical hemostatic method. It belongs to a very commonly used group including aluminum chloride hexahydrate, sliver nitrate and dichloroacetic or trichloroacetic acids. All act by denaturing and agglutinating proteins. Epinephrine is one of the more preferred methods as it physiologically affects the clotting mechanism. Thrombin activates fibrinogen to form a fibrin clot that results in instantaneous coagulation. Topical thrombin is probably the most effective hemostatic agent, but also one of the most costly. Gentian violet is a triarylmethane dye used as a histological stain and in Gram's method of classifying bacteria.

When assessing a 2 year old child for clubfoot (talipes equinovarus) which is the only bone that would not be seen on x-ray? A. Talus B. Calcaneus C. Cuboid D. Navicular

Correct answer: (D) Navicular. Explanation: The navicular does not ossify until a child is 2-3 years old and would not be seen on x-ray. In neonates only the talus and the calcaneus are ossified.

Which of the following is not a surgery related risk factor for development of deep vein thrombosis? A. Tourniquet time > 90 min B. Rearfoot procedure > 120 min C. Splinting D. Spinal anesthesia

Correct answer: (D) Spinal anesthesia. Explanation: Surgery related risk factors for development of DVT include general anesthesia, supine position, time of operation >105min, tourniquet time >90min, rearfoot/ankle procedure, splinting/immobilization, non-weight-bearing >1 week. Major rearfoot and ankle procedures will typically take more than 2 hours to perform. Thigh tourniquets or mid-calf tourniquets are used more frequently in rearfoot and ankle cases and there is a longer period of non-weight-bearing and immobilization with rearfoot and ankle surgery which all can increase the risk for DVT development.

Which of the following anticoagulant bridging agents are administered orally? A. Enoxaparin B. Deltaparin C. A and B D. Aspirin E. None of the above

Correct answer: (E) None of the above. Explanation: Aspirin is not a bridging agent. Bridging agents enoxaparin and deltaparin are administered parenterally.

An orthotic for functional hallux limitus must facilitate plantarflexion of the first ray in order to be effective. Which of the following are techniques or modifications that facilitate plantar flexion of the first ray? (Select all that apply) A. Dorsiflexion of the hallux B. Reverse Morton's extension C. Minimal cast fill D. Morton's extension

B - Reverse Morton's extension,A - Dorsiflexion of the hallux. Correct answer: (A) (B) Dorsiflexion of the hallux and reverse Morton's extension. Explanation: During the casting process firstly plantar flexion can be achieved by dorsiflexion of the hallux. Another method of plantar flexing the first ray is manually plantar flexing the first ray with the clinician's hand. A reverse Morton's extension is an addition onto the orthotic that increases plantarflexion of the first ray. This addition increases the ground reactive force under metatarsal heads two through five. (C) Minimal cast fill is a technique performed on the positive which increases the devices contact with the foot. (D) A Morton's extension would actually cause dorsiflexion of the first ray.

What radiographic findings are characteristic of a posterior cavus foot deformity? (Select all that apply) A. Meary's angle > 10° B. Meary's angle < 10° C. Calcaneal inclination angle > 30° D. Calcaneal inclination angle < 30°

C - Calcaneal inclination angle > 30°,B - Meary's angle < 10°. Correct answer: (B) (C) Meary's angle < 10° and calcaneal inclination angle > 30°. Explanation: The posterior cavus deformity is characterized on the lateral projection by a Meary's angle less than 10° and a calcaneal inclination angle greater than 30°. The posterior cavus foot deformity has a relatively "flat" forefoot hence the small Meary's angle. The posterior cavus deformity is characterized on the lateral projection by a Meary's angle less than 10° and a calcaneal inclination angle greater than 30°. The posterior cavus foot has a high calcaneal pitch hence the high calcaneal inclination angle. (A) A Meary's angle > 10° is characteristic of an anterior cavus foot deformity. (D) A calcaneal inclination angle < 30° is characteristic of an anterior cavus foot deformity.

From the options below, which fixation options are appropriate for graft fixation in the Evan's procedure? (Select all that apply) A. Screw fixation B. Staple fixation C. Plate fixation D. Kirschner wire fixation

C - Plate fixation,B - Staple fixation. Correct answer: (B) (C) Staple fixation and plate fixation. Explanation: Staple fixation is appropriate for fixation of the interpositional graft in the Evan's procedure. The goals of fixation in this procedure is to prevent loss of correction and to prevent dislocation of the graft. Compression is not needed as adequate compression is already inherently in the osteotomy and additional compression would likely result in graft resorption and loss of correction. Plate fixation is appropriate for fixation of the interpositional graft in the Evan's procedure. The goals of fixation in this procedure is to prevent loss of correction and to prevent dislocation of the graft. Compression is not needed as adequate compression is already inherently in the osteotomy and additional compression would likely result in graft resorption and loss of correction. (A) Screw fixation is not appropriate for fixation of the interpositional graft in the Evan's procedure. Compression is not needed as adequate compression is already inherently in the osteotomy and additional compression would likely result in graft resorption and loss of correction. (D) Kirschner wire fixation is not adequate to prevent dislocation of the interpositional bone graft.

Which congenital deformity is referred to as the deformity with a vertical talus? A. Talipes Equino Varus B. Congenital Pes Valgus C. Calcaneovalgus D. Metadductus

Correct answer: (B) Congenital Pes Valgus. Explanation: A vertical talus deformity is a rare condition that is also known as congenital pes valgus. The condition is rare but when left untreated can result in a painful rigid flat-foot. Calcaneovalgus is the most common congenital abnormality but is not usually as rigid.

Muscles in the calcaneal compartment include the: A. Flexor digitorum brevis B. Abductor digiti quinti C. Quadratus plantae D. Abductor hallucis

Correct answer: (C) Quadratus plantae. Explanation: The FDB is in the superficial compartment. The abductor digiti quinti is in the lateral compartment. The abductor hallucis is in the medial compartment.

Which of the following nerve blocks usually requires ultrasound guidance? A. Mayo block B. Ankle block C. Digital block D. Achilles tendon block E. Popiteal block

Correct answer: (E) Popiteal block. Explanation: The popliteal block is best accomplished with ultrasound guidance due to its high rates of dysesthesia and difficulty in localizing the nerve. The other blocks are easily accomplished without ultrasound guidance due to the superficial location of the nerves and the ease of locating them anatomically.

In regards to skin flap physiology which term below is best defined as the force applied per cross-sectional area? A. Stress B. Strain C. Creep D. Stress relaxation

Correct: A Explanation: There are four important concepts pertaining to flap physiology: stress, strain, creep and stress relaxation. Stress refers to the force plaid per cross-sectional area. Strain refers to the change in length divided by the original length of the given tissue to which a force is applied. Creep refers to the increase in strain seen when skin is under constant stress. This occurs in a matter of minutes and is due to an extrusion of fluid from the dermis and a breakdown of the dermal framework. Stress relaxation is the decrease in stress when skin is held in tension at a constant strain for a given time. This occurs over a matter of days to weeks and is due to an increase in skin cellularity and the permanent stretching of skin components.

An infant undergoes successful closed reduction of metatarsus adductus in casts followed by an appropriate period spent in therapeutic shoes after casting is complete. The deformity recurs twice. The hallux is noted to be in varus after the second recurrence. The next step is: Abducting osteotomies of all the metatarsals A tarsometatasal capsulotomy with a second metatarsal osteotomy Abductor hallucis tendon transfer followed by another attempt at closed reduction in casts Metatarsal base chondrotomies

Correct: C Explanation: In this case the hallux varus complication complicates the reduction, and may even be a major etiology in the metatarsus adductus. The tendon should be transferred and another attempt at closed reduction should be made. Abducting osteotomies of all the metatarsals is excessive and unnecessary as only the hallux is affected. Likewise there is no need for a second metatarsal osteotomy. For the same reason the notion of metatarsal chondrotomies would have no indication because the deformity is at the metatarsophalangeal joint level.

This is the maximum cyclic load that a metal can withstand without fracturing when subjected to a designated number of cycles. This is an average for the entire metal, meaning that half of the specimens break at lower level: A. Modulus elasticity B. Passivation C. Fatigue strength D. Shear E. Stiffness

Correct: C Explanation: The maximum cyclic load that a metal can withstand without fracturing when subjected to a designated number of cycles. This is an average for the entire metal, meaning that half of the specimens break at lower level. Modulus elasticity is a measure of stiffness of a substance along its elastic range. It is calculated by dividing stress load by the amount of strain or deformation. Shear is the mechanical term, for which there is parallel deformation on internal surfaces or material. Stiffness is a resistance of a solid body to bend or defect by an applied force.

Which procedures listed below incorporates a 60° osteotomy? (Select all that apply) A. Kalish B. Scarf C. Austin D. Proximal chevron

D - Proximal chevron,C - Austin. Correct answer: (C) (D) Austin and proximal chevron. Explanation: The classic Austin bunion procedure has been described with a 60° osteotomy. This brings to the table a great amount of inherent instability. Modifications of the angle have been described. The osteotomy in the proximal chevron bunion procedure is also 60°. (A) The Kalish modification has a 55° osteotomy which produces a long dorsal wing to allow easy screw fixation. (B) The scarf osteotomy has been described with a proximal osteotomy of 45-60° and a distal osteotomy 70-90°.

Which one of the following diabetic medications should be restarted once oral intake is tolerated after surgery? (Select all that apply) A. Pioglitazone B. Incretins C. Metformin D. Secretagogues

D - Secretagogues,B - Incretins,A - Pioglitazone. Correct answer: (A) (B) (D) Pioglitazone, Incretins and Secretagogues. Explanation: Antihyperglycemic agents such as Thiazoladinediones (Pioglitazone), incretins (Exenatide, Liraglutide) , and secretagogues (Glyburide, Glipizide, Glimeperide, Gliclazide) may be restarted once enteral intake is permitted, although metformin is commonly avoided postoperatively in the hospital in case intercurrent events ensue that might change renal function acutely such as hypotension, iodine dye induced renal dysfunction, sepsis. The same intercurrent events may lead to the risk of lactic acidosis. Metformin has been associated with the development of lactic acidosis. It should be withheld 24 hours preoperatively and restarted 48 to 72 hours postoperatively once normal renal function has been documented, therefore the restart of Metformin is dependent on the renal function.

The following patient will need "bridging therapy" before any lengthened foot and ankle surgery requiring long term immobilization and risk of bleeding. (Select all that apply) A. Hypertension B. Rheumatic atrial fibrillation C. Atrial fibrillation with a history of cardioembolic stroke D. Female gender E. Atrial fibrillation with a mechanical heart valve

E - Atrial fibrillation with a mechanical heart valve,C - Atrial fibrillation with a history of cardioembolic stroke,B - Rheumatic atrial fibrillation. Correct answer: (B) (C) (E) Explanation: If anticoagulation reversal is required before surgery, there will always be some risk of thromboembolism. This risk is dependent on the following factors: the underlying indication for warfarin therapy, the patient's risk factors for thromboembolism, the duration of anticoagulation cessation, and whether the anticoagulation is completely or partially reversed. Perioperative bridging therapy allows us to influence the amount of time the patient is not anticoagulated. Patient s at high risk of thromboembolic events will benefit from a bridging therapy, whereas patients at low risk are unlikely to benefit and may be exposed to unnecessary risks. The CHADS-2 risk classification is a validated tool designed to estimate the annual stroke risk in patients with atrial fibrillation by assigning points to risk factors. The CHADS-2 score could be applied to predict the perioperative risk of stroke in patients with atrial fibrillation. A CHADS-2 score of 5 or 6 are considered high risk for stroke and therefore may benefit from perioperative bridging. Other risks for stroke that warrant perioperative bridging therapy are rheumatic atrial fibrillation, atrial fibrillation with a history of cardioembolic stroke, atrial fibrillation with a mechanical valve.

You are asked to obtain consent for surgical repair of the ankle from a 90-year-old female. The patient has a history of major depression and multi-infarct dementia and lives in a nursing home, where she slipped and fell. No evidence of syncope or loss of consciousness was evident. She appears to fully understand the nature of her condition. When you enter the room to obtain patient consent, the patient is disoriented about place and time. She has no healthcare proxy listed in her medical record and no power of attorney. Which of the following is the most appropriate next step? A. Since patient has been cleared by psychiatry, consent her for surgery B. The physician should determine the patient's capacity to make an informed decision C. Do not consent patient for surgery D. Consent patient, but place note in chart explaining her condition E. Call patient's daughter to obtain consent for surgery

Main learning point: Understand the components of decision-making capacity, and that a patient must understand the nature/purpose of the procedure, risks and benefits as well as the availability of alternatives. Correct answer: (B) The physician should determine the patient's capacity to make an informed decision. Explanation: For a patient to give informed consent, he or she must understand the nature of the procedure, purpose of the procedure, risks and benefits, and availability of alternatives. The exceptions are emergency, patient waiver, lack of capacity (which refers to a physical and mental state that prevents simple communication), possible suicide, or gross psychosis and dysfunction. If you are uncertain, assume the patient is competent. A physician should determine whether the patient is legally and psychologically capable of making a healthcare decision. The components of decision-making capacity include the following principles: 1) The patient is eighteen years old or older or legally emancipated. 2) The patient makes and communicates a choice. 3) The patient knows and understands the choice (informed). 4) The patient's decision remains stable over time. 5) The patient's decision is consistent with the patient's values and goals and is not influenced by a mood disorder. 6) The decision is not the result of an altered mental state, including delusions, hallucinations, or delirium.

Which of the pathologies below is an indication for a Tibialis Anterior Tendon Transfer? A. rigid forefoot equinus B. weak invertors C. cavovarus deformity D. recurrent talipes equinovarus

Correct Answer: D. Recurrent talipes equinovarus. Explanation: The tibialis anterior tendon transfer involves releasing the tendon form its insertion and then transferring it into the third cuneiform. The goal of the procedure is to reduce the supinatory forces in the foot. It can also increase dorsiflexion strength if other dorsiflexors are weak. It is indicated for the following conditions: Recurrent clubfoot; Flexible forefoot equinus; Drop foot; tarsometatarsal amputations; Charcot-Marie-Tooth deformity. In general the STATT procedure has largely replaced this procedure with less complications. In the total tibialis tendon transfer a severe flatfoot deformity can develop if the tendon is transferred too laterally.

Dobbs' approach to vertical talus involves: A. Serial casting. B. Open reduction at an early via the Cincinnati incision. C. A minimally invasive surgical technique. D. Serial casting followed by a minimally invasive surgical technique.

Correct Answer: D. Serial casting followed by a minimally invasive technique. Explanation: Alaee, Boehm, and Dobbs have noted the complications associated with extensive open reduction of vertical talus. These include wound necrosis, talar AVN, rearfoot stiffness, subsequent DJD, undercorrection, and the need for multiple surgeries. Dobbs has developed a technique in which manipulative serial casting is performed weekly, with usually 4-6 casts required. Casting is continued until the TNJ is reduced, confirmed with x-ray. At this time, surgery is scheduled for percutaneous fixation of the TNJ, and a percutaneous tendo-Achilles tenotomy. In the OR, the foot is plantarflexed and inverted, and the wire placed across the joint. The Achilles tendon is tenotomized 1cm proximal to its insertion. A long leg cast is applied with the foot in neutral and the ankle in 5 degrees of dorsiflexion, changed at two weeks post-op. At that time, the patient is casted for a night-time brace.

Consider the following local anesthetics. Which local anesthetic below has the longest duration? A. A B. B C. C D. D

Correct answer: (A) Explanation: The physical and chemical properties that influence local anesthetic activity are lipid solubility, protein binding, and the physical-chemical parameter (pKa).The duration of the local anesthetic is determined by the degree of protein binding. The higher the degree of protein binding, the longer the duration of the local anesthetic. This is because the protein acts like a "sponge" and slowly releases the local anesthetic molecules which spreads out the action of the anesthetic over a longer duration. The protein binding in bupivicaine exceeds 90% while the protein binding of lidocaine is 64%. In clinical use this translates to bupivicaine being two to three times longer acting than lidocaine.

Which of the following sizes indicates the largest diameter suture? A. 0 B. 2-0 C.3-0 D. 9-0

Correct answer: (A) 0. Explanation: A suture's diameter, also known as caliber, is a measure of its cross-sectional diameter in millimeters. The United States Pharmacopeia (USP) is the official organization responsible for defining and characterizing suture material. One of the USP's responsibilities is designating suture size. The USP size indicates the precise metric diameter range for the suture material to be able to achieve a specified tensile strength. The higher the USP size, the smaller the diameter of the suture, and therefore the lower the tensile strength of the material. The smaller the suture diameter, the more 0's are used for its size, which is indicated by the number in front of the 0's, such as "4-0." "4-0," of 0000 suture, would be smaller than 3-0 suture, or 000. So, the higher number of 0's, the smaller diameter the suture will be. (B) (C) It is has more 0's than just 0 suture indicating a smaller diameter. (D) It is has more 0's than just 0 suture indicating a smaller diameter. 9-0 suture is a very thin suture as one would imagine. It typically requires the use of surgical loupes to be visualized and tied properly. It is commonly used in nerve repair.

As a resident, you forget to d/c the patient's heparin in anticipation of a 7:30am surgery. It is now 4:00am; you awake in a start and rush to write your d/c order. If discontinued now, what is the earliest the surgery can safely be performed? A. 10:00am today B. 5:30am tomorrow C. In five days D. In seven days

Correct answer: (A) 10:00am today. Explanation: The half-life of heparin is six hours; therefore, the surgery can be safely be performed without excessive bleeding six hours after discontinuing the drug. (C) An example of an anti-coagulant which requires stoppage five days prior to surgery is warfarin. (D) Aspirin is an example of an anti-coagulant which must be discontinued seven days prior to elective surgery, due to irreversible binding to platlets.

During gait cycle, what is defined as stride length? A. Number of steps per minute B. Distance between two consecutive contacts of the same foot C. Heel contact of one limb to heel contact of the opposite limb D. The angle formed by the longitudinal axis of the foot and line of progression

Correct answer: (B) Distance between two consecutive contacts of the same foot. Explanation: Stride length is defined as the distance between two consecutive contacts of the same foot. Therefore, it is measured from heel contact of one limb to heel contact of the same limb. The average stride length for normal persons is 1.41 m (4.5 ft). Men have a 14% longer stride length than women.

An SLE patient is scheduled for unilateral hammertoe surgery, to correct a digiti quinti varus deformity. The patient has been on 20mg oral prednisone for five years. The anesthesia is straight local. The correct protocol for steroid supplementation for this patient is: A. 25mg hydrocortisone @ induction B. 25mg hydrocortisone @ induction; 100mg through POD 1 C. 25mg @ induction; 100mg daily through POD 3 D. The patient does not require supplementation.

Correct answer: (A) 25mg hydrocortisone @ induction. Explanation: The traditioinal dosing of hydrocortisone supplementation to combat the stressors of surgery was excessively high; up to six-fold based upon mortality in a small number of patients dating back to the 1950's. Now, supplementation is based upon duration of use, amount taken daily, and whether the surgery can be classified as "minor", "moderate", or "major." In this scenario, the surgery can be classified as minor based upon length of surgery (20-30 minutes?) and type of anesthesia. According to the Washington Manual, daily therapy with less than 5mg prednisone should not result in suppression of the adrenal axis. Patients receiving more than 20mg prednisone for longer than three weeks should be considered to have "significant suppression of adrenal responsiveness." The function of the HPA cannot be readily predicted with doses between 5-20mg. (B) This, from the Royal Cornwall Hospitals, is the dosing for moderate surgery. The Washington Manual differs slightly, using 50-75mg the day of surgery and tapering over 1-2 days. (C) This is the recommendation from the Royal Cornwall Hospitals. The Washington Manual recommends 100-150mg the day of surgery, tapering over two days. (D) Although this is a relatively quick surgery under local only, and there are deleterious side effects from steroids, the duration and amount of steroid taken warrants some steroid coverage, albeit small. The benefit:risk ratio leans toward the former.

What is the toxic dose of lidocaine with epinephrine (1:200,000)? A. 500mg B. 300mg C. 175mg D. 225mg

Correct answer: (A) 500mg. Explanation: The epinephrine allows for vasoconstriction and therefore increased amount of added to lidocaine to enhance the effects of the local anesthesia and to retard its absorption. The maximum dose of lidocaine without epinephrine is 300mg. The maximum dose of Marcaine with and without epinephrine is 225mg and 175mg.

You are suspecting an Achilles tendon enthesis in an adult patient, what would be the projection of choice? A modification of the dorsoplantar calcaneal axial projection with the patient's both hand on the rails leaning forward, and the tubehead at 0 degrees The tubehead at 0 degrees, the patient with the knee flexed on the unaffected side, the knee of the affected side sits on the table and the medial side of the foot lies against the cassette, and a 45 degrees wedge is placed beside the sole of the foot, the knee lifted off the table The patient lies prone on the radiographic table, the tubehead is at 40 degrees from the vertical posteriorly, the knee rest on a foam cushion, the toes are forcibly dorsiflexed as they are positioned against the film and the central ray is on the ball of the foot The patient lies supine on the radiographic table, the tubehead is at 40 degrees from the vertical posteriorly, the knee rest on a foam cushion, the toes are forcibly dorsiflexed as they are positioned against the film and the central ray is on the ball of the foot

Correct answer: (A) A modification of the dorsoplantar calcaneal axial projection with the patient's both hand on the rails leaning forward, and the tubehead at 0 degrees. Explanation: Also known as the calcaneal apophysis or superior calcaneal projection, used to assess the posterior calcaneal surface in the adult and the apophysis in the child, with heels parallel to each other on the film and the lower limbs are bent slightly forward, the tubehead at 0 degrees vertical.

What is the role of pretibial muscles (anterior tibial, extensor digitorum longus, extensor hallucis longus) at contact period of the gait cycle in normal gait? A. Absorb impact loads from the floor B. Accelerate plantarflexion at the ankle C. Provide a stable midtarsal joint for shock absorption D. Transform the foot into a rigid lever at heel strike

Correct answer: (A) Absorb impact loads from the floor. Explanation: Role of pretibial muscles (anterior tibial, extensor digitorum longus, extensor hallucis longus) at contact period of the gait cycle in normal gait: 1. Decelerate plantarflexion of the ankle 2. Decelerate pronation and provide even weightbearing from the lateral to the medial side of the foot 3. Provide an unstable midtarsal joint for shock absorption and adaptation 4. Absorb impact loads from the floor At contact, all the muscles are active and reach their peak activity prior to foot flat. Perry52 states that the anterior tibial generates the largest torque around the ankle joint, followed by the extensor digitorum longus and the extensor hallucis longus. At heel contact, the subtalar joint is supinated. The anterior tibial assists in the deceleration of subtalar joint pronation.

What's the recommended management of patients with hyperthyroidism prior to elective surgery? A. Advised to take their anti-thyroid medications the morning of surgery B. Advised to take their anti-thyroid medications the night before surgery C. Advised to stop their anti-thyroid medications the night before surgery D. Advised to bring their anti-thyroid medications to pre-admission, then let anesthesia make the decision according to their most recent laboratory results

Correct answer: (A) Advised to take their antithyroid medications the morning of surgery. Explanation: Patients with hyperthyroidism should take their antithyroid medications on the morning of surgery because T3 and T4 have direct inotropic and chronotropic effects on the heart. In addition, they have a direct effect on vascular smooth muscle causing decrement in systemic vascular resistance and blood pressure. As a result, the renin-angiotensin- aldosterone system is activated, enhancing sodium reabsorption and increasing circulating blood volume therefore increasing cardiac output by 50 to 300%. Chronically elevated thyroid hormones may limit the ability of patients undergoing surgery to respond to stress and lead to cardiovascular collapse. So it is important to diagnose and treat hyperthyroidism, and be well controlled prior to surgery, as failure to identify and treat can drastically increase mortality.

Which one of the following represents AO principles of internal fixation? Anatomic reduction, Early mobilization, Rigid stable fixation, Atraumatic technique on soft tissue as well as bone Delay mobilization until 10 days post-op, anatomic reduction, rigid stable fixation Anatomic reduction, rigid immobilization, stable fixation Cast immobilization, atraumatic technique, surgeon's preference of fixation

Correct answer: (A) Anatomic reduction, Early mobilization, Rigid stable fixation, Atraumatic technique on soft tissue as well as bone. Explanation: AO (Arbeitsgemeinschaft fur Osteosynthesfragen) developed a protocol for the treatment of fractures in Switzerland. Up until then fracture treatment consisted of traction with prolonged bed rest. From continued discussion and observations, the organization decided to produce a system of techniques that would alter the treatment for their patients with fractures. Out of their research they developed these principles known as "The principles of AO fixation": Anatomic reduction, rigid internal fixation, atraumatic technique on soft tissue as well as on bone, and early "pain free" active range of motion during the first 10 post op days.

What slowly-progressing muscle dystrophy often presents in boys who will complain of cramping after physical exertion? A. Becker Muscular Dystrophy B. Blount Disease C. Emery-Dreifuss Muscular Dystrophy D. Duschenne's Muscular Dystrophy

Correct answer: (A) Becker Muscular Dystrophy. Explanation: Becker muscular dystrophy (BMD) is an x-linked condition associated with pseuodhypertrophy of the calves but has a much slower progression allowing the child to walk even after puberty. Life expectancy for someone with BMD is between 40 and 60 years old and largely depends on the degree of respiratory insufficiency. Like Duschenne's, the child will often have a waddling gait and lordosis, but will complain of cramping after any physical exertion. Muscle enzymes will be elevated on work-up and an abnormal dystrophin can be differentiated from the Duschenne type. (B) Blount's disease is not an x-linked condition is associated with a pseduohypertrophy of the calves along with difficulty walking or standing by age 10. Rather, it is the necrosis of the proximal medial tibial epiphysis, often leading to a tibial varum deformity of the child. (C) Emery-Dreifuss Muscular Dystrophy (EDMD) is an x-linked recessive muscular dystrophy that affects older children, teens, and even adults. It too presents with walking difficulties but may also present with cardiac arrhythmias. EDMD progresses slowly with slow stiffening of the spine resulting in fixed deformities of the extremities. Muscle biopsy reveals dystrophic muscle fibersmixed with atrophied muscle. (D) Duschenne's muscular dystrophy is an x-linked, hereditary disorder associated with pseudohypertrophy of the calves, along with the inability to walk by the ages of 10-11 as it it progresses much faster than many other muscular dystrophies. In most cases, the ability to walk to either delayed or never occurs. Some of the first signs of the disease is muscle weakness, usually up to age 5. Boys with Duschenne's muscular dystrophy often have an abnormal gait, such as waddling or toe walking, with hyperlordosis noted. The patient ages and the disease progresses, the child will likely be unable to stand from a seated position on the floor without the use of their arms to climb up their legs, also known as a Gowers Sign. By the age of 10-11, the ability to walk or stand is lost, confining the child to a wheelchair. By mid-teens, respiratory and cardiac muscle weakness often leads to the death of the child.

Which statement regarding the procedure pictured below is not correct? A. Bone cement is used for fixation of the plug. B. Best suited for centralized defects. C. Donor sites can be from the talus, knee or hip. D. Orientation of the plug in respect to cartilage topography is important.

Correct answer: (A) Bone cement is used for fixation of the plug. Explanation: Pictured above is a classic example of an OATS (Osteochondral Autograft Transfer Sytem) procedure. It indeed is best suited for centralized defects. Non-centralized defects or defects on a curvature are best suited for a mosaicplasty which uses multiple smaller plugs. Donor sites for the osteochondral plugs can be the medial aspect of the talus, a nonweightbearing portion of the knee or the hip. Fresh allograft can also be used and in that case the analogous bone is used. Due to the curvature of the affected joint surface it is very important that this is observed when placing the plug so that the curvature of the plug in all planes matches as closely as possible to that of the affected surface. In stark contrast, the notion of using bone cement for fixation of the plug highly inaccurate. The plugs are press fit and do not require fixation. Additionally, allowing the possibility of bone cement into the articulation is wrought with problems.

What condition results from premature closure of an epiphyseal plate of a metatarsal? A. Brachymetatarsia B. Clubfoot C. Macrodactyly D. Ectrodactyly

Correct answer: (A) Brachymetatarsia. Explanation: Brachymetarsia results from premature closure of a growth plate of a metatarsal. The reason for this premature closure of the epiphyseal plate is not known. It commonly occurs at the fourth metatarsal most often and typically presents bilaterally. When more than one metatarsal is involved it is termed brachymetapody. The condition is largely hereditary but can be caused surgically, idiopathically, developementally, or traumatically. It is associated with a number of disorders such as Down's Syndrome, Albright hereditary osteodystrophy, and Turner syndrome to name a few. (B) Clubfoot, or Talipes Equinovarus (TEV), does not result from premature closure of a epiphyseal plate of a metatarsal. The exact cause of TEV is unknown. Clinical evaluation of a patient with TEV reveals hindfoot equinus, hindfoot varus, forefoot adduction, and often cavus forefoot with varying degrees of rigidity. These are often the secondary result of medial displacement of the navicular on the talar head. Other pathoanatomical abnormalities seen in TEV include: lateral rotation of the talus in the ankle mortise, medial and plantar deviation of the talar neck, medial subluxation of the cuboid on the calcaneus. (C) Macrodactyly does not result from premature closure of an epiphyseal plate of a metatarsal. It is an abnormally hypertrophied digit. This congenital deformity often presents unilaterally, rarely seen in the fourth and fifth digits, and is more commonly seen in males. Its exact cause is unknown. It can involve enlargement of the bones, skin, nerves, blood vessels, and nails. (D) Ectrodactyly does not result from premature closure of an epiphyseal plate of a metatarsal. Ectrodactyly is congenital absence of two or more central rays. There are more than 175 terms used to describe the skeletal dysplasia describing ectrodactyly including "cleft foot" and "lobster foot." The tarsal bones are also typical deformed with this condition. The condition is usually present along with "lobster clawing" of the hand and other deformities such as cleft lip and cleft palate.

Comminuted fracture fragments can be stabilized when a plate is applied to maintain the defect cause by comminution. This can be utilized at sites near intramedullary implants, and it allows vascularized comminution to act as graft material. This is also known as: A. Bridge plate B. Transfixation C. Provisional fixation D. Temporary stabilization

Correct answer: (A) Bridge plate. Explanation: Although the technique is most commonly used for fractures associated with hip arthroplasty stems, the operation has also been successfully applied to fractures about intramedullary nails and those about total knee arthroplasty femoral stems. Each of these situations poses similar technical challenges to the surgeon. The hallmark of the described technique is the application of modern biologic fracture fixation principles that minimize the soft-tissue disruption about the fracture and therefore maximize the healing potential. When these techniques are combined with use of a single lateral plate, a periprosthetic femoral fracture about a stable implant can be treated successfully without the use of adjuvant bone-graft material.

This is the least common hyperdactyly. A. Central polydactyly B. Preaxial polydactyly C. Central hemydactyly D. Postaxial polydactyly

Correct answer: (A) Central polydactyly. Explanation: Central polydactyly is the least common and involves duplication of the innermost digits most often the second toe. Preaxial is applied to medial digital duplication. Polydactyly is a common congenital deformity defined as excess of five digits in humans. It has racial and geographic predilections and may occur as an isolated deformity or less commonly, in association with certain congenitally inherited syndromes. Postaxial polydactyly is the most common hyperdactyly variant and denotes lateral digital duplication.

67 year old male with chronic renal failure who underwent a bunion removal also had a right femoral popliteal bypass graft 5 days ago. He starts to complain of pain in the posterior portion of his left calf. On examination there is swelling, slight erythema, and pain upon dorsiflexion of the foot. The right extremity in comparison does not have these symptoms and the incision site is healing well with no signs of infection. Patient has a fever of 100.6 F. Which of the following is the next best step to help confirm the most probable diagnosis? A. Doppler Ultrasound of Lower Extremity B. X-ray of lower extremity C. MRI with contrast of the lower extremity D. CT scan with contrast of lower extremity

Correct answer: (A) Doppler Ultrasound of Lower Extremity. Explanation: The most likely diagnosis is a DVT secondary to surgery and prolonged immobilization. The patient has the symptoms of erythema, swelling and a positive Homan's sign. The best initial test to help with your diagnosis would be a Venous Doppler Ultrasound (A). An x-ray (B) would not show any signs of DVT. The MRI and CT scan with contrast (C+D) would not be the best initial step and would not be indicated in a patient with renal failure.

23 year old construction worker presents to the ED after a boulder fell on his left foot, damaging his steel toe boot. He denies any other trauma. The patient socks and shoes were removed before arrival to the fracture clinic. Past medical history: Unremarkable Past surgical History: None Social history: Non -smoker, no illicit drug abuse, no alcohol abuse Physical examination reveals the following: - No open wounds - Ecchymosis and swelling to the dorsum of the left foot - Swelling to the left foot - There is no pallor, or cyanosis - Gross sensations all intact to the left foot - Gentle palpation reveals tenderness and crepitus to the 2nd metatarsal Plain film radiographs results are below. What is your assessment? Closed displaced fracture of the 2nd metatarsal, and non- displaced fracture of the proximal 3rd phalanx Closed displaced fracture of the 3rd metatarsal Non displaced fracture of the 3rd proximal phalanx, and callous formation 2nd metatarsal Lisfranc dislocation with diasthesis

Correct answer: (A) Closed displaced fracture of the 2nd metatarsal, and non- displaced fracture of the proximal 3rd phalanx. Explanation: The two views present an Anterior and Medial oblique views of the same patient, minimal soft tissue swelling can be observed, with an extraarticular fracture in the mid-diaphysis of the 2nd metatarsal, with plantar dislocation, oblique in nature, no comminution, there is subsequent shortening of the 2nd metatarsal, as the metatarsal protrusion index disrupted. The second fracture is present on the 3rd proximal phalanx, incomplete, non-displaced and extraarticular, no comminution. (B) There are two fractures, one on the 2nd metatarsal, and the second on the 3rd digit, on the proximal phalanx, so this choice will be incorrect. (C) The first part is correct, but there is no callus formation on the 2nd metatarsal This will signify a healing fracture line, the second part of the answer renders the choice incorrect. (D) There is no presence of Lisfranc fracture or dislocation on the plain films, and the clinical exam did not indicate any tenderness over the Lisfranc joint which makes this choice incorrect.

Determining the need for a Dwyer calcaneal osteotomy with a cavovarus deformity utilizes the: (A) Coleman block test. (B) The single heel raise test is a test of the posterior tibial tendon. (C) The double heel raise test is a test of bilateral posterior tibial tendon function. The heels should normally invert when the patient rises. (D) Manual muscle testing is not sensitive nor specific to determine the need for a Dwyer calcaneal osteotomy.

Correct answer: (A) Coleman block test. Explanation: The Coleman block test evaluates hindfoot flexibility and pronation of the forefoot. The initial deformity in the cavovarus deformity is in the forefoot, followed by subsequent changes in the hindfoot. The Coleman block test is performed by placing the patient's foot on a wood block that is 2.5 to 4 cm thick, with the heel and lateral border of the foot on the block and bearing full weight. The 1st, 2nd, and 3rd metatarsals are allowed to hang freely into plantar flexion and pronation. The test is based on the premise that there is fixed flexion of the 1st metatarsal. The test negates the effect that the forefoot may have on the hindfoot in stance, and if the heel varus corrects while the patient is standing on the block, the hindfoot is considered flexible. If it is rigid, then surgical correction of both forefoot and hindfoot are required. If the subtalar joint is supple and correct with the block test, then surgical procedures may be directed to correction of forefoot pronation, which is usually due to plantar flexion of the first metatarsal.

With any toe surgery, adequate vascularity must be ensured before proceeding. Vascular injury to the toe is a possibility and needs to be discussed with the patient prior to surgery. You are planning a flexible contracted painful 2nd digit surgical correction on a patient and are discussing the blood supply to the 2nd digit. What provides blood supply to the 2nd metatarsal? Please be specific. A. Common digital arteries 3 and 4 B. Common digital arteries 2 and 3 C. Proper digital arteries 3 and 4 D. Proper digital arteries 2 and 4

Correct answer: (A) Common digital arteries 3 and 4. Explanation: Common digital artery 3 is a termination of the 1st metatarsal artery from the first dorsal metatarsal artery a terminal branch of the dorsalis pedis artery. It will supply the medial aspect of the 2nd metatarsal, 2nd toe. The common digital artery 4 is a termination of the 2nd dorsal metatarsal artery which arises between the second and third metatarsal bases and passes distally within the 2nd interspace. The common digital artery supplies the lateral side of the second toe.

During which phase of gait is subtalar joint pronation passive? A. Contact phase B. Midstance C. Propulsion D. Swing phase

Correct answer: (A) Contact phase. Explanation: Pronation of the subtalar joint during contact phase is a passive event. Plantarflexion of the ankle joint is resisted by contraction of the pre-tibial muscles, resulting in supination of the longitudinal axis of the midtarsal joint, which in open kinetic chain supinates the STJ. When the STJ is supinated, the calcaneus is inverted relative to the floor. However, when placed in contact with the floor, ground reactive forces place an everting force on the calcaneus resulting in pronation. During both midstance and propulsion, the STJ is being supinated in normal gait.

In order to derotate the aductovarus fifth digit below what orientation should the lenticular incision be made? A. Distal medial to proximal lateral B. Distal lateral to proximal medial C. Dorsal linear D. Dorsal transverse

Correct answer: (A) Distal medial to proximal lateral. Explanation: In order to derotate the adductovarus fifth digit an oblique incision should be made from distal medial to proximal lateral. The distal medial to proximal lateral elliptical incision when closed will promote a valgusization of the digit and derotate the deformity. (B) The opposite orientation, distal lateral to proximal medial, would provide a further varus de- formity. The lateral to medial and medial to lateral are one in the same and would simple provide some correction in the sagittal plane. (C) A dorsal linear incision would have very little corrective power for the adductovarus fifth digit. (D) A dorsal transverse incision would have very little corrective power for the adductovarus fifth digit.

To derotate an adductovarus fifth digit, how is the lenticular incision made? A. distal medial to proximal lateral B. distal lateral to proximal medial C. lateral to medial D. medial to lateral

Correct answer: (A) Distal medial to proximal lateral. Explanation: When performing a 5th digit proximal interphalangeal joint arthroplasty, resection of the head of the proximal phalanx results in correction of the frontal plane deformity. The frontal plane varus rotation correction can be enhanced with a derotational skin plasty. Two semi-elliptical incisions are placed over the area of bone resection oriented on an axis to derotate the fifth toe in the plane desired with closure of the incision, proximal lateral to distal medial. A wedge of skin removed should be 3-4mm wide.

What is the most frequent incision placement in the anatomic dissection of the first metatarsophalangeal joint? Dorsomedial aspect of the metatarsophalangeal joint, medial to the extensor hallucis longus tendon Dorsolateral aspect of the of the 1st insterspace Semi-elliptical incision along the first metatarsophalangeal joint Dorsolateral aspect of the metatarsophalangeal joint, lateral to the extensor hallucis longus

Correct answer: (A) Dorsomedial aspect of the metatarsophalangeal joint, medial to the extensor hallucis longus tendon. Explanation: Although the first metatarsophalangeal joint can be exposed through many different surgical approaches, a universal incision for hallux valgus surgery can be oriented along the dorsal medial aspect of the primary deformity. The incision begins proximally about the midshaft level of the dorsal medial aspect of the metatarsal, extends distally to the dorsal medial aspect of the metatarsal head and ends just proximal to the hallux interphalangeal joint, medial to the extensor hallucis longus tendon. The incision usually lies between the extensor hallucis longus tendon and the neurovascular structures.

If a patient is designated an ASA class IIE in the preoperative assessment what does that mean? A. Emergency surgery on an individual with minor systemic disease B. Difficult Emergence; individual with minor systemic disease C. Emergency surgery on individual with a non incapacitating severe disease D. Difficult Emergence; individual with a non incapacitating severe disease

Correct answer: (A) Emergency surgery on an individual with minor systemic disease. Explanation: In the ASA physical status classification, a class I patient is described as normal, healthy individual. A class II patient is described as one with mild systemic disease such as well- controlled diabetes. A class III patient is one who has a severe non-incapacitating systems disease, such as a severe diabetic. Class III patients have a 4% mortality. Class IV is described as a severe incapacitating disease that is a constant threat to life. An example patient that would fit this classification would be a patient with cardiovascular and renal disease with renal Impairment. Mortality for this class is 25%. Class V is described as a moribund patient who is not expected to survive 24 hours with or without operation. An example of this type of patient is one with a rupture aortic aneurysm. Lastly, there is a class VI which is best described as a brain- dead organ donor. One final note is that if a surgery is an emergency, the letter E accompanies the class.

The most frequently utilized surgical procedure in the treatment of the pes valgus deformity in CP is: A. Evans lateral column lengthening B. Arthroereisis C. Medial column fusion D. Triple arthrodesis

Correct answer: (A) Evans lateral column lengthening. Explanation: The Evans is probably the most widely utilized procedure for the CP pes valgus deformity, but is not suitable for all patients. Several planes of deformity can be corrected at the same time, including abduction of the forefoot and reduction of the valgus heel. In Drennan's The Child's Foot & Ankle, H. Kerr Graham authored the chapter on cerebral palsy, and found a high failure rate in patients classified as Gross Motor Function Classification System (GMFCS) III or IV. Thus, his indications for the Evans is a patient classified as GMFCS I or II, with a mild to moderate pes abductovalgus deformity; age 6-10. Following the Evans, re-check for equinus, as the foot structure has changed, and a gastrocnemius recession may be required. A medial column fusion, like a Hoke or Miller, will primarily correct the sagittal plane component, but not abduction or heel valgus. Although not a choice above, the subtalar fusion is another commonly utilized procedure, especially with a GMFCS III or IV patient. The deformity can be moderate to severe, but must be passively correctable in equinus and inversion. Again, re-check the status of equinus following the fusion. There is a subset of patients in whom the lateral column is very, very short, and the talus so vertical, that both procedures are used in combination. The triple arthrodesis is reserved for GMFCS IV or V non-ambulators, but there is concern about the effects of fusing the rearfoot, on distal joints. There are few long-term studies regarding outcomes in the CP population. Regarding arthroereisis, the failure rate is high, and revision surgery needed in many cases. Arthroereisis is not indicated for the CP flatfoot deformity.

Any factor or characteristic that may explain or predict the presence of an outcome is referred as _____________? A. Exposure B. Outcome C. Result D. Likelihood ratio

Correct answer: (A) Exposure. Explanation: Exposure refers to any factor or characteristic that may explain or predict the presence of an outcome. Exposure is often compared to outcomes and constitutes a large area of study within epidemiology. Common "exposures" in medical research include the use of certain medications, smoking, and blood pressure. The term "outcome" is used to refer to the factor that is being predicted. The outcome is often a disease but can be any clinical outcome such as cholesterol levels, medication use, or vaccination.

Mrs. Jones presents with a complaint of radiating pain along her medial ankle and into the arch of her left foot. She states the pain is constant and becomes more intense with ambulation and activity. Physical exam reveals intact pedal pulses. Tenderness on palpation along the retromalleolar area extending into the arch. Mrs. Jones reports these problems began about eight weeks ago. She also reports past surgical history of tarsal tunnel release on the same foot six months prior. She states that for the first few months after tarsal tunnel surgery her initial symptoms subsided. What is the most likely cause of Mrs. Jones symptoms? A. External scarring of the tibial nerve B. Tendinopathy of the posterior tibial tendon C. Inadequate release of the tibial nerve D. Baxter's neuritis

Correct answer: (A) External scarring of the tibial nerve. Explanation: The most likely cause of Mrs. Jone's symptoms is from external scarring of the tibial nerve. The clinical scenario described above is classic for this complication. When external scarring is present and symptomatic, it typically presents after a period of time of temporary short-term relief for a few weeks to months. The initial release was effective however as scar tissue continued to form post operatively it tethered the nerve causing mechanical circumferential constriction, scarring and potential ischemia. (B) Posterior tibial tendinopathy is not the cause of symptomatology described above. Given the history of temporary relief after the initial tarsal tunnel release, the notion of extra-neural scar involvement is much more likely. (C) Inadequate release of the tibial nerve is not likely as there was, indeed, temporary short-term relief of Mrs. Jone's initial tarsal tunnel symptoms indicating adequate release. (D) Baxter's neuritis is not the cause of symptomatology described above. Given the history of temporary relief after the initial tarsal tunnel release, the notion of extra-neural scar involvement is much more likely.

A 26 year old female ballet dancer presents to the clinic complaining of sharp pain under her first metatarsophalangeal joint for approximately eight months. The following radiograph was performed at her visit. What would be the most appropriate surgical intervention? A. Fibular sesamoidectomy B. Open reduction and internal fixation of the fibular sesamoid C. Incorporation of an external bone stimulator D. Resection of the proximal fracture fragment

Correct answer: (A) Fibular sesamoidectomy. Explanation: This is a chronic, comminuted fracture of the fibular sesamoid. The sclerotic margins are consistent with avascular necrosis. The most appropriate procedure would be a complete fibular sesamoidectomy. As the bone is avascular open reduction and internal fixation would be fruitless. With the AVN changes, a bone stimulator would have no effect. In addition, resection of just the proximal fragment would not likely eliminate the patient's symptoms. The fibular sesamoidectomy is not an uncommon adjunctive procedure in a hallux valgus repair. Generally it is well tolerated and functional.

Excision of a second intermetatarsal neuroma results in an abducted third digit. Damage to which intrinsic muscles would produce this deformity? A. first plantar interosseus B. third dorsal interosseus C. second lumbrical D. flexor digitorum brevis

Correct answer: (A) First plantar interosseus. Explanation: The plantar interosseus inserts at the medial aspect of the third digit, in the second interspace. It functions to adduct the 3rd digit. The third dorsal interosseus inserts in to the lateral aspect of the 3rd digit. It functions to abduct the 3rd digit. If the 1st plantar interosseous is cut, the third dorsal interosseus muscle abducts unopposed and therefore the third digit is abducted. If the second lumbrical was damaged the 3rd digit would be flexed, since the second lumbrical extends the 3rd digit. If the flexor digitorum brevis was damaged the 3rd digit would be extended, since the flexor digitorum brevis flexes the 3rd digit.

A patient presents to a podiatrist for evaluation of a toe that lacks structural stability and moves freely in all planes. The patient relates to a history of previous surgery on the toe to correct a hammertoe deformity. What is the term given for this common complication of hammer digit correction? A. Flail toe B. Floating toe C. White toe D. Hammer toe

Correct answer: (A) Flail toe. Explanation: Flail toe is defined as a toe that lacks structural integrity or stability. It is commonly caused from over zealous resection of the proximal phalangeal head during hammer toe correction. The excessive bone resection can cause the lack of stability by either removing the supporting skeleton or removing the function or "unloading" the flexors and extensors by shortening the digit. Surgical correction options include syndactilization, implant arthroplasty, amputation, and autogenous bone graft to lengthen the digit.

Which of the following options below is an indication for a Hibbs tendosuspension? A. Flexible forefoot equinus without claw toes B. Rigid forefoot equinus with claw toes C. Rigid cavus deformity without claw toes D. Clawtoes secondary to flexor substitution

Correct answer: (A) Flexible forefoot equinus without claw toes. Explanation: The Hibbs procedure is the transfer of the extensor digitorum longus tendons into the region of the third metatarsal base or lateral cuneiform. The distal slips of the extensor digitorum longus are tenodesed into the corresponding brevis tendons. The fourth and fifth extensor digitorum longus slips are tenodesed into the fourth extensor digitorum brevis. The Hibbs procedure is performed primarily to release the retrograde buckling at the metatarsophalangeal joints. The plantarflexory forces causing the flexible forefoot equinus and the dorsiflexory power of the foot is enhanced. The procedure is indicated for flexible forefoot equinus with or without claw toes and flexible caves deformity with claw toes secondary to extensor substitution.

The sesamoids serve as a pulley to increase the lever arm of this muscle in stabilizing the hallux against the ground? A. Flexor hallucis brevis B. Flexor hallucis longus C. Adductor hallucis D. Abductor hallucis

Correct answer: (A) Flexor hallucis brevis. Explanation: The sesamoids serve as a pulley to increase the lever arm of the flexor hallucis brevis in stabilizing the hallux against the ground. Plantar stability of the hallux against the ground in propulsion is provided by the flexor hallucis longus and brevis. Unassisted dorsiflexion of the first metatarsophalangeal joint has been found to be on average, 77 degrees with precisely identified surface landmarks. Average assisted dorsiflexion is 82 degrees. In dynamic studies, necessary dorsiflexion of the first metatarsophalangeal joint during gait has been found to be 50 to 60 degrees.

Which of the below is true of congenital pes valgus deformity? A. Forced plantarflexion view is used for diagnosis. B. Decrease in talocalcaneal angle. C. On lateral radiograph the talus and tibia are perpendicular in orientation. D. Metatarsals tend to be plantarly displaced in regard to talus.

Correct answer: (A) Forced plantarflexion view is used for diagnosis. Explanation: In a normal foot, a forced plantarflexion view would allow for lining up of the metatarsals and the talus. In a congenital pes valgus deformity, doing the same test does not allow for lining up as above and the talar head increases its declination. (B) As in all flat foot deformities, there is an increase in the talocalcaneal angle and thus a decrease of articulation between talus and navicular. (C) On lateral x-ray, the tibia and the talus become parallel to one another due to the excessive plantarflexion of the talus. The tibio talar angle can be used to measure this relationship (D) Metatarsals are dorsally displaced in regard to the talus.

Which of the following will do the most to increase chances of successful skin graft integration or "take."? A. graft thickness of .008" - .012" B. graft thickness of .012" - .016" C. stent type dressing for 48 hours after grafting D. inadequate immobilization

Correct answer: (A) Graft thickness of .008" - .012". Explanation: Skin graft integration or "take" depends on a multitude of factors. Skin graft failure has a higher incidence as the graft thickness increases. Grafts commonly are classified by thickness as follows: Thin .008" - .012"; Intermediate .012" - .016"; Thick 0.16" - 0.20". It is important to stabilize and create even contact of the graft with the host tissue by use of a stent- type dressing. It is standard of care to leave a stent-type dressing on for 5 days. Furthermore, strict immobilization is paramount in graft success. Motion of the graft will guarantee failure. Poor immobilization and/or contact will lead to failure. In conclusion, a graft of .008" - .012" in thickness will have the best chance of success integration or "take."

The most common complication following preaxial polydactyly repair is: A. Hallux varus B. Forefoot splaying C. Residual valgus deformity D. Extensive scarring

Correct answer: (A) Hallux varus. Explanation: Polydactyly is a duplication of the digits and in the foot is located preaxially in 15% of cases. A short first metatarsal can contribute to the varus deformity. Abductor hallucis tenotomy or lateral capsular/adductor hallucis reefing may be necessary. First to second toe syndactyly or a K-wire across the joint are other options. Forefoot splaying may be seen with central polydactyly repair. Residual valgus deformity is seen with postaxial polydactyly repair.

What is duty? A. The existence of a doctor-patient relationship. B. The express contract between a doctor and physician stating the desires of the patient. C. A tort D. Negligence

Correct answer: (A) The existence of a doctor-patient relationship. Explanation: Duty is defined as the existence of a doctor-patient relationship. The most common way a duty attaches is by mutual consent. Mutual consent is a principle of contract law. In contract law, one party performs or promises to perform something that is of value to another party. If one party fails to complete his promise, that party is in breach of their contractual responsibility and the aggrieved party may seek money damages in court as a remedy.

Walking is a series of repetitive motions strung together. Each set of repetitive motions is term a gait cycle. How is gait cycle defined? A. Heel contact to following heel contact of the same limb B. Heel contact to heel contact to the contralateral limb C. Heel off to heel off the same limb D. Heel off to heel off of the contralateral limb

Correct answer: (A) Heel contact to following heel contact of the same limb. Explanation: One gait cycle is measured from floor contact of the heel to the following heel contact of the same limb. Each gait cycle is divided into two parts. The swing phase and the stance phase. One gait cycle lasts approximately 1 second, with 0.6 second in the stance phase.Correct answer: (A) Heel contact to following heel contact of the same limb. Explanation: One gait cycle is measured from floor contact of the heel to the following heel contact of the same limb. Each gait cycle is divided into two parts. The swing phase and the stance phase. One gait cycle lasts approximately 1 second, with 0.6 second in the stance phase.

Which of the following is a risk factor for infection in forefoot trauma wounds? A. High energy B. Early stabilization C. Debridement D. Early wound exposure

Correct answer: (A) High energy. Explanation: High-energy mechanisms, high degrees of fracture comminution, gross contamination, and extreme soft toss loss are all risk factors that lead to infection. Additional risk factors leading to infection include increased age, obesity, steroid use, immunosupression, malnutrition, diabetes, shock, blood loss, multiple fractures, poor fixation, premature closure, delay in soft tissue coverage, delay in antibiotics, prolonged surgery time, multiple wound exposures and remote site of infection. (B) Early fracture stabilization is a factor that decreases infection rate and not one that increase it. (C) Early and multiple debridements if necessary are factors that decreases infection rate and not one that increase it. (D) Early wound exposure is a factor that decreases infection rate and not one that increase it.

Stabilization of fracture fragments through interfragmentary compression imparts preload across fracture surfaces and prevents which one of the following action: A. Interfragmentary motion B. Osteogenic formation C. Restoration of endosteal blood flow D. Restoration of periosteal blood flow

Correct answer: (A) Interfragmentary motion. Explanation: Stabilization at a fracture site can be achieved through application of interfragmentary compression. Compression in and of itself is not osteogenic. Compression imparts preload across fractured surfaces and prevents interfragmentary motion. Compression also generates friction when two fractured ends with multiple irregularities are reapproximated in a near-anatomic fashion. Employing the concept of compression through application of internal fixation devices has been shown to reduce callus production, yet it facilitates bone formation within a fracture.

Understanding that surgery is critical for diabetic patients, the American College of Endocrinology has recommended the following approach on maintaining blood glucose during the intraoperative and postoperative periods. Please pick the one that applies. A. Intraoperative glucose < 200mg/DL, and < 150 mg/dL postoperatively B. Maintaining glucose levels < 100 C. Tight glycemic control of 70-90mg/dL D. Intraoperative glucose of < 80 mmg/dL

Correct answer: (A) Intraoperative glucose < 200mg/DL, and < 150 mg/dL postoperatively. Explanation: The American college of endocrinology recommends maintaining blood glucose less than 200 mg/dL intraoperatively and less than 150 mg/dL postoperatively, and avoid levels less than 80 mg/dL. Recent data suggests that extremely tight glycemic control of 80-110 mg/dL in critically ill patients may not be beneficial. Patients on regular insulin will require change in their scheduled dosing depending on how long they have been NPO before surgery.

One of the most important surgical concerns during biologic fracture fixation is which of the following: A. Limitation of operative exposure B. Application of temporary fixation C. Avoidance of infection D. Acceleration of the rate of fusion at the fracture site and early return to activities

Correct answer: (A) Limitation of operative exposure. Explanation: In order to maintain length and alignment, a plate fixation to bridge a badly comminuted segment of bone may be required. Biologic fixation is recommended in association with comminuted fracture fragments when a lag screw would result in fragment devitalization, or when there is a gap in the fracture and a plate will be used to maintain the relative position of the fragments. This fixation is not stable, and requires adjunct methods to maintain all the soft tissue attachments and blood supply of the intervening comminuted fragments, since union will depend on the formation of a bridging callus rather than primary bone union. In these situations, techniques of indirect reduction are particularly useful. The following are key surgical concerns during biologic fracture fixation: 1- repositioning and realignment of the fracture, preserving soft tissue attachments 2- keeping comminution fragments out of the mechanical construct. If a fragment is incorporated into the construct it should have a vascular supply 3- use of biocompatible materials is required 4- maintenance of minimal contact between bone and implant 5- limitation of operative exposure when possible

In a diabetic patient, metform should be withheld 48 hours pre-operatively and can be restarted 48-72 hours after surgery. What should be documented prior to resuming Metformin? A. Normal renal function B. Ability to tolerated PO intake C. Lack of lactic acidosis D. Postoperative glucose levels of less than 150 mg/dL

Correct answer: (A) Normal renal function. Explanation: Metformin generally should be withheld 48 hours prior to surgery and withheld 48- 72 hours post-surgery. Renal function should be normal prior to resuming therapy. Glucose levels should be checked regularly and elevated levels of > 180 mg/dL can be treated with intermittent doses of short-acting insulin.

During the gait cycle, among the elements used to define it, cadence is one of them. What defines cadence in gait cycle? A. Number of steps per minute B. Distance between two consecutive contacts of the same foot C. Heel contact of one limb to heel contact of the opposite limb D. Heel contact of one limb to heel contact of the same limb

Correct answer: (A) Number of steps per minute. Explanation: Cadence, or step rate, is defined as the number of steps per minute when a person walks. Most people will select a natural or free cadence. The average cadence ranges from 101 to 122 steps per minute, Women have a faster cadence than men (117 vs. 111 steps per minute). Cadence generally slows down with age. It should be noted that as cadence increases, both stance and swing time decrease, but stance phase decreases 3.5 times as rapidly as swing phase. In addition, an increase in cadence shortens both double support periods. Murray et al in a study of older men, included subjects up to 87 years old and noted that for those older than 65, both cadence and stride length decreased.

Cerclage wire is an indication for fixating fractures in patients with the following comorbidity: A. Osteoporotic bone B. Peripheral neuropathy C. Diabetes D. Complex regional pain syndrome

Correct answer: (A) Osteoporotic bone. Explanation: Cerclage may be the only option for stabilization of fractures or osteotomies in patients with osteoporotic bone, in which screw purchase is inadequate and intramedullary fixation may not be indicated. Defined as the as the encircling of a part with a ring or loop. Cerclage involves the circular application of various diameters of malleable wire to stabilize fractures not amenable to other forms of internal fixation. Cerclage can be useful when exposure to a fracture is limited, yet interfragmentary compression is required for healing. Cerclage may be the only option for stabilization of fractures or osteotomies in patients with osteoporotic bone, in which screw purchase is inadequate and intramedullary fixation may not be indicated. A modification of the cerclage technique involves the transosseous or intraosseous passage of malleable wire. This technique allows for interfragmentary compression to be generated across transverse osteotomies or fractures that are situated in a juxtaarticular fashion.

Digital radiography can be conducted at higher ______, resulting in lower patient dose. A. Peak kilovoltage (kVp) B. Milliamp Seconds (mAs) C. Distance D. Resolution

Correct answer: (A) Peak kilovoltage (kVp). Explanation: Kvp: Peak kilovoltage (kVp) is the maximum voltage applied across an X-ray tube. It determines the kinetic energy of the electrons accelerated in the X-ray tube and the peak energy of the X-ray emission spectrum. Increasing kVp allows the mAs to be lowered, resulting in a lower patient dose. The Higher the kVp, the lower the contrast. In essence, the kVp controls the number of shades of grey, and the mAs controls how dark they are.

When performing ORIF for a talar neck fracture, the strongest screw orientation is: A. Posterolateral B. Posteromedial C. Anteromedial

Correct answer: (A) Posterolateral. Explanation: The lateral aspect of the head and neck of the talus is the most dense portion of the bone. Directing a screw from posterolateral to anteromedial allows for placement perpendicular to the fracture line. This approach avoids the TNJ and blood supply through the dorsal part of the talus. This construct is stronger than the anterior to posterior approach. The disadvantage is that the surgeon cannot directly visualize the fracture line. Dissection is between the Achilles tendon and the peroneal tendons. (B) The posteromedial approach is avoided simply because the approach is too dangerous; the neurovascular bundle is sitting right there. (C) The main disadvantage to this approach, which is made medial to the tibialis anterior tendon at the talar neck, is disruption of the medial blood supply to the talus. Also, the medial side provides narrow limits for screw placement. Thirdly, the screws must pass eccentrically through the dorsal aspect of the neck, and thus oblique to the fracture line. This approach does provide excellent exposure, however. If there is tibialis posterior tendon interposition, this approach is indicated. (D) This approach preserves the blood supply, utilizing an Ollier type incision. Exposure is poor, and does not allow access to the tibialis posterior tendon, if it is interposed.

Which of the following is the method of metabolism for Procaine? A. Pseudocholinesterase B. Kidney Excretion C. Liver D. Respiratory

Correct answer: (A) Pseudocholinesterase. Explanation: Pseudocholinesterase-is an ester local anesthetic, is metabolized in the plasma by the pseudocholinesterase through hydrolysis into para-amino benzoic acid (PABA), which is then excreted by the kidneys into the urine. Amide anesthetics are the other class of local anesthetics, which are metabolized by the liver.

Twenty-one hours after an Austin bunionectomy under general anesthesia, the patient develops a temperature of 102 degrees Fahrenheit. What is the most likely cause? A. pulmonary atelectasis B. postoperative wound infection C. superficial thrombophlebitis D. constipation

Correct answer: (A) Pulmonary atelectasis. Explanation: The most common cause for postoperative fever within the first 24 hours is pulmonary atelectasis secondary to general anesthesia. After 24-48 hours the most common cause of post-op fever is UTI, dehydration, and constipation. After 72 hours a post-op fever is most commonly caused by a DVT/superficial thromophlebitis, wound infection or a drug reaction. Therefore in this question after 24 hours post-op, a fever would most likely be caused by pulmonary atelectasis.

The AO group described in their Manual of Internal Fixation four biomechanics principles in order to achieve early return to function of the injured limb. Those four principles are: Restoration of anatomy; stable fracture fixation; preservation of blood supply; early mobilization of the limb and patient Atraumatic operative technique; accurate anatomic reduction; rigid internal compressive fixation; early immobilization Accurate anatomic reduction; internal splint age fixation; early immobilization; targeted aggressive passive range of motion therapy Percutaneous surgical approach; image-guided reduction, rigid internal compression fixation, avoidance of soft tissue damage

Correct answer: (A) Restoration of anatomy; stable fracture fixation; preservation of blood supply; early mobilization of the limb and patient. Explanation: This answer to this question is a direct statement and huge fundamental concept in the AO Manual of Internal Fixation. AO principles: • fracture reduction and fixation to restore anatomical relationships • fracture fixation providing absolute or relative stability as the "personality" of the fracture, the patient, and the injury requires • preservation of the blood supply to soft tissues and bone by gentle reduction techniques and careful handling • early and safe mobilization and rehabilitation of the injured part and the patient as a whole.

During contact phase, at heel contact, what causes rapid plantarflexion of the forefoot? A. Vertical force from ground reactive forces B. Anteroposterior shear force C. Stride length D. Cadence of the gait

Correct answer: (A) Vertical force from ground reactive forces. Explanation: At heel contact, the ground reaction force (vector) is directed behind the ankle joint, which applies a vertical force on the heel, resulting in rapid plantarflexion of the forefoot. The heel serving as a rocker expedites this action. This rapid plantarflexion occurs until the pretibial muscles (eccentrically contracting) gain sufficient strength to effectively resist this movement. The slow deceleration action of the muscles serves to absorb some of the ground reaction force. Once this occurs, the movement is slowed, and even weight distribution takes place.

A 27 year old female patient presents to your office for a pre-operative evaluation for bunionectomy right foot. X-rays are taken and below are the measured angles. Which of the following procedure choices are the best? IM Angle: 16 degrees PASA: 14 degrees Sesamoid Position: 5 Hallux Abductus Angle (HAA): 13 Hallux Abductus Interphalangeal Angle (HAI): 7 Hypermobility- None Noted A. Reverdin and Closing base wedge bunionectomy B. Austin and closing base wedge bunionectomy C. Reverdin Green D. Lapidus

Correct answer: (A) Reverdin and Closing base wedge bunionectomy. Explanation: In going through a question like this, it is important to take the x-ray angles one at a time and determine whether they are normal or abnormal. In this case there is severe elevation of the IM angle, elevated PASA, abnormal sesamoid position, normal HAA, normal HAI and no hypermobility. Thus in this case you need to find a choice that will correct for high PASA and high IM angle. The Reverdin bunionectomy is primarily used for an increase in the PASA but used in conjunction with a base wedge bunionectomy, the IM angle can be fixed as well and thus this is the best answer choice here. A Lapidus fusion is primarily used for hypermobility of the 1st ray with elevated IM angle. The Reverdin Green alone would only correct for PASA and not for IM angle. The regular austin bunionectomy would not be paired with closing wedge bunionectomy as they both primarily correct IM angle and not PASA which is needed in this case.

A patient presents complaining of nagging persistent right ankle pain and swelling. Radiographic examination is performed of both ankles and an abnormality is noted on the mortise views. On the left ankle there is a 2 mm medial clear space while on the right there is a 5 mm medial clear space. What is the significance of this finding? A. Right ankle syndesmotic instability B. Left ankle degenerative changes C. Right tibiotalar dislocation D. Left ankle syndesmotic instability

Correct answer: (A) Right ankle syndesmotic instability. Explanation: A medial clear space that is greater than 1-2 mm as compared to the contralateral ankle is indicative of syndesmotic instability. To be more specific the increase in the medial clear space indicates a lateral shift of the talus. (B) No radiographic evidence of degenerative changes was noted and with the history of ankle sprains, pain in the right ankle and the increased medial clear space the diagnosis of right ankle syndesmotic instability id justified. (C) No evidence right ankle dislocation is present. (D) There is no evidence of left ankle syndesmotic instability.

Cerebral Palsy (CP) can be classified by movement disorder. This knowledge is important in pre- operative planning, as some movement disorders have less predictable surgical outcomes. The most common movement disorder, with the most predictable surgical outcomes is: A. Spastic B. Dyskinetic C. Mixed D. Ataxia

Correct answer: (A) Spastic. Explanation: Movement disorder classification systems are not as reliable as, say, the GMFCS. That being said, it helps with prognosis. The spastic type is most common (60-80%), and gives the most favorable prognosis. It is usually associated with pyramidal tract lesions. Dyskinetic (10-25%) demonstrates involuntary limb movement. There are less fixed deformities than the spastic type. It is associated with extrapyramidal lesions. Surgical outcomes are less predictable. Mixed (10-25%) are common, with affectation of both the pyramidal and extrapyramidal systems. Surgical outcomes are less predictable. Ataxic (2-5%) is rare. Fixed contractures do not develop and surgery is usually not necessary.

What are the most common pathogens found in infected and compromised implants? A. Staphylococcus epidermidis and Staphylococcus aureus B. Pseudomonas and Staphylococcus epidermidis C. Pseudomonas and Staphylococcus Aureus D. Pseudomonas, staphylococcus aureus and staphylococcus epidermidis

Correct answer: (A) Staphylococcus epidermidis and Staphylococcus aureus. Explanation: The use of implants increases the risk of infection because an implant provides a potential site for bacterial colonization that may not make the organism as accessible to the body's normal defense systems. Implants also create a nonvascularized space in the body that increases the potential for late infections. The first phase of infection associated with foreign body generally occurs on the material itself, and it involves the bacterial attachment to these artificial surfaces. All patients with these devices share increased risk of infection because a small number of microorganisms is capable of inducing this complication, the more common flora being staphylococcus epidermidis and staphylococcus aureus.

During contact phase, what is the position and motion of the longitudinal midtarsal joint axis? A. Supinated and supination B. Pronated and pronation C. Pronated and supination D. Supinated and pronation

Correct answer: (A) Supinated and supination. Explanation: The longitudinal axis of the midtarsal joint supinates owing to contraction of the anterior tibial and extensor digitorum longus, which insert medial to the axis. Since supination of the longitudinal axis of the midtarsal joint is accompanied by subtalar pronation, joint instability occurs, and adaptation to the floor surface is expedited. An inverted attitude about this axis permits normal excursion of the subtalar joint. position motion LMJ supinated supination OMJ pronated pronation

Which type of suture is broken down by the action of hydrolysis when implanted in the body? A. Synthetic absorbable B. Synthetic nonabsorbable C. Natural absorbable D. Natural nonabsorbable

Correct answer: (A) Synthetic absorbable. Explanation: Synthetic absorbable sutures are broken down by the action of hydrolysis when exposed to water in the soft tissues. The process of hydrolysis causes a relatively mild soft tissue reaction as compared with that of enzymatic degradation seen with natural absorbable sutures.

Which of the following is true of Talar Neck Fractures? The Hawkins Sign is a good prognostic indicator. AVN is the least common complication following talar neck injuries. Hawkin's Type 2 injury is a displaced vertical fracture of the talar neck with navicular dislocation. Talar fractures are never treated surgically.

Correct answer: (A) The Hawkins Sign is a good prognostic indicator. Explanation: Talar fracture can occur commonly after motor vehicle accidents. The talus has a very tricky blood supply and it is therefore very common to develop an avascular necrosis after this fracture pattern. The Hawkins sign is a lucency that is seen in the dome of the talus after a fracture (6 weeks down the line) which is a good prognostic indicator meaning that the bone has stayed vascularized. Talar fractures are sometimes treated surgically depending upon the displacement and patient's status. The Hawkin's classification system is the main classification for this injury. A type 1 is a non displaced vertical fracture of the talar neck. A stage 2 is a displaced vertical fracture of talar neck with subtalar joint dislocation/subluxation. A stage 3 injury is the same as stage 2 but there is also an ankle dislocation/subluxation and stage 4 is the same as stage 3 but with a navicular dislocation as well.

Which of the following is true of a forefoot varus deformity? The forefoot is inverted to the rearfoot when the subtalar joint is in neutral and the midtarsal joint is maximally pronated and locked. In fully compensated forefoot varus it is uncommon to see Tailors bunions and adducto-varus rotation of toes. In an uncompensated forefoot varus, the medial column can dorsiflex to reach the ground. It is common to see an posterior break in the cyma line with fully compensated forefoot varus.

Correct answer: (A) The forefoot is inverted to the rearfoot when the subtalar joint is in neutral and the midtarsal joint is maximally pronated and locked. Explanation: A forefoot varus deformity is one of the most common deformities seen on biomechanical evaluation. In this deformity, the forefoot is inverted to the rearfoot when the subtalar joint is in neutral and the midtarsal joint is maximally pronated and locked. (Hence A is the correct choice) In a fully compensated forefoot varus, it is very common to see adducto-varus rotation of the toes as well as Tailors bunions and regular bunions. If the forefoot varus is uncompensated, it is common for the 1st ray to plantarflex down to the ground to help bring the medial column to the ground. A fully compensated forefoot varus shows all the typical radiographic findings for a pronating foot including anterior break in the cyma line and increase in talocalcaneal angle.

Listed below are statements regarding how diabetes mellitus can affect wound healing. Out of the statements below which is not correct? The small vessels required for blood supply and nutrient transport for wound healing are in a hyper-dilated state. Hyperglycemia of greater than 200 mg/dl decreases phagocytosis and decreases killing of bacteria by leukocytes. A deficiency in insulin may lead to a decrease in collagen production and lessened wound tensile strength, as insulin is required to enhance both granulation tissue and collagen formation. The wound exudate in diabetics is found to have substandard numbers of leukocytes and exhibits a poorly defined inflammatory response.

Correct answer: (A) The small vessels required for blood supply and nutrient transport for wound healing are in a hyper-dilated state. Explanation: The small vessels required for blood supply and nutrient transport for wound healing are dysfunctional or occluded in diabetics which can significantly interfere with wound healing. Hyperglycemia of greater than 200 mg/dl decreases phagocytosis and decreases killing of bacteria by leukocytes. Additionally, the wound exudate in diabetics has been found to have significantly decreased numbers of leukocytes and exhibits a poorly defined inflammatory response. A deficiency in insulin may lead to decreased collagen production and decreased wound tensile strength, as insulin is needed to enhance granulation tissue and collagen formation.

A failed or delayed local anesthetic block on a severely infected paronychia is associated with which of the following? The soft tissues in the area of infection are acidic which fractionates the drug largely to its cationic form creating a marked decrease in transport across the cellular membrane. The soft tissues in the area of infection are basic which fractionates the drug largely to its cationic form which has a marked decrease in transport across the cellular membrane. The uncharged portion of the drug in the interior of the cell has stronger binding action and therefore greater sodium channel inhibition. A higher pKa allows the drug to cross the epineurium, perineurium and endoneurium and inhibit sodium channels more quickly.

Correct answer: (A) The soft tissues in the area of infection are acidic which fractionates the drug largely to its cationic form creating a marked decrease in transport across the cellular membrane. Explanation: This questions centers around the fact that the closer the pKa of a local anesthetic is to the pH of the soft tissues in which it is injected, the more efficient, effective and quicker the block will be. All local anesthetics are weak bases that exist in two forms: lipid soluble uncharged and hydrophilic charged. The lipid soluble form of the anesthetic must be present to cross the cell membrane. Once crossing the axonal membrane, the pH of the interior is less and the drug picks up a charge converting it to the hydrophilic charged form. The charged form is bioactive after it is inside the cell because ionic binding is stronger than the physiochemical binding of the uncharged anesthetic. A lower pKa allows the drug to cross the epineurium, perineurium and endoneurium and inhibit sodium channels more quickly.

What is the appropriate reduction maneuver for a fifth metatarsal tuberosity fracture? A. There is not any useful reduction maneuver. B. Plantarflexion and eversion. C. Dorsiflexion and eversion. D. Dorsiflexion and inversion.

Correct answer: (A) There is not any useful reduction maneuver. Explanation: There is no useful way to reduce a fifth metatarsal tuberosity fracture. For that reason only open reduction and internal fixation is used. Fixation is typically performed with inter fragmentary screw fixation or a tension band technique. (B) Plantarflexion and eversion is not an effective maneuver to reduce a fifth metatarsal tuberosity fracture. (C) Dorsiflexion and eversion is not an effective maneuver to reduce a fifth metatarsal tuberosity fracture. (D) Dorsiflexion and inversion is not an effective maneuver to reduce a fifth metatarsal tuberosity fracture.

Which is not a complication of the procedure pictured below on the hallux? A. Tibial sesamoiditis B. Diminished propulsion of digit C. Stress fracture of the second metatarsal D. Loss of hallux purchase

Correct answer: (A) Tibial sesamoiditis. Explanation: The procedure pictured above is the Keller arthroplasty. Removal of the base of the first proximal phalanx greatly affects the mechanics of the hallux. The propulsion will be diminished as well as a lack of hallux purchase. To to the lessened mechanical effectiveness of the hallux some of the load will transfer to the second metatarsal. This can lead to a stress fracture of the second metatarsal. The Keller arthroplasty has no effect on tibial sesamoid.

You perform a medial sliding calcaneal osteotomy, what should be the most appropriate choice for screw fixation? A. Two 4.0 cancellous screws B. One 4.0 cancellous screws C. 3.5 mm partially threaded screw D. 2.7 mm partially threaded screw

Correct answer: (A) Two 4.0 cancellous screws. Explanation: Because the screw is inserted entirely within cancellous bone, the thread pattern of the 4.0-mm screw seems appropriate. Another option is a 3.5-mm fully threaded cortical screw with a core diameter of 2.4 mm. Theoretically, the thread pattern of the 3.5-mm screw is designed to provide more aggressive purchase in cortical bone, an anatomic impossibility when stabilizing a calcaneal osteotomy with screws inserted from posterior to anterior. In this case, the 3.5-mm screw is the biomechanical screw of choice because of its greater ability to resist bending and traction forces imparted by the Achilles tendon. The 4.0-mm cancellous screws are less tolerant of such forces and are more likely to undergo progressive fatigue and subsequent failure. In considering which implant is most appropriate for a particular construct, it is worthwhile to develop a hierarchy of biomechanical requirements necessary to achieve complete osseous stabilization.

If there is greater than ____° of dorsal malunion present following a distal bunionectomy then revision of the osteotomy must be performed. A. 10° B. 15° C. 20° D. 25°

Correct answer: (B) 15°. Explanation: If there is greater than 15° of dorsal malunion present following a distal bunionectomy then revision of the osteotomy must be performed. The correction for a dorsal malunion is relatively easy and straightforward. Most commonly a medial to lateral crescentic osteotomy is performed in the distal metaphysis. (A) A dorsal malunion with an angulation of 10° is not sufficient to warrant revision of the osteotomy. (C) A dorsal malunion with an angulation of 20° is sufficient to warrant revision of the osteotomy, however the question asks for a minimum angulation which is 15°. (D) A dorsal malunion with an angulation of 25° is sufficient to warrant revision of the osteotomy, however the question asks for a minimum angulation which is 15°.

Genu Varum is normal in pediatric patients until approximately what age? A. Never B. 2 years C. 5 years D. 7 years

Correct answer: (B) 2 years. Explanation: 2 years old is when a pediatric patient should no longer have genu varum. The knees are in a varus alignment typical until the age of 2, near the time of their first steps. During this time, the varus deformity can be quite exaggerated, but the child will grow out of it. The varus is associated with pronounced tibia torsion, which will cause the deformity to look more severe than it really is. Normal genu varum at this age may range from 15-20 degrees. Overall, the prognosis of for genu varum in this age of child is very good as long as there is not underlying pathology, such as in Blount disease, Rickets, and from trauma. (A) Throughout normal child development, the child is typically born with a genu varum until the age of two years. (C) 5 years. Between the ages of 2 and 4, the normal child has a rectus, or straight, position of the knees. This occurs after developing out of the genu varum present until approximately the age of 2, until the development of a slight genu valgum that exists between the ages of 4 and 6. (D) 7 years of age would not be a normal age for genu varum to be present in the normally developing child. At 6 years of age, the normal child may have a genu valgum between the ages of 5-15 degrees. This is normal between the ages of 4-6, which is then followed by the development of a rectus, or straight, knee in the frontal plane.

What ratio of inversion to eversion is considered normal at the subtalar joint? A. 1:1 B. 2:1 C. 3:1 D. 4:1

Correct answer: (B) 2:1. Explanation: Average subtalar joint range of motion is approximately 25-30 degrees with 2/3 of it being inversion and 1/3 being eversion. In other words, the average normal STJ should have 20 degrees of inversion and 10 degrees of eversion available. However, functional ranges of motions used in gait have been reported to be only 6 degrees total motion to 4-6 degrees of both inversion and eversion. The axis of the STJ is 16 degrees from the sagittal plane, 42 degrees from the transverse plane, and 48 degrees from the frontal plane. This indicates that there is relatively equal motion at the transverse and frontal planes, or roughly there is three times the amount of motion available at the transverse and frontal planes than in the sagittal plane. In other words, for every degree of sagittal plane motion, there will be three degrees of transverse and frontal plane motion.

What is the longest acceptable delay from the time of open fracture to the time of administration of appropriate intravenous antibiotics? A. 2 hours B. 3 hours C. 4 hours D. 5 hours

Correct answer: (B) 3 hours. Explanation: The longest time period acceptable from the time of open fracture to the time of administration of intravenous antibiotics is 3 hours. Any delay beyond 3 hours increases the risk of infection. (A) Although 2 hours is well within the 3 hour window of time of open fracture and time to administration of intravenous antibiotics, the question asked for the longest acceptable delay. (C) A delay of 4 hours is beyond the 3 hour threshold pertaining to the time of open fracture to the time of administration of intravenous antibiotics. A delay of 4 hours placed the injury at greater risk of infection. (D) A delay of 5 hours is beyond the 3 hour threshold pertaining to the time of open fracture to the time of administration of intravenous antibiotics. A delay of 5 hours placed the injury at greater risk of infection.

Dorsiflexion of the first ray is a particularly problematic complication of the Lapidus procedure. The most common reason this malposition occurs is due to inadequate first metatarsocuneiform joint preparation. Typically in this scenario, insufficient joint resection is performed at the inferior aspect of the joint due to its depth. What is the approximate depth of the first metatarsal cuneiform joint in an average adult? A. 2.5 cm B. 3.0 cm C. 3.5 cm D. 4.0 cm

Correct answer: (B) 3.0 cm. Explanation: The first metatarsal cuneiform joint is approximately 3 cm deep. For this reason special attention must be pain to ensure the joint surfaces are sufficiently prepared for arthrodesis in their entirety. To assist this endeavor the use of a lamina spreader or joint distractor is recommended. This facilitates access to the inferior-most aspects of the articulation. Furthermore, to better engage the windlass mechanism and ability for the first ray to bear weight, the metatarsal should be plantarflexed. This is most commonly performed by either plantar translation of the metatarsal relative to the cuneiform or removal of a plantar based wedge at the fusion site.

Until what age should conservative methods be used prior to surgical intervention in the treatment of metatarsus adductus? A. 1 year B. 4 years C. 7 years D. 12 years

Correct answer: (B) 4 years. Explanation: Patients aged 4-6 years who have failed conservative treatment of metatarsus adductus may benefit from soft tissue releases such as recession of the abductor hallucis muscle, medial release, tarsometatarsal and intermetatarsus release. Children any older may require multiple metatarsal osteotomies and other osseous procedures. Conservative treatment options such as corrective shoes, serial casting, passive stretching exercises have higher satisfactory rates when begun in children less than one year old with declining success rates as the child ages.

Approximately, to what percentage of lower limb amputations does diabetes contribute? A. 25% B. 50% C. 75% D. 90%

Correct answer: (B) 50%. Explanation: Diabetes contributes to nearly half of all lower limb amputations and is the strongest single risk factor for limb loss. Furthermore, the relative risk for lower limb amputation is nearly 40X greater in the diabetic population. Roughly 15-20% of diabetics will develop a foot ulcer within their lifetime. What's more, foot related complications remains the number one reason diabetic patients are hospitalized.

Callus distortion is an effective and popular technique to treat brachymetatarsia. When the osteotomy is performed in the proximal metatarsal metaphysis, what latency period needs to be ob-served before lengthening can be initiated? A. 4 days B. 7 days C. 10 days D. 14 days

Correct answer: (B) 7 days. Explanation: The latency period for an osteotomy in the proximal metaphysis of a metatarsal is one week or 7 days. During this period the initial steps of bone healing occur and when subse- quent distortion is applied callus distraction in initiated. If the osteotomy is made more in the di- aphysis, then the latency period will increase as this is a less vascular region of the bone. (A) A latency period of 4 days is too short of a period for initial bone healing to establish itself enough to support callus distraction. (C) In the metaphysis, a latency period of 10 days is likely too long and the bone will heal too much to permit callus distortion. (D) In the metaphysis, if a latency of 14 days is observed the osteotomy will solidify too much and it wouldn't be possible to distract.

When considering the management of open fractures what particular modification of the antibiotic therapy needs to make for an open forefoot fracture from a farm auger? A. Addition of antibiotics to the irrigation fluid B. Additional of 20 million units of penicillin C. Addition of fluoroquinolones D. 0.5 ml tetanus toxoid and 250-500 U of tetanus immune globulin

Correct answer: (B) Additional of 20 million units of penicillin. Explanation: Whenever soil inoculation or farm injuries occur that result in an open fracture, an additional 10 - 20 million units of penicillin is necessary to cover Clostridium species. (A) The addition of antibiotics to the irrigation fluid is not particularly indicated in the cases of open fracture, but rather a general surgeon's preference. (C) The addition of fluoroquinolones does not add any benefit int he cases of open fractures due to farm injuries. (D) The presence of soil inosculation and/or farm injuries is independent of tetanus prophylaxis.

Which of the following instruments is generally narrow and allows the surgeon to securely hold structures with the minimum amount of tension? A. Allis Clamp B. Adson-Brown thumb forceps C. Hemostatic Forceps D. Senn Retractor

Correct answer: (B) Adson-Brown thumb forceps. Explanation: Adson-Brown thumb forceps are generally narrow and allow the surgeon to securely hold structures with the minimum amount of tension. The Adson-Brown thumb forceps are also known as rat-tooth pick-ups or one-two pick-ups due to the arrangement of the teeth at the end of the forceps. (A) Allis clamp is incorrect as this is a type of traction forceps. Traction forceps are hinged instruments made up of a set of jaws for grasping tissues, finger rings, and a locking mechanism. The grasping aspect of the jaws is designed to provide traction without unnecessary pressure that may crush the object being grasped. The Allis clamp in particular has serrated edges on opposite sides of the locking jaws, which make it effective for retracting and grabbing fascia and the subcutaneous layer. They are frequently used for clamping towels and drapes in the operative field, as well. (C) Hemostatic forceps is incorrect. Hemostatic forceps are the primary instrument used for acquiring hemostasis during anatomic dissection. They are hinged instruments made up of a set of jaws, finger rings, and a locking mechanism. (D) Senn retractor is incorrect. Retractors are designed to be used in way to increase the visual field of the operating site by holding tissues aside as well as protecting them from incidental injury.

You are requested to place the following In order from the most radiolucent the least radiolucent: air, bone, surgical pin, fat, and connective tissue, Which one is the correct order? A. Fat, air, connective tissue, surgical pin bone B. Air, fat, connective tissue, bone, surgical pin C. Bone, surgical, connective tissue, air, fat D. Surgical pin, bone, connective tissue, fat, air E. Connective tissue, bone, surgical pin, air, fat

Correct answer: (B) Air, fat, connective tissue, bone, surgical pin. Explanation: X-rays interact with matter in different ways, they may be scattered in different directions, or they can be absorbed by an atom. The energy of the photon and the atomic number influence whether the x-ray is absorbed. The difference is known as differential absorption. Objects of greater anatomic absorb x-rays more readily than those with lower anatomic numbers. The thickness of the material also plays a role into absorption of x-rays. The thicker the substance is the more x-rays it absorbs, this is known as attenuation. Radiolucent applies to those substances that x-rays penetrate; they appear dark or black on the exposed film. Bone is radiopaque relative to muscle, but radiolucent when compared to a heavy metal such as lead. In the same manner a thinner foot is more radiolucent than the thicker foot, when the views are the same and the radiographic techniques are the same for both feet.

You are presented with the following xray (see images) of a male patient. He is complaining of osseous pain with a deformity of the right foot that has been present since birth, and the patient is now 33 year old. He reports having undergone surgery to remove an accessory bone on the right foot 10 years ago. The pain has not improved, he is unable to walk barefoot, and has difficulty fitting into shoes. After a complete history and physical, you ordered pedal X-rays and a CT scan. What is the most appropriate diagnosis? A. Phocomelia B. Amelia of the medial cuneiform C. Sympodia D. Hemimelia of the medial cuneiform E. Normal radiograph

Correct answer: (B) Amelia of the medial cuneiform. Explanation: Radiographic evaluation demonstrated continued concavity on the medial aspect of the foot with the hypoplastic first metatarsal. Computed tomography evaluation was obtained to verify the presence or absence of the medial cuneiform. Computed tomography revealed a normal lateral cuneiform; however, it showed the first and second metatarsal bases articulating with the intermediate cuneiform, with the absence of the medial cuneiform. On the basis of the results of the computed tomography scan, it is believed that the initial osseous mass was not an accessory ossicle but a hypoplastic medial cuneiform.

Which of the following muscle groups are active in both the swing phase and stance phase of the gait cycle? A. Lateral leg compartment muscles B. Anterior leg compartment muscles C. Posterior deep compartment muscles D. Posterior superficial compartment muscles

Correct answer: (B) Anterior leg compartment muscles. Explanation: There are not many muscles in the lower extremity that are biphasic meaning that they are active in both the swing and the stance phase of the gait cycle. The anterior compartment muscles, extensor muscle group, are active in both stance and swing. During the stance phase, they are active early in the gait cycle to eccentrically contract to slowly lower the foot down to the ground. Without these functioning, the foot would tend to slap the ground when walking. The anterior leg muscles also work during the swing phase to concentrically contract to allow the foot to clear the ground and prevent the toes dragging on the ground. The remaining muscle groups in the question choices, are all primarily stance phase muscles.

Which of the following statements regarding digital amputations is correct? A. Should never be performed in patients with diabetes B. Are likely to result in contracture of the adjacent digit C. Should always be accompanied by a metatarsal head resection D. May result in regeneration of the digit in polydactyly cases

Correct answer: (B) Are likely to result in contracture of the adjacent digit. Explanation: The abnormal mechanics after digital amputation causes a tendon imbalance in the adjacent digits leading to contracture. This is particularly troubling in diabetics where the buckling of the adjacent digit will cause a retrograde pressure and a new ulceration might develop under the adjacent metatarsal head. Digital amputations are very common among diabetic patients. The idea that a metatarsal head resection should always accompany the digital amputation is incorrect as it would cause even a greater biomechanical imbalance. And there is no validity of regeneration of digits even in the setting of polydactyly.

Reaming produces debris which can be used as the following: A. It is a dead bone. B. Autografting. C. They are disregarded. D. To prevent fat embolization.

Correct answer: (B) Autografting. Explanation: Reaming is an excellent technique for producing intramedullary autografting. The debris which is created usually is collected at the fracture gap which facilitates the local fracture healing response. Besides the different experimental trials there is also plenty of evidence about the beneficial effect of the reaming being considered as a bone stimulating procedure which is the reason it is used as a means of treating nonunions of long bones. When the bone is reamed, there is a byproduct of increased concentration of growth factors. The reaming debris is also a source of stem cells which are able to grow and proliferate in vitro. The reaming debris contains different molecules which are known to enhance the normal fracture healing physiological process. Reaming therefore increases the amount of growth factors which are released into the intramedullary cavity.

This bracing shoe for the treatment of clubfoot is a non-weightbearing prescription brace. It can be adjusted with an Allen wrench. A. Dennis Brown Bar brace B. Bebax shoe C. Mitchell/Ponseti brace D. Fillauer bar

Correct answer: (B) Bebax shoe. Explanation: It is a non-weightbearing prescription brace. It applies continuous corrective force, and the angles are set by a prescribing physician. It has a plantar bar that can be adjusted up to three planes. The brace can be adjusted during treatment as needed using an Allen wrench, provided with shoes. This brace can be used for moderate and flexible to semi-flexible deformity or as a maintenance therapy, with little tibial deformity. (See picture below) (A) The Denis Brown bar is a prescription bar for clubfoot, the shoes attach to the bar with screws, the screws or shoe clips should be set at 70 degrees abducted to follow the treatment after serial casting for severe clubfoot. It should be worn for 18-23 hours after casting. (C) Developed by Dr. Ponseti and John Mitchell, the shoes attach to the bar with screws that attach to plate into shoe. Prescribed after serial casting cases, it should be worn for 23 hours, daily for 3 months, then while sleeping until 3.5 to 4 years old. The length of the bar is similar to the Denis Brown Bar. The bar should be between the heels. The shoe is set at 60 degrees abduction. The shoe is also in 15 degrees dorsiflexion when attached to the bar. (D) The Fillauer bar is also known as the Denis Brown Bar.

The Ponseti technique is a highly effective treatment for idiopathic clubfoot. It is based on several core tenets and manipulations. One basic, yet essential, tenets the Ponseti technique is that the clinician should never touch which bone during manipulation? A. Talus B. Calcaneus C. Navicular D. Cuboid

Correct answer: (B) Calcaneus. Explanation: A fundamental tenet in the Ponseti technique is that the calcaneus should never be touched during manipulations of the foot. Touching or pressure on the calcaneus during manipulation is known as Kite's error, as Kite advocated counterpressure at the calcaneocuboid joint level while manipulating the forefoot. This prevents abduction of the calcaneus and its rolling out of the calcaneus from under the talus to reduce the heel varus and subsequently the equinus. (A) The talus is not the bone that Ponseti insisted must be avoided. (C) The navicular is not the bone that Ponseti insisted must be avoided. (D) The cuboid is not the bone that Ponseti insisted must be avoided.

A 29 year old non insulin dependent diabetic patient is scheduled for Lapidus bunionectomy right foot. On the day of surgery her Hemoglobin A1c, which was drawn the day prior, comes back as 11.1mg/dl. Patient is already in same day surgery and consent is signed. What is the best option to give the patient the best possible outcome? Go forward with procedure as consent already signed. Cancel the procedure due to uncontrolled Diabetes. Give patient 30 units of insulin and complete the procedure. Have patient avoid sugar for two days and then come back to surgery center for procedure.

Correct answer: (B) Cancel the procedure due to uncontrolled Diabetes. Explanation: Poor glycemic control can slow down healing immensely. A patient with a hemoglobin A1c of 11.1 is at high risk for healing complications. In this case, being that the case is elective, surgery should be cancelled pending better glycemic control. Giving patient 30 units of insulin just before the procedure will temporarily decrease blood sugar but not lower HbA1c over long term. Normally red blood cells live for 100-120 days and thus a new HbA1c should be drawn after 3 months of good glycemic control of the patient.

A 60 y/o RA patient is scheduled for a TAR. Pre-operatively, she relates to you a history of neck pain and stiffness, headaches, arm weakness, hyperreflexia and diarrhea. The duration is of approximately six months. She was diagnosed with RA 10 years ago. As part of your pre- operative testing, which test should be ordered first to follow up on these symptoms? A. Head CT B. Cervical neck x-rays C. Cerebral Arteriogram D. Thoracic back x-rays

Correct answer: (B) Cervical neck x-rays. Explanation: 80% of RA patients have cervical spine involvement, especially with increasing duration of the disease. An early symptom is neck pain; as involvement continues, multiple neurological symptoms as described above are noted. Bladder and bowel dysfunction may be seen. Increasing age, seropositivity, and peripheral joint involvement should lead one to suspect cervical spine involvement. The concern, of course, is the intubation process during anesthesia. An A/P, lateral, oblique, and full flexion lateral x-ray should be ordered. If equivocal, order a neck CT. (A) A head CT is not out of the question, but considering cost containment issues, an x-ray should be ordered first. Also, your history of RA, with these signs and symptoms, should lead you to cervical neck pathology first. (C) Again, this is not a first choice test. Board answers should be logical and stepwise. Plain films first, followed by advanced imaging as necessary. (D) The history and signs and symptoms should lead you to the cervical level. These views do not show you that level.

Your 49 year old patient has rheumatoid arthritis and is on multiple drug therapy including oral steroids and methotrexate. You are planning a two-piece metallic implant arthroplasty of the first metatarsophalangeal joint and antibiotic prophylaxis pre-operatively. The best therapy for this patient is: A. Ciprofloxacin 1500 mg IV pre-operatively B. Clindamycin 600 mg IV pre-operatively C. Cefazolin 1 gm IV pre-operatively D. Vancomycin IV pre-operatively

Correct answer: (B) Clindamycin 600 mg IV pre-operatively. Explanation: Surgical prophylaxis is indicated in this patient due the immunocompromised state due to the steroid use, methotrexate use and of particular note here the implant. Clindamycin is the drug of choice here. Although Cefazolin and Vancomycin could be used, clindamycin is best indicated due to the metallic implant. Clindamycin has been shown to be effective in penetrating the bacterial glycocalyx that forms around implants. Cefazolin has excellent anti-staphylococcal activity and also some gram negative coverage as well. Vancomycin is indicated in the presence of penicillin or cephalosporin allergy and in scenarios where a high incidence of methicillin- resistant staphylococcus aureus is present. Ciprofloxacin should not be used as its anti- staphylococcal activity is quite poor.

Which of the following qualities of local anesthetics does not affect the duration of action? A. Lipid Solubility B. Concentration of anesthetic used C. Protein Binding D. Addition of Epinephrine

Correct answer: (B) Concentration of anesthetic used. Explanation: The concentration of local anesthetic or amount of local anesthetic per milliliter does not affect the duration of action in any regard. Lipid solubility indeed does affect duration greatly. The more lipid soluble the anesthetic the more it can traverse the lipophilic membrane and bind to the sodium channels. Similarly the protein binding capacity is important regarding the duration of action as the anesthetic with a higher protein binding capacity has a higher affinity to bind with the cell membrane. The addition of epinephrine also increase the duration of action by producing vasoconstriction, therefore limiting systemic absorption and maintaining the drug in the vicinity of the nerve fibers for a longer period of time.

When evaluating medial and lateral subtalar dislocations radiographically which statement is true for both medial and lateral subtalar dislocations? A. Medially displaced navicular B. Congruous calcaneocuboid joint C. Inversion-type ankle fracture patterns D. Eversion-type ankle fracture patterns

Correct answer: (B) Congruous calcaneocuboid joint. Explanation: In both medial and lateral subtalar joint dislocations have congruous calcaneocuboid joints. (A) Medially displace navicular injuries are seen in the medial subtalar joint dislocation. (C) Inversion-type ankle fracture patterns are seen in medial subtalar joint dislocations. (D) Eversion-type ankle fracture patterns are seen in lateral subtalar joint dislocations.

This angle is viewed from a lateral projection and evaluates the angular relationship between the calcaneal facets and the lateral process of the talus. The angle may be disrupted in the case of an intra-articular calcaneal fracture. A. Boehler's angle B. Critical angle of Gissane C. Phillip and Fowler angle D. Talar declination angle

Correct answer: (B) Critical angle of Gissane. Explanation: The critical angle of Gissane is often assessed in calcaneal fractures. On a lateral film, it correlates the calcaneal facets with the lateral process of the talus. The normal angle averages between 130 to 145 degrees. Angular distortion generally correlates with intra-articular calcaneal fracture. (A) Boehler's angle measures the relationship between the anterior dorsal aspect of the calcaneus and the posterior dorsal aspect of the calcaneus. This angle is used in determining whether there is a depressed posterior facet in a calcaneal fracture. However, this does not measure the lateral process of the talus. (C) The Phillip and Fowler angle is the line of the posterosuperior surface of the calcaneus and the plantar calcaneal line. This is often used ine valuation of a Haglund's deformity. (D) The talar declination angle is between a line through the central head and neck of the talus with a line of the supporting surface. The normal angle averages 21 degrees, but may increase with pronation and decrease with supination.

A 51 year old male patient undergoes total hip arthroplasty under general anesthesia. The patient is an ASA Class 3 and has a history of heart disease and previous MI. On which of the following days is a post-op MI most likely and how many months after an MI should elective surgery be postponed? A. Day 2, 5 months B. Day 3, 6 months C. Day 3, 12 months D. Day 2, 8 months

Correct answer: (B) Day 3, 6 months. Explanation: The most common day for post operative myocardial infarction is days 3 post operatively. It is also thought that the amount of time after MI to postpone elective surgery is 6 months.

A 55 year old female patient who has worn high heels her entire life presents to the office with pain in her 3rd interspace right foot. Patient states that she was seeing another podiatrist and had 2 steroid injections and a series of sclerosing alcohol injections with minimal relief in pain. She also had MRI showing a neuroma 3rd interspace. Patient opts for surgical intervention. A dorsal approach in the interspace is taken for surgical exposure. Which of the following structures should be the first seen in dissection? A. Common proper nerve B. Deep Transverse Inter-metatarsal Ligament C. Proper digital nerves D. Metatarsal Bone

Correct answer: (B) Deep Transverse Inter-metatarsal Ligament. Explanation: There are two approaches to neuroma surgery, a dorsal approach and a plantar approach. The dorsal approach is most commonly utilized in first time neuroma excision surgery. In doing this dissection, first you will see subcutaneous tissue in between the metatarsals. The next structure you will look for is the deep transverse intermetatarsal ligament. Once this is severed, the next structure you would look for is the common proper nerve and the proper digital nerve. The proper digital nerves would be transected distally and the common proper nerve would be transected as far proximally as possible (proximal to met heads).

In the Austin osteotomy pictured below, what direction should the saw blade be angulated to produce shortening and plantarflexion? A. proximal-medial dorsal and distal-lateral plantar B. distal-medial dorsal and proximal-lateral plantar C. dorsal proximal-lateral and plantar-medial distal D. medial plantar and dorsal lateral

Correct answer: (B) Distal-medial dorsal and proximal-lateral plantar. Explanation: By angulating the surgeon's saw blade during the creation of the Austin or Chevron osteotomy, the final position of the capital fragment can be altered significantly. For a goal of shifting the capital fragment plantarly and producing a shortening of the metatarsal, the saw blade must be angulated from distal to proximal and from dorsal to plantar to produce a plantarflexion and shortening effect as the capital fragment is shifted from medial to lateral across the width of the metatarsal. The first option would produce lengthening of the metatarsal with plantar flexion. The third and fourth options are purely distracters as they do not make sense regarding the question presented.

An adult patient in your office undergoes a tibial nerve injection. The patient becomes pale, diaphoretic, develops a hoarse voice with wheezing and angioedema. It would be most appropriate to administer: A. Epinephrine 3mg of 1:1000 subq B. Epinephrine 0.3mg of 1:1000 subq C. Ondansetron 16mg tab D. Ondansetron 32mg IV

Correct answer: (B) Epinephrine 0.3mg of 1:1000 subq. Explanation: Epinephrine is the first line treatment for anaphylaxis. Epinephrine 1:1000 at a dose of 0.2-0.5mg in adults and 0.01mg/kg in children up to 0.3mg should be administered. (A) Epinephrine 1:1000 at a dose of 3mg would be too large of a dose. The recommended amount of 0.2-0.5mg for an adult. (C) (D) It is important to recognize that these are early signs of an anaphylactic reaction, most likely to the local anesthetic in this scenario. It would not be appropriate to administer an anti- emetic at this time.

The apex of deformity in an anterior cavus foot deformity would occur at? A. Talo-navicular joint B. First metatarsal - medial cuneiform joint C. Navicular - cuneiform joint D. First metatarsal-phalangeal joint

Correct answer: (B) First metatarsal - medial cuneiform joint. Explanation: The apex of deformity in an anterior cavus foot deformity would occurs at the first metatarsal - medial cuneiform joint. By definition the calcaneal inclination angle is less than 30 degrees. Meary's Angle is the angle formed between the lines drawn through the longitudinal axis if the talus and the first metatarsal and in the anterior cavus it intersects in the first metatarsal and medial cuneiform joint.

These long plantarflexors divide into four separate tendon slips, each tendon slip into the distal phalanx of the lesser digits; they belong to this muscle layer in the foot: A. Flexor digitorum longus, 1st layer B. Flexor digitorum longus, 2nd layer C. Flexor digitorum longus, 3rd layer D. Flexor digitorum longus, 4th layer

Correct answer: (B) Flexor digitorum longus, 2nd layer. Explanation: The FDL divides into 4 separate tendon slips in the second layer of muscles in the plantar aspect of the foot.

What is the most appropriate surgical option for the 73 year old non-insulin diabetic female with the distal toe ulceration pictured below? The deformity is not rigid. The patient has an apropulsive, shuffling gait with walker assistance. A. Proximal interphalangeal joint arthrodesis B. Flexor tenotomy C. Proximal interphalangeal joint arthroplasty D. Distal interphalangeal arthroplasty

Correct answer: (B) Flexor tenotomy. Explanation: The primary problem is the excessive load on the tip of the toe leading to ulceration. This is secondary to the reducible toe contracture. This is an ideal indication for a flexor tenotomy. This will allow manual reduction of the deformity effectively offloading the tip of the toe. Additionally, it will require significant healing like the procedures on the bone. It will be acceptable from a functional standpoint and allow the patient to ambulate minimally and in a propulsive manner. It is not that the proximal interphalangeal joint arthroplasty or arthrodesis would not work, but given the specifics of the patient and the deformity, the flexor tenotomy is ideally indicated. The distal interphalangeal joint arthroplasty is simply a correction at the incorrect joint level.

Which of the following drugs have an adverse effect on bone healing? A. Hydrochlorothiazide B. Hydrocodone C. Carvedilol D. Carisprodol E. Promethazine

Correct answer: (B) Hydrocodone. Explanation: Hydrocodone has been reported to cause osteoporosis and fracture. Hydrochlorothiazide has a protective effect on bone. Carvedilol, carisprodol, and promethazine have no effect on bone healing.

Which statement regarding the radionuclides technetium and gallium is correct? A. Gallium is a bone-imaging radionuclide. B. If both technetium and gallium are used in a study, technetium should be given first. C. Technetium is an inflammatory-imaging radionuclide. D. If both technetium and gallium are used in a study, gallium should be given first.

Correct answer: (B) If both technetium and gallium are used in a study, technetium should be given first. Explanation: Technetium is a bone-imaging radionuclide while gallium is an inflammatory- imaging radionuclide. Gallium binds to white blood cells, plasma proteins, transferrin, ferritin, lactoferrin and siderophores and travels to areas of inflammation. Technetium binds to hydroxyapatite crystals and travels to area of high bone turnover. These two radionuclides can be used together in a test which may very well give more information than either test can do in isolation. If both gallium and technetium are used, technetium should given first, followed by gallium in 24 to 48 hours.

Which of the following is true of uncompensated rearfoot varus? Eversion ankle sprains are common Increased lateral wearing of the shoe People don't get calluses in tripod positioning (Sub heel, 1st metatarsal head and 5th metatarsal head) Bunions are very common with this foot type

Correct answer: (B) Increased lateral wearing of the shoe. Explanation: Rearfoot varus is a frontal plane deformity that causes the foot to hit the ground with more heel inversion than would normally be seen. There are multiple causes of this deformity including tibial varum, talar varum, subtalar joint varum, and Bloundt's Disease. With an uncompensated rearfoot varus, the heel strikes the ground inverted. Since it is uncompensated, the foot does not have any motion at the subtalar joint so the calcaneus cannot evert to fix this deformity. In order to get the medial column down to the ground, the 1st ray plantarflexes. This causes a lesion pattern that some term the tripod position (calluses sub heel, sub 1st metatarsal head and sub 5th metatarsal head). People with this deformity are at a high risk for inversion ankle sprains since the foot is inverted more than normal. There is also an increase in lateral shoe wear for the same reason. There is not a large pronatory mechanism with this foot type so it is fairly uncommon to see bunions develop.

The 2007 The American College of Cardiology/American Heart Association have established risk stratification with any given surgery. What risk is associated with foot and ankle/orthopedic surgeries? A. Low-risk surgeries B. Intermediate surgeries C. Highest risk surgeries D. Higher risk surgeries

Correct answer: (B) Intermediate surgeries. Explanation: The ACC/AHA did establish guidelines on perioperative cardiovascular evaluation and care for non- cardiac surgery; they offer a step wise approach for perioperative evaluation. Step 1: Establish the urgency of surgery. Step 2: Assess active cardiac conditions. Step 3: Determine the surgery-specific risk. The foot and ankle surgery/orthopedic surgery is designated as intermediate risk surgeries, with 1%-5% risk of adverse cardiac events. Step 4: Assess the patient's functional capacity. Step 5: Assess patient's clinical risk factors. (A) The ACC/AHA did assign the low-risk surgeries to superficial procedures, cataract surgery, breast surgery, endoscopic procedures. (C) The highest risk surgeries were assigned to vascular procedures involving revascularization and aortic surgeries. (D) There is no such classification with the ACC/AHA guidelines.

Callus formation is a result of indirect osseous repair; this process is most likely to occur with which one of the following fracture stabilization: A. External fixation B. Intramedullary nailing C. Screw fixation via lag technique D. Compression plates E. LCP plate application

Correct answer: (B) Intramedullary nailing. Explanation: Unlike plate fixation, intramedullary nailing does not provide absolutely rigid fixation, so that some motion at the fracture site exists. The process of reaming causes circulatory disturbances in the inner two-thirds of the diaphyseal cortex. However, this does not impede the formation of external callus. The process of indirect osseous repair is a progression from hematoma to cartilage callus to ossification. This process occurs in the absence of internal or external stabilization and has been called spontaneous or indirect healing. The expansiveness of the callus is directly related to the degree of instability at the fracture site. An unstable fracture demonstrates more callus formation, whereas a stable fracture generates little or no callus. The increasing diameter of callus tissue mound a fracture provides greater resistance to bending and rotary forces. Specifically, "the rigidity efficiency of the tissue increases with the fourth power of the distance from the center of rotation or bending, whereas the strength efficiency increases with the third power". With intramedullary nailing/fixation, fracture healing mainly proceeds by the formation of periosteal callus. Over the years intramedullary nailing has become the standard treatment of long-bone diaphyseal and selected metaphyseal fractures providing sufficient stability with limited soft-tissue dissection. Furthermore, this technique preserves the muscle envelope around the fracture and therefore the extra-osseous blood supply to the bone. As a consequence, revascularization of the injured bone is enhanced and periosteal callus formation is promoted.

A 27 year old female presents to the emergency room complaining of pain and swelling to the right ankle. The evening prior she had twisted her ankle when her right high heel shoe gave away. Pictured below are radiographs from her emergency room visit. What is the mechanism of injury here? A. eversion - dorsiflexion B. inversion - plantarflexion C. dorsiflexion - inversion D. plantarflexion - eversion

Correct answer: (B) Inversion - plantarflexion. Explanation: Pictured above is a fracture of the anterior process of the calcaneus. Key triggers in the question include the fact the patient is a female and it occurred while wearing high heels. This is the most common and accepted mechanism of injury for this fracture. Women have a higher incidence of this fracture secondary to wearing high-heeled shoes that plantarflexes the foot on the leg and subsequently unlocks the ankle joint, thereby predisposing them to ankle sprains. The anterior process of the calcaneus usually fractures due to an avulsion or traction injury. Typically the injury is caused by excessive inversion of the ankle and subtalar joints, placing a traction force on the lateral ligaments. The bifurcate ligament attaches the anterior process of the calcaneus to the navicular and cuboid bones. Excessive traction to this ligament can result in a fracture of the anterior process of the calcaneus.

Please refer to the radiograph below. Please note that the second most distal wire is arched. What is the purpose of this wire? A. It is providing neutralization. B. It is providing compression. C. It is providing distraction. D. It is providing translation.

Correct answer: (B) It is providing compression. Explanation: The wire referenced above is providing compression of the midtarsal joint, more specifically the talonavicular joint. This is the simplest method of generating compression. The technique is performed by applying a thin wire through an osseous segment, but instead of attaching the ends of the wire to a ring so that the wire is straight, it is applied more proximal on the ring so that when the wire is attached at the other end, it bows. When the wire is tensioned, the wire will want to straighten and in doing so, compresses the attached segment to the proximal part. This technique is referred to as "Russianing." An alternative to "Russianing" is the application of compression by translating a distal ring toward a more proximal ring, along parallel connecting rods.

A 6 year old patient goes to the operating room for a subtalar joint arthroeresis. The podiatrist is determining what type of local anesthesia to give the patient and also the maximum dose of the anesthesia. Which of the following is the best local anesthesia to give patient and the maximum dose of that type of anesthesia? A. Marcaine 0.5% plain, Max dose: 2.5mg/kg B. Lidocaine 1% plain, Max dose: 4.5mg/kg C. Marcaine 0.5% plain, Max Dose: 7.5mg/kg D. Lidocaine 1% plain, Max dose 7.5mg/kg

Correct answer: (B) Lidocaine 1% plain, Max dose: 4.5mg/kg. Explanation: The first thing that you need to pull out of the stem of the question is the age of the patient being 6. Marcaine is not usually used in children under the age of 12 so you can automatically remove two answer choices as Lidocaine is the best answer. The max dose of 0.5% marcaine plain is 2.5mg/kg. The max dose of Lidocaine 1% plain is 4.5mg/kg.

When debriding soft tissue, blood vessels larger than 2 to 3 mm should be ligated. Which method of ligation would be most advisable to use? A. Silk suture B. Liga-clips C. Polyglycolic-woven suture D. Polyglycolide-woven suture

Correct answer: (B) Liga-clips. Explanation: During debridement of subcutaneous tissue, small blood vessels should be cauterized by bipolar cautery. If the vessels are larger than 2 to 3 mm, then they should be ligated. Liga-clips are the least reactive foreign body material to accomplish this. If a suture is to be used, then a small diameter monofilament suture should be used to minimize the risk of facilitating further infection. For example, silk acts like a foreign body and stimulates a vigorous foreign-body reaction, whereas bacteriostatic, polyglycolic-woven suture and polyglycolide- woven suture have multiple recesses within which bacteria can survive in a semi-protected state. A suitable suture would be a 5-0 polydioxanone which is a monofilament suture.

Which of the following is FALSE regarding pediatric metatarsus adductus? A. Occurs primarily at the Lisfranc joint B. Medial border of foot is convex C. In infants, the metatarsus adductus angle is measured utilizing a bisection of the calcaneus and a bisection of the second metatarsal D. Engel's angle is used to measure metatarsus adductus in older children

Correct answer: (B) Medial border of foot is convex. Explanation: With metatarsus adductus, the metatarsals are adducted at Lisfranc joint thus making and concave medial border of the foot and convex lateral border of the foot (sometimes referred to as a "C" sign). (A) Metatarsus adductus occurs primarily at the Lisfranc articulation. This is different than talipes calcaneovarus which occurs primarily at Chopart's joint. (C) In infants, this is the angle used as many bones are not visible in infant (only have cartilage until ossification occurs). Normal value for this angle is less than 22 degrees. (D) Engel's Angle is the bisection of the 2nd metatarsal with the bisection of the middle (intermediate cuneiform). Normal mean value of Engel's angle is 18 degrees.

What structure is most at risk for iatrogenic injury when using a dorsomedial incision one the first metarasocuneiform joint as part of a Lapidus procedure (arthrodesis of the first metatarsocuneiform joint)? A. deep peroneal nerve B. medial dorsal cutaneous nerve C. dorsalis pedis artery D. arcuate artery

Correct answer: (B) Medial dorsal cutaneous nerve. Explanation: This question focuses on the knowledge of surgical anatomy and how it relates to mitigating complications with the Lapidus procedure. The dorsomedial cutaneous nerve very often is located directly dorsal to the first metatarsocuneiform joint and is in significant danger of injury with the dorsal medial single incision approach. The deep peroneal nerve lies much more laterally at this level. It is at greater danger with overly aggressive interspace dissection. The dorsalis pedis artery lies lateral to the dorsomedial incision and typically is easily identified and retracted laterally. The arcuate artery is distal to to this area and not at risk at all.

Which oral diabetic medication should be held 48 hours prior to surgery, and may be restarted within 48-72 hours post-surgery if normal renal function is documented? A. Chlorpropramide B. Metformin C. Thiazoladinediones D. Incretins

Correct answer: (B) Metformin. Explanation: Metformin generally should be withheld 48 hours prior to surgery and withheld 48- 72 hours post-surgery. Renal function should be normal prior to resuming therapy. Glucose levels should be checked regularly and elevated levels of > 180 mg/dL can be treated with intermittent doses of short-acting insulin. Pioglitazone are thiazoladinediones, which can be held on the morning of surgery as they are given once daily with a duration of action of 24 plus hours. Secretagogues must be held preoperatively. Long acting sulfonylureas are best withheld 48 to 72 hours preoperatively to avoid potential hypoglycemia.

Which of the following factors carries the greatest risk in the perioperative evaluation of risk for deep venous thrombosis? A. Tourniquet time over 90 minutes B. Multiple trauma C. Operating room time over 105 minutes D. Congestive heart failure E. Rearfoot surgery

Correct answer: (B) Multiple trauma. Explanation: Although all of the above are risk factors as presented in the Caprini risk assessment scheme, multiple trauma and acute spinal cord injury are weighted the most heavily.

Which of the following is the most common complication of tourniquet use? A. Lymphedema B. Nerve injury C. Compartment syndrome D. Wound hematoma E. Vascular injury

Correct answer: (B) Nerve injury. Explanation: Nerve injury is the most common complication of tourniquet use. Both mechanical compression and neuronal ischemia are thought to play a role in this injury. Other complications include edema, delay of recovery of muscle strength, compression neurapraxia, wound hematoma, vascular injury, tissue necrosis, compartment syndrome, skin trauma, among others.

A 35 year old patient with a past medical history of Rheumatoid Arthritis is going for foot surgery. The patient is normally on glucocorticoids daily for the RA. Which of the following is false regarding his peri-operative treatment? Cervical Spine x-rays should be ordered and reviewed prior to surgery. Patient should stop his daily glucocorticoid a week before surgery and not start until a week after surgery. Supplemental glucocorticoid should be given to patient in per-operative period. Many patients with RA have poor bone quality and therefore require more fixation and longer periods of non weight bearing after surgery.

Correct answer: (B) Patient should stop his daily glucocorticoid a week before surgery and not start until a week after surgery. Explanation: Most patients who have RA are on either glucocorticoids or are taking methotrexate, penicillamine etc. A patient with RA should always have cervical spine x-rays taken before undergoing anesthesia to look for atlanto-axial subluxation. Patients who are undergoing surgery and are currently on steroids usually need stress dose (supplemental) steroids prior to surgery. Chronic glucocorticoid therapy can suppress the hypothalamic - pituitary- adrenal axis (HPA) during high stress situations (as with surgery); the adrenal glands will not be able to produce enough cortisol. There are many different algorithms to determine how much extra glucocorticoid to give the patient but for most foot surgeries 100 mg methylprednisone is given pre-op. If the patient has severe disease or the procedure is extensive, it could be continued and tapered after surgery. You should never abruptly stop corticosteroids prior to surgery which is why (B) is the correct answer to this question. (D) in this question is a correct statement.

In the discussion of pediatric flatfoot there is an important mantra of Valmassy's Rule of Seven. The rule has two components. The first is that a normally developing child should have a maximally everted calcaneus to the floor at early walking of 7°. The second component of the rule states that by the age of 7 the child should develop a visible longitudinal arch and a calcaneus in what position relative to the ground? A. Inverted B. Perpendicular C. Everted D. Pronated

Correct answer: (B) Perpendicular. Explanation: The second component of Valmassy's Rule of Seven is that by the age of 7 the child should develop a visible longitudinal arch and the calcaneus should be perpendicular to the ground. So, in other words, an everted calcaneus at the age of 7 or excessive eversion greater that 7° in the early walker can be classified as a pediatric hyper mobile flatfoot. (A) An inverted heel position at age 7 is not a correct of component of Valmassy's Rule of 7. (C) An everted calcaneus at the age of 7 or excessive eversion greater that 7° in the early walker can be classified as a pediatric hyper mobile flatfoot. (D) Pronations a triplanar term so it does not apply to the isolated frontal plane of the calcaneus.

Which of the following techniques is appropriate as an isolated treatment for unrepairable painful nerve segment? A. More proximal neurectomy B. Placement of the cut end of the nerve into muscle or bone C. External neurolysis D. Internal neurolysis

Correct answer: (B) Placement of the cut end of the nerve into muscle or bone. Explanation: Painful, unrepairable nerve segments can occur in multiples situations. Some include trauma, amputation and neurectomy. Nonsurgical modalities that have the possibility of success are cryotherapy and chemical sclerosis either with alcohol or phenol. Surgically there have been several treatments described. They include capping of the nerve, the use of silicone shielding, covering of the nerve with a muscle or vein and lastly placement of the cut end of the nerve into a muscle or bone. A more proximal neurectomy seldom leads to success as there is still an exposed nerve ending just more proximally located. Internal and external neurolysis have shown to have little success as solitary procedures in the case of painful, unrepairable nerve segments.

Brushing up on your anatomy before your first surgical procedure, you are placing all the intrinsic foot muscles in their correct layers, origin and insertion, you can't quite well remember if the lumbricles are plantar or dorsal to the deep transverse intermetatarsal ligaments, please choose the correct answer? A. Deep to the Deep Transverse intermetatarsal ligament and 3rd muscle layer B. Plantar to the Deep transverse intermetatarsal ligament and 2nd muscle layer C. Plantar to the deep transverse intermetatarsal ligament and 4th layer D. Dorsal to the deep transverse intermetatarsal ligment and 2nd muscle layer E. Dorsal to the deep transverse intermetatarsal ligament and 3rd muscle layer

Correct answer: (B) Plantar to the Deep transverse intermetatarsal ligament and 2nd muscle layer. Explanation: The lumbricales are 4 small muscles that have an unusual tendon origin. Most muscles originate from bone. Each of the lumbricales originate from the medial side of the corresponding flexor digitorum longus slip to the lesser toe. They course plantar to the deep transverse intermetatarsal ligament and insert medially into the base of the proximal phalanx and the oblique portion of the extensor hood apparatus. The lumbricales flex the MTPJ, and extend the interphalangeal joints. They have a stronger effect over the PIPJ, and DIPJ due to their distal attachment compared to the interossei, which are weak extensors.

Congential Vertical Talus includes all of the deformities listed EXCEPT: A. Convexity of the plantar aspect of the foot B. Plantarflexion of the foot C. Valgus of the heel D. Talonavicular dislocation

Correct answer: (B) Plantarflexion of the foot. Explanation: Congenital convex pes valgus or congenital vertical talus deformity is uniquely described as a deformity with convexity of the plantar aspect of the foot, valgus of the heel, abduction of the forefoot, talonavicular dislocation and dorsiflexion of the foot to the point that the foot is touching or near touching the anterior aspect of the tibia. Therefore plantarflexion of the foot is an incorrect description of the deformity.

Which of the following is true regarding perioperative fluid management? Maintenance therapy is used when a patient is on a regular diet. Replacement therapy is used to replenish water and electrolyte deficits. Normal saline and Lactated Ringer's are both equivalent to human plasma composition. Colloid solutions include NS and LR.

Correct answer: (B) Replacement therapy is used to replenish water and electrolyte deficits. Explanation: Postoperative fluid management consists of two types of therapy: maintenance and replacement. Maintenance therapy is initiated when the patient is not expected to eat or drink for a certain amount of time perioperatively. Replacement therapy is aimed at replenishing any additional water and electrolyte deficits. Lactated Ringers more closely mimics human plasma with a pH of 6.5 than does normal saline. Crystalloid solutions are most frequently used and include NSS and LR.

Which one of the following is a principle of biologic fixation? When the goal is to obtain pseudoarthrosis across the fracture site Repositioning and realigning by manipulation at a distance to the fracture site, preserving soft tissue attachments Illizarov external fixator Volkman's principle Vassal principle

Correct answer: (B) Repositioning and realigning by manipulation at a distance to the fracture site, preserving soft tissue attachments. Explanation: In order to maintain length and alignment, this method of fixation can be used. It occasionally may require a plate fixation to bridge a badly comminuted segment of bone. Biologic fixation is recommended when in presence of comminuted fracture fragments, a lag screw would result in fragment devitalization, or when there is a gap in the fracture and a plate will be used to maintain the relative position of the fragments. This fixation is not stable and requires adjunct methods to maintain all the soft tissue attachments and blood supply of the intervening comminuted fragments, since union will depend on the formation of a bridging callus rather than primary bone union. In these situations, techniques of indirect reduction are particularly useful. The following principles are respected: 1- repositioning and realigning the fracture, preserving soft tissue attachments 2- leaving the comminution fragments out of the mechanical construct, if fragment is incorporated into the construct it should have a vascular supply 3- use biocompatible materials required 4- minimum contact between bone and implant 5- limiting operative exposure when possible

You performed an ankle fusion on a 75 year old patient. The patient is on chronic warfarin therapy with a CHADS-2 score of 5. Prior to the procedure, the patient was placed on a bridging therapy with lower molecular weight heparin. Following a successful surgery, the lower molecular weight heparin was restarted without incident. When can the patient start back on warfarin? A. Restart the warfarin right away at the patient's usual dose and monitor the INR. B. Restart the warfarin at patient's usual dose on the evening of the surgical procedure. C. Discontinue the LMWH after the surgical procedure. D. Restart warfarin 5 days post procedure.

Correct answer: (B) Restart the warfarin at patient's usual dose on the evening of the surgical procedure. Explanation: A CHADS-2 score of 5 indicating a high annual risk of stroke (1 point for each for congestive heart failure, hypertension, age of 75 or greater, and Diabetes mellitus, 2 points for a history of stroke or transient ischemic attack (TIA). This case justifies a bridging therapy. So preoperatively, ensure that the patient does not have any contraindications to LMWH such as allergy to LMWH, history of heparin induced thrombocytopenia, extremes of weight, creatinine clearance of < 15 m/min, history of non-compliance, inability to administer injections. The bridging instructions should be as follows: stop warfarin 5 days before surgery if INR is 2-3, stop warfarin 6 days before surgery if INR is 3-4.5, start LMWH 36 hours after the last dose of warfarin, check INR the morning of surgery to ensure an INR < 1.5 and in some cases <1.2. Then restart the LMWH approximately 24 hours post procedure or consider thromboprophylaxis dosing of LMWH on post op day # 1. If patient is at high risk for bleeding. Restart warfarin at patient's usual dose on the surgical day. Check INR daily until patient is discharged and periodically thereafter until INR is therapeutic. Check CBC on post-op days 3 and 7 to monitor platelets, discontinue LMWH when INR is therapeutic for two consecutive days.

What structure is at risk at the plantar aspect of the first metatarsal cuneiform joint during joint preparation for the Lapidus procedure? A. Medial plantar artery B. Medial plantar nerve C. Flexor hallucis longus D. Posterior tibial tendon

Correct answer: (C) Flexor hallucis longus. Explanation: When denuding the cartilage of the first metatarsal cuneiform joint the surgeon must be aware of the flexor hallucis longus at the far plantar aspect of the joint. Over zealous joint debriding plantarly can lead to iatrogenic (A) The medial plantar artery is a minimal risk during debridement of the first metatarsal cuneiform joint. (B) The medial plantar nerve is a minimal risk during debridement of the first metatarsal cuneiform joint. (D) The posterior tibial tendon is a minimal risk during debridement of the first metatarsal cuneiform joint.

Which type of fifth toe deformity is the Lapidus procedure (first tarsometatarsal arthrodesis) indicated? A. Cock-up fifth toe B. Severe overlapping fifth toe C. Underlapping fifth toe D. Fifth hammertoe

Correct answer: (B) Severe overlapping fifth toe. Explanation: The Lapidus procedure is indicated for severely overlapping fifth toes. It consists of a curvilinear incision over the dorsum of the fifth metatarsophalageal joint and proximal phalanx with a secondary more proximal incision over the extensor tendon. The extensor tendon is transacted proximally and bought through the distal incision. The dorsal, medial and lateral meta-tarsophalangeal joint capsule are released. The extensor tendon is wrapped around the phalanx from dorsal medial to plantar lateral and attached to periosteum of the head of the fifth metatar-sal, soft tissue or muscle proximal to the fifth metatarsal head. (A) The cock-up fifth toe is most commonly corrected via the Ruiz-Mora procedure. (C) An underlapping fifth toe deformity is best treated with the Thompson procedure. (D) A fifth toe hammertoe deformity is most commonly treated by a Post procedure or a proxi- mal interphalangeal joint arthroplasty.

Following an endoscopic gastrocnemius recession, the patient complains of dysesthesias. Which structure was most likely damaged? A. Lesser saphenous vein B. Sural nerve C. Saphenous nerve D. Intermediate dorsal cutaneous nerve

Correct answer: (B) Sural nerve. Explanation: The sural nerve is just lateral to the Achilles tendon and at risk of injury if the procedure is not performed correctly. Once the cannula is in proper position, there should be visualization of the gastrocnemius aponeurosis only. This ensures that the sural nerve is not involved when the hook blade is used to incise the aponeurosis. (A) The lesser saphenous vein runs adjacent to the sural nerve and is at risk of injury if the procedure is not correctly performed. However, injury to the vein itself would not likely cause dysesthesias unless the sural nerve was also involved. (C) The saphenous nerve is on the medial aspect of the ankle, further anterior than the gastrocnemius aponeurosis. (D) The intermediate dorsal cutaneous nerve is on the anterolateral aspect of the ankle and branches from the superficial peroneal nerve more distal than the level at which the EGR is performed.

The earliest symptom of malignant hyperthermia is? A. Hyperthermia B. Tachycardia C. Hypoxia D. Tetany E. Pain

Correct answer: (B) Tachycardia. Explanation: The earliest signs of malignant hyperthermia are tachycardia, a rise in end-tidal carbon dioxide concentration despite changes in ventilation, and muscle rigidity. Hyperthermia is often dramatic but is usually a late sign.

You are a consultant to the pediatric clinic. A mother presents for initial evaluation for her 2 week old boy. Her chief complaint is that her newborn sons feet are in a "down and in" position. She reports that both feet appear very small, and are contracted in the heel. You suspect the following: A. Metatarsus adductus B. Talipes equinovarus C. Calcaneovalgus D. Flexible pes plano valgus

Correct answer: (B) Talipes equinovarus. Explanation: The question wants to emphasize the visual characteristics on this congenital deformity described by the patient's parent. The most important descriptor here are the "Down and in", and the fact that the patient is just 2 weeks old, this deformity is bilateral. This will lead the clinician to suspect the presence of an equinovarus deformity which presents with an inversion and adduction of the forefoot, inversion of the heel and hind foot, an equinus throughout the ankle and the subtalar joint. (A) This deformity may be misdiagnosed as a met adductus which presents as a C-shaped foot with a concave medial border with a splay noted between the hallux and the second toe. This is not the correct answer for this case. (C) This is a congenital flexible flatfoot, with the foot near or touching the anterior ankle. (D) The descriptors are floppy foot, fat foot, out-toe gait or too many toes sign will be part of the description. This is not the case here.

What is the most appropriate fixation technique for a small, mildly displaced intraarticular fracture of the styloid process of the fifth metatarsal? A. Single lag screw B. Tension band wiring C. Single lag screw and a plate D. Dynamic compression plate

Correct answer: (B) Tension band wiring. Explanation: Tension band wiring is especially useful for reattachment of small fracture fragments that would be irreversibly damaged with passage of a screw. Intra-articular avulsion fracture of the styloid process of the fifth metatarsal. A single screw with a plate as well as a dynamic compression plate both require screw insertion which may damage the bone. Therefore answer choice B is the correct choice.

Which of the following refers to the portion of the AO/ASIF screw that is located within the thread pattern? A. The runout B. The core C. The shaft D. The rake

Correct answer: (B) The core. Explanation: The core of the screw refers to the portion of the shaft extending within the thread pattern. Increasing the width of the core allows the screw to be more resistant to bending loads. For example, two 4.0-mm cancellous screws may be an appropriate choice of fixation for a calcaneal osteotomy. The 4.0-mm screw has a 1.9-mm core diameter. Because the screw is inserted entirely within cancellous bone, the thread pattern of the 4.0-mm screw seems appropriate. Another option is a 3.5-mm fully threaded cortical screw with a core diameter of 2.4 mm. Theoretically, the thread pattern of the 3.5-mm screw is designed to provide more aggressive purchase in cortical bone, an anatomic impossibility when stabilizing a calcaneal osteotomy with screws inserted from posterior to anterior. In this case, the 3.5-mm screw is the biomechanical screw of choice because of its greater ability to resist bending and traction forces imparted by the Achilles tendon. The 4.0-mm cancellous screws are less tolerant of such forces and are more likely to undergo progressive fatigue and subsequent failure. In considering which implant is most appropriate for a particular construct, it is worthwhile to develop a hierarchy of biomechanical requirements necessary to achieve complete osseous stabilization.

A 50 year diabetic male presents to the emergency department with severe pain to the left foot which started when he sustained trauma to the left foot. He appears lethargic, diaphoretic, and apathic, with fever. The left foot appears tense with swelling skin that blanches in some areas with a large broken bullae with necrosis, and foul- smelling brown red drainage containing gas bubbles. The patient is diagnosed with clostridial myonecrosis (gas gangrene). The podiatric surgical team plans to take him to the operating room as soon as possible. The patient is also on warfarin therapy for a deep venous thrombosis sustained 3 months ago. His internationalized normalized ratio (INR) is 2.8. What will be the best way to reverse his anticoagulation in preparation of the surgical procedure? A. Oral vitamin K tablets B. Subcutaneous vitamin K C. Fresh frozen plasma D. Protamine sulfate

Correct answer: (C) Fresh frozen plasma. Explanation: Reversal of anticoagulant is often required when emergent or urgent surgical procedures are required. Reversal of anticoagulation is typically achieved with the administration of fresh frozen plasma or vitamin k. FFP has an immediate onset of action, its effects are short- lived, and it does not cause resistance to anticoagulation with warfarin postoperatively. The INR should be monitored after initial administration of FFP and every few hours subsequently to determine if additional treatment is needed to achieve the INR goal. For less urgent situations, such as surgery to be performed within 24 to 96 hours, vitamin K is the treatment of choice. Intravenous vitamin K will provide a more rapid reversal than any other routes of administration. Oral vitamin K is only available in 5-mmg tablets in the United States; these tablets are scored allowing for easy administration of 2.5 mg. Lower doses can be administered by having the patient drink the intravenous preparation. Subcutaneous delivery of vitamin K used to be common place; it has been shown that absorption by this route is not predictable and thus no longer recommended.

Which one of the following is a criteria of the lag effect? The screw in the near side fragment, threaded or non-threaded, has added benefit to being solidly arrested to the bone on the near side- however this is not required. Threads in the far side fragment screw hole must continue to progress through the bone as the screw turns and must firmly grip the bone without stripping. The screw in far side fragment, threaded or non- threaded, must glide freely in the far side screw hole engaging the bone. The advancement of the head of the screw must be solidly arrested on the surface of the far side.

Correct answer: (B) Threads in the far side fragment screw hole must continue to progress through the bone as the screw turns and must firmly grip the bone without stripping. Explanation: One of the most powerful and useful compression methods is that of the lag screw. Lag literally means "to follow". Lag screws are one of the most common forms of fixation. Any screw type may be used to produce a lag effect as long as three criteria are met: 1) The screw in the near side fragment, threaded or non-threaded, must glide freely in the near side screw hole without engaging the bone. 2) Threads in the far side fragment screw hole must continue to progress through the bone as the screw turns and must firmly grip the bone without stripping. 3) The advancement of the head of the screw must be solidly arrested on the surface of the near side.

During a thorough history and physical for a patient undergoing forefoot surgery, the patient is found to have a chronically elevated T3 and T4. What should be of the utmost concern regarding this patient when planning surgery? A. No concern while anesthesia is on board B. Thyrotoxicosis C. Hashimoto thyroiditis storm D. Sheehan syndrome

Correct answer: (B) Thyrotoxicosis. Explanation: Thyrotoxicosis is a rare and potentially fatal complication of hyperthyroidism in patients with chronically elevated free T3 and T4 with suppressed TSH. It typically occurs in patients with untreated or partially treated thyrotoxicosis who experience a precipitating event such as surgery, infection, or trauma. Thyroid storm must be recognized and treated on clinical grounds alone, as laboratory confirmation often cannot be obtained in a timely manner. Patients typically appear markedly hyper metabolic with high fevers, tachycardia, nausea and vomiting, tremulousness, agitation, and psychosis. Late in the progression of disease, patients may become stuporous or comatose with hypotension.

Which diagnostic exam is the most useful in evaluation of an interdigital neuroma? A. radiographs B. ultrasonography C. magnetic resonance imaging D. sensory nerve conduction tests

Correct answer: (B) Ultrasonography. Explanation: In the right circumstances each of the four modalities listed above display some utility, however diagnostic ultrasound is by far and away the most useful. Ultrasonography has been shown to have a 95% to 98% accuracy when compared to intraoperative findings. On an ultrasound the neuroma typically is has the appearance of an oval, hyperechoic mass that runs parallel to the long axis of the metatarsals. Radiographs generally are most commonly used to rule out other osseous pathology that could produce similar symptoms. One occasionally seen radiographic sign is an increase in the intermetatarsal angle of the affected region. Nerve conduction studies for interdigital neuromas lack accuracy as there is extreme difficulty isolating an intermetatarsal nerve. False negatives are relatively common with magnetic resonance imaging most often due to the lesion fallen in between the image slices. With this fact and the cost of the exam, MRI is generally reserved with recurrent neuromas or in atypical presentations.

What is the effect on mu receptors in the periphery in response to injury and inflammation as created by surgery? A. down regulation B. up regulation C. blockade D. no effect

Correct answer: (B) Up regulation. Explanation: In response to an injury, such as surgery or trauma, the mu receptors in the periphery of the body are up regulated. This up regulation of the mu receptors enhances the degree of anesthesia that can be provided by opioids at these periphery receptor sites. This up regulation occurs via axonal transport of mu receptors from the dorsal root ganglions out to the periphery. In simple terms, this means that a greater pain-relieving effect will occur at the site of surgery.

Which of the following imaging results best supports a diagnosis of acute osteomyelitis? A. (+)Tc-99m MDP in phase 1 & 2; (-)Tc-99m MDP in phase 3 B. (+)Tc-99m MDP in phase 1, 2 & 3; (-)Gallium-67 citrate C. (+)Tc-99m MDP in phase 1, 2 & 3; (+)Indium-111 D. (-)Tc-99m MDP in phase 1, 2 & 3; (-)Tc-99m HMPAO/Ceretec

Correct answer: (C) (+)Tc-99m MDP in phase 1, 2 & 3; (+)Indium-111. Explanation: Technetium (Tc-99m methylene diphosphonate) is commonly referred to as a bone scan or scintigraphy, and is a type of nuclear medicine study that is very sensitive at detecting subtle increases in metabolic activity. Where this type of study falls short is its specificity as it is difficult to differentiate one cause of bone turnover from another cause unless a secondary study is performed. When this radioisotope is injected into the subject, it accumulates in areas of physiologic activity or cellular turnover. This cellular activity occurs in response to infection, tumors, fractures, arthritides, and surgery (to name a few). The traditional technetium bone scan includes 3-phases and these are: 1) Blood Flow a. Occurs immediately after radioisotope injection b. Is basically an angiogram that reflect vascularity 2) Blood Pool a. Occurs a few minutes after radioisotope injection b. Reflects soft tissue involvement 3) Delayed/Bony Uptake a. Occurs 2-4 hours after radioisotope injection b. Reflects osteoblastic activity and physiologic response c. At this phase, the soft tissue has been cleared of isotope d. Isotope is now incorporating into bone where there is increased metabolic activity If Tc-99m MDP is positive in only the 1st and 2nd phases, then that means that the physiologic activity is restricted to the soft tissue and does not involve the bone. Tc-99m MDP would have to be positive in all three phases for there to be osseous involvement, ie. Osteomyelitis. Therefore, answers (A) and (D) can be crossed out as there is not isotope uptake in all three phases so bone pathology is not present. Gallium and Indium are often used in conjunction to Tc-99m scan as it improves specificity for infection. Both of these radioisotopes do not localize in healing bone but rather accumulate in areas of infection. If a Tc-99m scan is positive in all three phase, we know that bone is involved but we don't know the reason for the physiologic activity. When Gallium is negative, even in the presence of a positive 3-phase scan, we can feel comfortable saying that there is no acute osteomyelitis. Therefore answer (B) is incorrect. Because the 3-phase bone scan and Indium-111 are positive, we know that isotope has accumulated in bone due to the presence of infection. This is why (C) is the correct answer.

The peak analgesic effect of morphine sulfate is how long after administration: A. 1-2 minutes B. 5-10 minutes C. 30-60 minutes D. 60-90 minutes

Correct answer: (C) 30-60 minutes. Explanation: Morphine is a strong agonist of the mu receptor. The bioavailability of morphine is 15-50% due to extensive metabolism on first pass through the liver. Metabolism of morphine sulfate occurs in the liver by morphine-3-glucuronide and normorphine. The metabolites are excreted primarily through urine. Furthermore, the half-life of morphine is 1.7-4.5 hours. The peak analgesic effect of morphine sulfate is 30 to 60 minutes after intravenous administration and a duration of 3-4 hours.

In which of the following scenarios would spinal anesthesia be relatively contraindicated? 58 yo diabetic M presenting for Charcot reconstruction of left foot with a pre-op blood sugar of 228 65 yo F undergoing elective hammertoe repair of bilateral feet 46 yo M with a past history of a DVT, whose last dose of warfarin was 16 hours ago, undergoing non-elective repair of an open ankle fracture 60 yo F who suffered an MI 8 months ago, presenting for elective repair of a right foot bunion deformity

Correct answer: (C) 46 yo M with a past history of a DVT, whose last dose of warfarin was 16 hours ago, undergoing non-elective repair of an open ankle fracture. Explanation: Spinal anesthesia has few absolute contraindications, but relative contraindications must be weighed against the benefits obtained from spinal anesthesia. Diabetes is not a relative contraindication to spinal anesthesia, even with an elevated pre-operative blood sugar. Answer B does not give any additional information that would indicate any relative contraindications to spinal anesthesia. A past history of a heart attack is not a relative contraindication for spinal anesthesia, although if the patient had suffered a heart attack less than 6 months prior to surgery, their overall risk would be elevated and it would be recommended to delay elective surgery. Anticoagulation is a relative contraindication to spinal anesthesia due to concerns with increased risk of development of an epidural hematoma. Other relative contraindications to spinal or epidural anesthesia include sepsis, local cellulitis, hypovolemia or shock, and increased intracranial pressure.

A patient is having surgery in the second interspace. During the surgery, the podiatrist lets the resident work on the dissection and the resident accidentally cuts the 1st plantar interossei muscle. Which of the following can be a long term outcome of this accidental cut and what plantar layer is the 1st plantar interossei located in? A. Abduction deformity 3rd toe, 3rd plantar layer B. Adduction deformity 3rd toe, 4th plantar layer C. Abduction deformity 3rd toe, 4th plantar layer D. Abduction deformity 2nd toe, 3rd plantar layer

Correct answer: (C) Abduction deformity 3rd toe, 4th plantar layer. Explanation: Both the dorsal and plantar interossei are located in the 4th layer of the plantar compartment of the foot. The interossei function to assist with flexion of the MTPJ of the toes and extension of the proximal interphalangeal joint. The plantar interossei insert onto the medial aspect of the base of the proximal phalanx of toes 3/4/5. They adduct the toes in the transverse plane. Damage to the plantar interossei will cause abduction of the affected toe as there will be no adduction and there will be overpowering of the abductors.

Which of the following is true regarding insertional Achilles tendonitis? Commonly seen in runners and athletes. Often seen in young women with rheumatoid arthritis. Achilles tendonitis and retrocalcaneal bursitis are often seen with insertional posterior heel pain. Conservative treatment is often successful in resolution of symptoms.

Correct answer: (C) Achilles tendonitis and retrocalcaneal bursitis are often seen with insertional posterior heel pain. Explanation: Insertional achilles tendonitis affects two very different patient populations. The first is an older, less athletic, overweight and sedentary individual. The second is young adult males with seronegative spondyloarthropathies. Typified by posterior heel pain at achilles attachment that increase with standing, walking or running. The condition is aggravated by either passive or active range of motion. Equinus commonly accompanies the condition. Achilles tendonitis and retrocalcaneal bursitis are often seen with insertional posterior heel pain. Conservative treatment often can be helpful early, but majority of cases progress to surgical intervention.

A 73 year old male patient with uncontrolled Diabetes goes in for an emergency amputation of his right hallux. The metatarsal head is inspected and no infection noted in it. Which of the following muscles that normally would insert into the proximal phalanx of the hallux would be affected by this amputation? A. Abductor Digiti Minimi B. Flexor digitorum longus C. Adductor hallucis D. Extensor digitorum longus

Correct answer: (C) Adductor hallucis. Explanation: To answer this question effectively, you need to know where the muscles in the foot insert. This patient had a hallux amputation so any muscles inserting into the hallux would be affected. In this case, the only muscle listed that inserts into the hallux would be the adductor hallucis. The remaining muscles do not insert into the hallux. Other intrinsic muscles that insert onto the hallux are the flexor hallucis brevis and the abductor hallucis.

Congenital vertical talus: A. Is most frequently seen as an isolated anomaly. B. Is easily distinguishable from flexible flatfoot. C. Age and severity of the deformity dictate the surgical procedure. D. Is successfully treated via serial casting.

Correct answer: (C) Age and severity of the deformity dictate the surgical procedure. Explanation: Congenital convex pes valgus is not commonly seen as an isolated deformity. It is frequently seen in association with neuromuscular disease; myelomeningocele and arthrogryposis are examples. Although there are some seemingly obvious characteristics of vertical talus that distinguish it from a flexible flatfoot deformity, at times, especially in the very young child, identifying the difference can be difficult. Clinically, there is a rounded prominence to the plantar/medial aspect of the foot. The foot is dorsiflexed at the midtarsal joint. Deep skin creases are seen on the dorsal/lateral aspect of the foot. As the child begins to walk, the heel does not touch the ground due to its equinus positioning, and the forefoot is abducted. Radiographically, the flexible flatfoot deformity can also demonstrate a severely plantarflexed talus, but usually does not demonstrate the severe TNJ dislocation seen with vertical talus. To differentiate between the two radiographically, a lateral plantarflexion stress view will confirm vertical talus. In children less than four years of age, open reduction of the TNJ and STJ is indicated. Between 4-8 years of age, open reduction and soft tissue rebalancing is indicated; possibly adding an extra-articular STJ arthrodesis. Excision of the navicular may be necessary in children older than three, with severe deformity. Older than twelve, a triple arthrodesis may be the only option. Before surgery, serial casting can be attempted, but the deformity tends to recur, especially because of failure to reduce the TNJ.

An advantage of a dorsal incisional approach for neuroma excision versus a plantar transverse approach is: A. The intermetatarsal ligament is incised B. Significant dissection C. Allows for immediate weight bearing D. Greater chance of hematoma

Correct answer: (C) Allows for immediate weight bearing. Explanation: A dorsal incisional approach does allow immediate weight bearing post operatively. However the other choices are true disadvantageous statements. There is a significant amount of dissection needed to reach the plantar neuroma from the dorsal approach. In this dissection the transverse inter metatarsal ligament has to be incised and there is a greater chance of bleeding and post-operative hematoma.

A patient on anticoagulation therapy is scheduled for a closing base wedge osteotomy. What is the reasonable goal of international normalized ratio (INR) at which surgery can safely be performed? A. An INR of < 2.5 is typically a reasonable goal B. An INR of < 2.0 is typically a reasonable goal C. An INR of < 1.5 is typically a reasonable goal D. An INR of > 1.5 is typically a reasonable goal

Correct answer: (C) An INR of < 1.5 is typically a reasonable goal. Explanation: Though the international normalized ratio at which surgery can safely be performed is subjective, an INR of <1.5 is typically a reasonable goal. The importance of preventing venous thromboembolism is undisputed. Venous thromboembolism is the second most common cause of postoperative medical complication and third most common cause of excess mortality in the postoperative patient. Pulmonary embolism is the most common preventable cause of hospital death. Without prophylaxis, the risk of developing venographically apparent DVT among surgical patients is between 15% - 40% in general surgery, 40%to 60% in lower extremity orthopedic patients, and greater than 60% in trauma patients. VTE must be a priority in the postoperative setting.

The last component of Talipes Equinovarus to be treated in casting sequence is: A. forefoot adductus B. subtalar inversion C. ankle equines D. forefoot varus

Correct answer: (C) Ankle equines. Explanation: Ankle equinus should be corrected only after the midtarsal and subtalar joints have been restored to alignment. This avoids the complication of mid sagittal breech which can occur if the equinus is corrected too early. Correction of the equinus is delayed until the navicular and cuboid are reduced and there is divergence between the talus and calcaneus, because the calcaneus cannot dorsiflex unless it is everted. The dorsiflexion force should be applied to the anterior calcaneus rather than to the forefoot to avoid stretching the mid tarsal joints, which can result in mid tarsal breech or a rocker bottom deformity.

A 33 year old female presents to the emergency room complaining of pain and swelling to the right ankle. The evening prior she had twisted her ankle when her high heel got caught in a street drainage grate. Pictured below are radiographs from her emergency room visit. What is the likely diagnosis? A. Fracture of the lateral process of the talus B. Shepherd's Fracture C. Anterior process fracture of the calcaneus D. Fracture of the calcaneal posterior tuberosity

Correct answer: (C) Anterior process fracture of the calcaneus. Explanation: Pictured above is a fracture of the anterior process of the calcaneus. Key triggers in the question include the fact the patient is a female and it occurred while wearing high heels. The mechanism of this fracture, inversion and plantarflexion, forces placed on the anterior process of the calcaneus, cause the bifurcate ligament to become taut and avulse the anterior process of the calcaneus. This is the most common and accepted mechanism of injury for this fracture. Women have a higher incidence of this fracture secondary to wearing high-heeled shoe gear that plantar flexes the foot on the leg and subsequently unlocks the ankle joint, thereby predisposing them to ankle sprains. There are two other possible mechanisms. A second possible mechanism occurs when a foot is forcefully dorsiflexed against an everted rearfoot. A third possible mechanism is a forceful abduction of the forefoot and fixed hindfoot. A fracture of the lateral process of the talus is caused by a forceful dorsiflexion on an everted foot. A Shepherd's fracture is a fracture of the posterior lateral talar process.

Which of the following is correct in reference to cadence during the gait cycle? A. Will increase with age B. As cadence increases, both stance and swing time increases C. As cadence increases, both stance and swing time decreases D. As cadence increases, double limb support increases

Correct answer: (C) As cadence increases, both stance and swing time decreases. Explanation: As cadence increases, both stance and swing time decreases. Cadence, or step rate, is defined as the number of steps per minute when a person walks. Most people will select a natural or free cadence. The average cadence ranges from 101 to 122 steps per minute. Women have a faster cadence than men (117 vs. 111 steps per minute). Cadence generally slows down with age. It should be noted that as cadence increases, both stance and swing time decrease, but stance phase decreases 3.5 times as rapidly as swing phase. In addition, an increase in cadence shortens both double support periods. Murray et al in a study of older men, included subjects up to 87 years old and noted that for those older than 65, both cadence and stride length decreased.

Interruption of the axons of a nerve that results in degeneration of its distal segment without appreciable damage to the supporting structures (endoneurium; perineurium; epineurium) of the nerve at the site of the injury allowing for spontaneous and good regeneration potential. What term below best describes this condition? A. Neuropraxia B. Neurotmesis C. Axonotmesis D. Neuritis

Correct answer: (C) Axonotmesis. Explanation: The above is a description of axonotmesis. Axonotmesis can best be described as interruption of the axons of a nerve that results in degeneration of its distal (peripheral) segment (wallerian degeneration) without appreciable damage to the supporting structures (endoneurium; perineurium; epineurium) of the nerve at the site of the injury. This allows for spontaneous and good regeneration potential. Neuropraxia is best describes as the mildest type of focal nerve lesion that produces clinical deficits; localized loss of conduction along a nerve without axon degeneration; caused by a focal lesion, usually demyelinating, but occasionally ischemic (when of shorter than a few hours' duration), followed by a complete recovery. Neurotmesis is the most severe nerve injury is a partial or complete severance of a nerve, with disruption of the axon and its myelin sheath and the connective tissue elements. Neuritis is simply inflammation of the nerve.

The American Society of Anesthesiologists has set the basic standard for the evaluation of patients prior to surgery. The classification is designed to estimate overall mortality risk in patients undergoing surgery. Which class has a 2-3 fold increased risk of post-operative complications? A. Class I B. Class II C. Class III D. Class F

Correct answer: (C) Class III. Explanation: The ASA classification is designed to estimate the mortality risk in patients undergoing surgery; it also predicts cardiovascular and pulmonary complications. Patients who are graded higher than class III have a 2-to- 3 fold increased risk of postoperative pulmonary complications compared with those graded class II or lower in the five class. (A) Class I is designed for a normal healthy patient, a patient with good exercise tolerance. A healthy, non-smoking, no or minimal alcohol. (B) Class ASA II Is designed for a patient with mild systemic disease. Mild diseases only without substantive functional limitations, examples include but not limited to current smoker, social alcohol drinker, pregnancy, obesity, well controlled DM/HTN, mild lung disease. (D) There is no class F, but there is a class E indicating an emergency surgery, defined as existing when delay in treatment of the patient would lead to a significant increase in the threat to life or body part.

A retired army Para jumper presents to your office for evaluation secondary to chronic ankle pain for many years. Patient has a history of jumping out of planes with the military for missions. Patient states that the pain is a dull ache in the ankle that is worst with running, jumping and walking. Patient gets improvement with resting. X-ray taken shows no abnormalities in the ankle joint other than mild joint space narrowing. An MRI is ordered which shows a completely detached osteochondral fracture fragment that remains in the defect. Which of the following is the correct classification of this lesion? A. Berndt Harty Stage 1 B. Berndt Harty Stage 2 C. Berndt Harty Stage 3 D. Berndt Hardy Stage 4

Correct answer: (C) Berndt Harty Stage 3. Explanation: Athletes and military personnel commonly develop osteochondral lesions in the talar articular surface and tend to be described as painful with ambulation. Lesions that are seen on the anterior-lateral aspect of the talar dome tend to be caused by a dorsiflexion and inversion force and those seen on the posterior and medial aspect of the talar dome tend to be caused by an inversion and plantarflexion force. A stage 1 injury is a small area of compressed subchondral bone, usually not visible on x-ray. A stage 2 is a partially detached fracture fragment. Stage 3 is a completely detached fragment of subchondral bone that remains in the defect. Stage 4 is a completely detached fracture fragment that is loose in the joint.

Which of the following is defined as an abnormal shortness of the toes? A. Hemimelia B. Amelia C. Brachydactyly D. Sympodia E. Polydactyly

Correct answer: (C) Brachydactyly. Explanation: Brachydactyly is an abnormal shortness of the toes. Syndactyly is the fusion or fleshy webbing between the digits of the hands or feet. Hemimelia is the absence of all or part of the distal limb, while the proximal limb is normal. Amelia is a congenital absence of a limb or limbs. Phocomelia is a congenital absence of the proximal limbs such that the feet or hands are attached to the trunk by an anomalous segment. Sympodia is the fusion of the lower extremities. Polydactyly is supernumerary digits on the hands or feet.

What radiographic view is best suited to evaluate intra-operative position and correction during a medial displacement calcaneal osteotomy? A. Lateral oblique B. Lateral C. Calcaneal axial D. Anterior posterior

Correct answer: (C) Calcaneal axial. Explanation: The calcaneal axial view is the ideal radiological view to assess intra-operative position and correction while performing a medial displacement calcaneal osteotomy. This can easily be performed with fluoroscopy. (A) The lateral oblique view does not provide adequate information to appropriately evaluate intra-operative position and correction during a medial calcaneal osteotomy. (B) The lateral view does not provide adequate information to appropriately evaluate intra- operative position and correction during a medial calcaneal osteotomy. (D) The anterior posterior view does not provide adequate information to appropriately evaluate intra-operative position and correction during a medial calcaneal osteotomy.

Which type of tarsal coalition is most common in pediatric patients? A. Talonavicular coalition B. Talocalcaneal coalition C. Calcaneonavicular coalition D. Calcaneocuboid coalition

Correct answer: (C) Calcaneonavicular coalition. Explanation: Coalitions are defined as bony or connective tissue bridges between two bones. In the pediatric population, the most common tarsal coalition is the calcaneonavicular coalition, most commonly symptomatic between ages 6 to 12. This coalition is best visualized on a lateral oblique x-ray, which may display the "anteater nose sign." This is the anterior process of the calcaneus toward the navicular. (A) Talonavicular coalitions can become symptomatic in children as young as 2 years old, as ossification generally occurs between 3-5 years old. Symptoms are generally correlated to the medial aspect of the foot. (B) Talocalcaneal coalitions are generally not symptomatic until adolescence, as ossification generally occurs between 12-16 years old. This type of coalition can cause a slight valgus deformity of the rearfoot and cause decrease in mobility. (D) Calcaneocuboid coalitions are not very commonly seen in the pediatric/adolescent population.

A 53 year old male is admitted to the hospital prior to total ankle joint replacement. Medical history is consistent with well controlled diabetes and hypertension. Patient does have a history of MI 10 years ago and now follows with cardiologist routinely. The anesthesiologist comes in for her peri-operative evaluation. What ASA classification would this patient be considered? A. Class 1 B. Class 2 C. Class 3 D. Class 4

Correct answer: (C) Class 3. Explanation: During the perioperative anesthesia assessment, the anesthesiologist will give the patient an ASA classification (American Society of Anesthesiologists). ASA 1 is a normal healthy patient excluding the very old and very young. ASA 2 is a patient with mild systemic disease (well controlled DM), pregnancy, mild obesity. ASA 3 is a patient with severe systemic disease or an old MI. ASA 4 is a patient with severe systemic disease that is a constant threat to life. ASA 5 is a patient not expected to survive without the operation and ASA 6 is a brain dead patient whose organs are being harvested for donor purposes. Hence in this question since the patient has an old MI, he is an ASA class 3.

A diabetic patient presents to the emergency department and is diagnosed with gas gangrene to the left foot and is scheduled for an emergency fasciotomy. What is the ASA class assigned to this patient? A Class I B Class II C Class E

Correct answer: (C) Class E. Explanation: Class E indicates an emergency surgery, defined as existing when delay in treatment of the patient would lead to a significant increase in the threat to life or body part, this patient falls into that category. (A) Class I is designed for a normal healthy patient, a patient with good exercise tolerance. A healthy, non-smoking, no or minimal alcohol. (B) Class ASA II Is designed for a patient with mild systemic disease. Mild diseases only without substantive functional limitations, examples include but not limited to current smoker, social alcohol drinker, pregnancy, obesity, well controlled DM/HTN, mild lung disease.

The maximal stress a material can undergo without permanent deformation is also known as this? A. Passivation B. Creep C. Elastic limit D. Elongation E. Modulus elasticity

Correct answer: (C) Elastic limit. Explanation: Elastic limit Is the maximal stress a material can undergo without permanent deformation. Creep is the process of a solid material to slowly and permanently migrate or deform while mechanical stresses that are below yield strength for the material. Passivation is production of corrosion resistance by a surface of reaction products. This layer is normally an oxide layer, which is impervious to gas and water. Elongation is the amount of deformation that a tensile stress or load will produce. A tensile stress is the force that attempts to pull apart or stretch a material. Modulus elasticity is the measure of stiffness of a substance along its elastic range. It is calculated by dividing the stress load by the amount of strain or deformation. The strain is the ratio of change in a dimension that takes in a material under stress.

A 25 year old female presents to your office for evaluation of a bunion deformity. She has a history of seizures and is currently on Depakote. You schedule her for a Scarf osteotomy of the right foot. What is the best perioperative regimen for her anti-epileptic medication? A. Discontinue her Depakote 24 hours before surgery. B. Discontinue Depakote 3 days before surgery. C. Continue Depakote. D. Discontinue Depakote, order an EEG.

Correct answer: (C) Continue Depakote. Explanation: Epilepsy is abroad term encompassing the various seizure disorders. Seizures maybe partial, focal, or generalized, patients with epilepsy are known to have an increased risk for operative seizure secondary to the choice of anesthetic or physiologic derangement. Most patients are treated with antiepileptic drugs. The AED such as Depakote in this case should be continued during the perioperative period. Clinical signs of toxicity such as ataxia, dizziness, and confusion should be investigated and corrected. (A) Depakote should be continued to avoid a risk of perioperative seizure. (B) AED should be continued throughout the operative period, toxicity levels should be evaluated. (D) Unless, subject to seizures in the perioperative period, and if the patient is well controlled, there is no emergent need of EEG, and AED should be continued. ***AED: Antiepileptic drugs

A 23 year old female ballet dancer presents to the clinic complaining of sharp pain under her first metatarsophalangeal joint for approximately 3 weeks. The following radiograph was performed at her visit. A segmented fibular sesamoid is noted on the radiograph. The clinician wants to distinguish between a fibular sesamoid fracture and a bipartite fibular sesamoid with sesamoiditis. What would be the first step to initiate this process? A. Bone scan B. MRI C. Contralateral radiograph D. Ultrasound imaging

Correct answer: (C) Contralateral radiograph. Explanation: The key to this question is answering in terms of cost effectiveness. With that said obtaining a contralateral radiograph would be the most cost effective way to initiate the process of distinguishing a fractured sesamoid from a bipartite sesamoid. If the contralateral foot shows a sesamoid as one unit in its entirety then this is likely a fractured sesamoid. On the contrary, if the contralateral side shows a bipartite sesamoid then this is less likely a fractured sesamoid and may be sesamoiditis. A bone scan and an MRI would likely accomplish the same goal, but would be significantly more expensive. An ultrasound is of no value in this instance.

An example of a direct thrombin inhibitor which can be utilized for DVT prophylaxis is: A. Rivaroxaban (Xarelto) B. Apixaban (Eliquis) C. Dabigatran Etexilate (Pradaxa) D. Warfarin

Correct answer: (C) Dabigatran Etexilate (Pradaxa). Explanation: Dabigatran is an oral, direct thrombin inhibitor. It prevents thrombus formation by inhibiting free and fibrin-bound thrombin. Its half-life is 14-17 hours. Verapamil and quinidine will increase plasma levels of dabigatran. The only recommendations thus far are for hip and knee surgery; one 110mg capsule is given 1-4 hours after surgery. For hip surgery, 220mg daily for 28-35 days. For knee replacement surgery, 220mg daily for ten days. Regarding discontinuing the drug before surgery to reduce bleeding complications, it is based upon creatinine clearance. Discontinue 1-2 days before surgery with a creatine clearance greater than 50. If less than 50, a 3-5 day timeframe is recommended, as the drug is 80% excreted unchanged through the kidney. (A) Rivaroxaban is a Factor Xa inhibitor. It is FDA approved for treatment of DVT and PE, reduction of risk of recurrence of DVT and PE, and DVT prophylaxis; all following hip or knee surgery. The half-life can be up to ten hours. Drugs such as phenytoin, St. John's Wort, azole antifungals, and macrolide antibiotics are contraindications to use. Currently, there are only recommendations for hip and knee surgery. An initial dose of 10mg is taken 6-10 hours after surgery. Duration of treatment is 35 days following hip surgery and 12 days following knee surgery. It may be discontinued 24 hours before surgery, to reduce bleeding complications. (B) Apixaban is a Factor Xa inhibitor, with the same indications as rivaroxaban. Its half-life is 12 hours. It is contraindicated in patients takin g CYP3A4 and P-gp inhibitor type drugs. The current recommendations following hip or knee surgery are 2.5mg 12-24 hours after surgery; 35 POD's for hip surgery and 12 POD's following knee replacement surgery. It can be discontinued 48 hours before surgery, to reduce bleeding complications. (D) Coumadin is a vitamin K antagonist, affecting synthesis of factors II, VII, IX, and X, and proteins C and S. It does not reach its' full anticoagulant effect for 4-5 days, primarily due to the long half-life of factor II. A parenteral anticoagulant must be utilized with warfarin; treatment with warfarin only is inadequate for the acute DVT/PE. Common podiatric medications that can increase warfarin's effect (increase the INR) include Cipro, Bactrim, Erythromycin, Lamisil,and Sporanox. Smoking and herbal medications can decrease warfarin's effect. Warfarin should be discontinued five days before elective surgery, to reduce bleeding complications.

In regards to the role of opioids in surgical anesthesia, what is the effect opioids have on the nociceptive stimulus of a skin incision? A. Accentuation B. Prolongation C. Dampening D. No effect

Correct answer: (C) Dampening. Explanation: One of the opioid's functions is surgical anesthesia is the blunting or dampening of nociceptive input, such as a skin incision amongst a plethora of surgical maneuvers. Opioids modulate nociceptive responses in two ways. The first being by inhibiting calcium influx into the presynaptic terminal which in turn blocks neurotransmitter release. The second manner is by opening potassium channels causing a hyperpolarization of the neurons and thereby inhibiting neuronal spike activity. Utilization of opioids in surgical anesthesia lessens physiologic stress during the operation created by nociceptive stimuli as well as make post operative pain control easier by dampening the overall nociceptive "peak" the body experienced during the surgery.

A 44 year old construction worker presents with intense ankle and hindfoot pain. He suffered an intra articular calcaneal fracture a little over 12 months prior and was treated non surgically. On exam, he has a widened and shortened calcaneus with nearly absent subtalar joint motion. Ankle joint motion is painful along the anterior ankle but he is able to just barely get to neutral position. The best treatment for this patient would be which of the following? A. Achilles tendon lengthening B. Triple arthrodesis C. Distraction bone block STJ fusion D. TTC fusion E. Ankle arthroscopy and in situ subtalar joint fusion

Correct answer: (C) Distraction bone block STJ fusion. Explanation: Distraction bone block arthrodesis will allow for the correction of lost calcaneal height and depressed talar declination angle in addition to relief of symptoms related to arthritis in the subtalar joint. Lateral wall decompression could be considered in patients with lateral impingement complaints.

Regarding the gait cycle, please fill in the gap with the appropriate answer: As cadence increases, _____________________? A. Swing phase increases B. Stance phase increases C. Double limb support decreases D. Double limb support increases

Correct answer: (C) Double limb support decreases. Explanation: As cadence increases, double limb support decreases. Cadence, or step rate, is defined as the number of steps per minute when a person walks. Most people will select a natural or free cadence. The average cadence ranges from 101 to 122 steps per minute, Women have a faster cadence than men (117 vs. 111 steps per minute). Cadence generally slows down with age. It should be noted that as cadence increases, both stance and swing time decrease, but stance phase decreases 3.5 times as rapidly as swing phase. In addition, an increase in cadence shortens both double support periods. Murray et al in a study of older men, included subjects up to 87 years old and noted that for those older than 65, both cadence and stride length decreased.

Your 50 y/o patient demonstrates a unilateral flatfoot. There is pain along the course of the tibialis posterior tendon. The patient cannot perform a single heel raise test, but the heels invert with the double heel raise test. There is ten degrees of heel valgus with stance, which is reducible. There is no DJD on x-ray. The best surgical procedure is: A. Debridement and synovectomy of the tibialis posterior tendon B. FDL transfer C. FDL transfer with PCDO D. Subtalar fusion

Correct answer: (C) FDL transfer with PCDO. Explanation: This patient demonstrates a classic Johnson and Strom Stage 2 PTTD. The single heel raise test is a clinical predictor of tibialis posterior tendon integrity. Because the patient cannot perform the test, a tendon transfer, either the FDL or FHL can be utilized, is indicated. The rearfoot is flexible; inversion of the heels with the double heel raise test is indicative of this. A PCDO will realign the calcaneus under the tibia. (A) This procedure is indicated in early PTTD; Stage 1 Johnson and Strom. There is a concern that this procedure should be limited to a non-progressive deformity. Some argue for a PCDO in addition, even in Stage 1, to prevent progression. (B) The FDL transfer will rebalance the peroneus brevis abduction forces, but by itself will not correct a flatfoot deformity. The literature seems quite consistent in determining that a FDL transfer by itself, in this clinical scenario, will not realign the heel under the tibia, and probably will fail. A posterior calcaneal displacement osteotomy should be added. (D) In their original article, Johnson and Strom recommended a STJ fusion for a Stage 3 PTTD. Stage 3, in their article, demonstrated fixed hindfoot valgus and pain both medially and laterally. There is no DJD, and the foot is flexible. Therefore, an arthrodesis is not indicated.

A 34 year old female presents complaining of a painful callus under her right first metatarsal head. She states just started noticing the callus about 6 months ago. She denies any trauma, change in activity or shoe gear. Approximately nine months earlier a Lapidus procedure (first tarsometatarsal arthrodesis) was performed on her for a hallux valgus deformity. She states she never has a callus at this location prior to her bunion surgery. What is the likely cause of the callus? A. First ray is in varus (under corrected). B. First ray is in valgus (overcorrected). C. First ray is plantarflexed. D. First ray is dorsiflexion.

Correct answer: (C) First ray is plantarflexed. Explanation: The likely cause of the sub first metatarsal head callus is excessive plantarflexion of the first ray as a result of malposition of the Lapidus procedure. This will lead to excessive loading of the first metatarsal head and the resultant callus. Sesamoiditis is another possible complication from a plantarflexed first ray. (A) A malpositioned varus first ray would not result in a sub first metatarsal head callus. (B) A malpositioned valgus first ray would not result in a sub first metatarsal head callus. (D) A malpositioned dorsiflexed first ray would not result in a sub first metatarsal head callus.

A patient with atrial fibrillation is scheduled to undergo a right ankle arthroplasty with implant. When is the appropriate time to stop warfarin prior to surgery? A. Warfarin does not need to be discontinued prior to surgery B. Four hours prior to surgery C. Five days prior to surgery D. Four weeks prior to surgery

Correct answer: (C) Five days prior to surgery. Explanation: If the surgery is not emergent, warfarin is generally stopped five days prior to surgery to achieve an INR <1.3. It is important to check the INR pre-operatively. (A) In the case of an emergent surgery, warfarin cannot always be stopped five days prior. However, this is an elective case and should undergo appropriate precautions for safe surgery. (B) Warfarin has a half-life of three days, therefore discontinuing the medication four hours prior to surgery will not make a significant impact. (D) It is not necessary to hold warfarin for four weeks leading to surgery. A more appropriate time period is five days to reach an INR of <1.3.

A 58 year old male smoker presents complaining of painful contracted toes on her left foot. Her medical history is positive for a cerebral vascular accident that occurred two years prior and primarily affects her left side. The second through the fifth toe are all involved and demonstrated nonreducible contracture. No adductovarus component is noted to the fourth and fifth digits. A semi-reducible supinated foot structure is noted on the left foot. Gait examination reveals a cal- caneus gait on the left extremity. What is the likely etiology to her hammertoes? A. Flexor Stabilization B. Extensor Substitution C. Flexor Substitution D. Extensor Stabilization

Correct answer: (C) Flexor Substitution. Explanation: Flexor substitution hammertoes occur when the long flexors fire longer in the gait cycle to compensate for weak triceps surae. Flexor substitution occurs in a supinated foot in the late stance phase of gait. This overpowers the long extensors and deforms the toe. Aductovarus deformity of the fourth and fifth toes is not seen with this mechanism. The weak triceps surae results in a calcaneus gait. (A) Flexor stabilization hammertoes occur when the long flexor tendons fire for a longer duration in the gait cycle to assist a failing tibialis posterior tendon in a pronated foot. The long flexors overpower the extensors in the tug-o-war. This then pulls the distal phalanx of the digit toward the sulcus of the foot therefore causing buckling of the digit. (B) Extensor substitution hammertoes are commonly encountered in cavus and anterior cavus foot types. Due to the plantarflexed nature of the forefoot the long extensor tendons fire for a longer period of time in the gait cycle to facilitate the clearance of the toes to the ground. This causes an uneven tug-o-war phenomenon and the long extensors overpower the long flexors. This deforms the toes and leaves very prominent extensor tendons tenting the skin on the dorsum of the foot. (D) Extensor stabilization is not a substantiated theory on hammertoe formation.

A 54 year old female smoker presents complaining of painful contracted toes on her left foot. Her medical history is positive for a cerebral vascular accident that occurred two years prior and primarily affects her left side. The second through the fifth toe are all involved and demonstrate reducible contracture. No adductor varus component is noticed to the fourth and fifth digits. A semi-reducible supinated foot structure is noted on the left foot. Gait examination reveals a calcaneus gait on the left extremity. What would be the most likely etiology to her hammertoes? A. Flexor Stabilization B. Extensor Substitution C. Flexor Substitution D. Extensor Stabilization

Correct answer: (C) Flexor Substitution. Explanation: Flexor substitution hammertoes occur when the long flexors fire longer in the gait cycle to compensate for weak triceps surae. Flexor substitution occurs in a supinated foot in the late stance phase of gait. This overpowers the long extensors and deforms the toe. Aductovarus deformity of the fourth and fifth toes is not seen with this mechanism. The weak triceps surae results in a calcaneus gait. Extensor substitution hammertoes are commonly encountered in cavus and anterior caves foot types. Due to the plantarflexed nature of the forefoot the long extensor tendons fire for a longer period of time in the gait cycle to facilitate the clearance of the toes to the ground. This causes an uneven tug-o-war phenomenon and the long extensors overpower the long flexors. This deforms the toes and leaves very prominent extensor tendons tenting the skin on the dorsum of the foot. Flexor stabilization hammertoes occur when the long flexor tendons fire for a longer duration in the gait cycle to assist a failing tibialis posterior tendon in a pronated foot. The long flexors overpower the extensors in the tug-o-war. This then pulls the distal phalanx of the digit toward the sulcus of the foot therefore causing buckling of the digit. Extensor stabilization - doesn't exist (stable ex lol)

External ring fixation with thin wires has a prevalent role in diabetic charcot neuroarthropathy reconsctructive foot surgery. All of the following are advantages that this type of fixation has over internal fixation except: External fixation can allow a degree of weight bearing on the operative foot. Post operative adjustments can be made as the clinical picture may change. Generally well accepted and tolerated by patients. The thin wires can exert interfragmental compression despite the compromised bone stiffness and density.

Correct answer: (C) Generally well accepted and tolerated by patients. Explanation: External fixators are generally not well tolerated by patients. In fact the term "cage rage" has been coined to describe the compelling urge that most patients feel to have the fixator removed. Reasons for poor tolerance are lack of psychological and social acceptance, injury to the contralateral limb, and injury to property. Despite patient complaints, the external fixture provides optimal stabilization that allows varying degrees of weight bearing. Likewise, if adjustments need to be made postoperatively, it is possible. Additional compression, distraction, and fixation can all be performed days or even weeks after the initial procedure. Unlike internal fixation, which largely relies on stiff, dense bone to provide compression, this is not required with the thin wires of an external fixator. It has been documented that patients with neuropathy demonstrate reduced bone stiffness and bone density. Bone stiffness is best thought of as a measure of cortical integrity and bone density as a measure of how compact the cancellous bone is.

Often patients with late stage III flatfoot have an inability to dorsiflex their ankle to 90°. When treating these individuals with a gauntlet ankle foot orthotic the lab will make the device slightly plantarflexed. What shoe accommodation is necessary in this scenario? A. Midsole rocker B. SACH heel C. Heel lift D. Medial flange

Correct answer: (C) Heel lift. Explanation: To accommodate the patient's inability to achieve 90° of dorsiflexion of the ankle the gauntlet ankle foot orthotic must be also plantarflexed. However, if a heel lift is not incorporated with the accompanying shoe then the device will be poorly tolerated. Without the heel lift the front of the brace will apply significant pressure to the anterior aspect of the leg during mid stance as the tibial attempts to pass over the foot. (A) midsole rocker would not provide any benefit to a slightly plantarflexed ankle foot orthotic. (B) A SACH heel would not provide any benefit to a slightly plantarflexed ankle foot orthotic. (D) A medial flange would not provide any benefit to a slightly plantarflexed ankle foot orthotic.

Which of the following is not a contraindication for electrical bone stimulation? A. Malunion B. Defect > 1/2 the bone diameter C. Infection D. Synovial pseudoarthrosis

Correct answer: (C) Infection. Explanation: Electrical bone stimulation is indicated for the following conditions: nonunion, delayed union, bone grafting, infection, Charcot arthopathy, avascular necrosis, acute fractures. It is contraindicated for malunions, synovial pseudoarthrosis and as an isolated treatment when there is a bony deficit > 1/2 the bone diameter. The manner in which electrical bone stimulation is indicated in the presence of infection is as it relates to osteomyelitis. When there is an infected fracture, arthrodesis or osteotomy stability of the bone is paramount. To that end electrical bone stimulation combined with internal and/or external fixation has a complimentary effect in stabilizing and healing the unstable bone.

In regards to the Ponseti technique for manipulation and reduction of the clubfoot deformity, a basic tenet is to never touch the calcaneus during manipulation. What is the rationale behind this tenet? A. It could misshape the calcaneus. B. It prevents calcaneal adduction. C. It prevents calcaneal abduction. D. It potentiates the equinus deformity.

Correct answer: (C) It prevents calcaneal abduction. Explanation: A basic tenet of the Ponseti technique is never touching the calcaneus during manipulation. Ponseti referred to this as Kite's error, because Kite advocated counter pressure at the calcaneocuboid joint level while manipulating the forefoot. Holding or lateral pressure on the calcaneus prevents its abduction and its rolling out from underneath the talus to reduce the heel varus and, later, the equinus. Firm pressure on the lateral head of the talus keeps it stable in the ankle mortise while the foot distally is abducted out from underneath it.

Which Tailor's bunionectomy procedure is indicated for this patient 11 years of age? A. Distal chevron osteotomy B. Hohmann type osteotomy C. Ludloff-type osteotomy D. Closing base wedge osteotomy

Correct answer: (C) Ludloff-type osteotomy. Explanation: The key to this question is to recognize the developmental stage of the fifth metatarsal and the apex of the deformity. The correct answer above is the Ludloff-type osteotomy. At 11 years of age there is an open growth plate at the neck of the fifth metatarsal. Additionally when one looks at the CORA (center of rotation angulation) for the above deformity it is noted to be located in the mid shaft diaphysral region of the bone. This angle is formed at the intersection of the bisection of the base of the metatarsal and bisection of the head and neck of the metatarsal. Both the distal chevron and the Hohmann osteotomy would violate the open physic. While the closing base wedge osteotomy would respect the open physis, its correction would occur proximal to the CORA thereby leaving a secondary deformity distally.

The anatomical structure most commonly mistakenly severed during endoscopic nerve decompression of a Morton's neuroma is the: A. DTML B. Interossei tendon C. Lumbrical tendon D. FDL tendon

Correct answer: (C) Lumbrical tendon. Explanation: The lumbrical tendon lies at the lateral plantar aspect of the DTML, and is inferior to the ligament. With its intimacy to the ligament, it can be mistakenly cut. (A) Releasing the DTML is the goal of endoscopic decompression. The key to successful decompression is isolating, separating, and severing the DTML. (B) The interossei are superior to the DTML, and do not have the intimacy to the DTML that the lumbrical tendon has. (D) The FDL tendon is centered under the metahead as it slips through Camper's chiasm; thus, is safely away from the DTML release.

A 60-year-old patient presents complaining of severe joint pain from osteoarthritis that has been worsening over the past year. He has always worked as a janitor in a large school but now states he would like to file for disability. His wife suffers from major depression, and he confides that it would be much easier for him if he could be home to care for her. You have treated him and his entire family for 40 years. You are sympathetic, but he has nearly full range of motion in the joints with 4+/5 strength bilaterally, intact sensation, and brisk DTRs. What is the correct response to this patient's request? Emphasize positive findings and/or exaggerate patient's functional limitations in disability evaluation to help him and his family. Since he has asked to do something unethical, discontinue doctor-patient relationship. Inform patient that you will fill out a disability form for his wife if she completes an evaluation. Discuss treatment options and listen sympathetically but do not distort patient's functional status to help him and his family. Inform disability administration of patient's intention to commit fraud.

Correct answer: (D) Discuss treatment options and listen sympathetically but do not distort patient's functional status to help him and his family. Explanation: A healthcare provider has the ability to justify exemption from work and to legitimize other forms of financial support in cases where patients may have chronic, overwhelming, or catastrophic illnesses. In some cases, a patient may not meet a standard definition of disability but may seem deserving of assistance. A healthcare provider should never distort medical information or misrepresent a patient's functional status in an attempt to be helpful to a patient. This action would jeopardize the healthcare provider's trustworthiness and ability to advocate for those who truly meet criteria for disability. Although healthcare providers will often find themselves confronted with patients who seem deserving of assistance, disability evaluation forms should reflect the truth and must be completely honest.

What is paramount in the intraoperative management of patients with a history of cardiovascular diseases? Dropping the mean arterial blood pressure by a least 10 mm Hg from the preoperative baseline. Aspirin should be stopped 5 days prior. Maintenance of adequate cerebral perfusion pressure. Warfarin should be discontinued until 48 hours after surgery when renal function is normal.

Correct answer: (C) Maintenance of adequate cerebral perfusion pressure. Explanation: The baseline neurologic deficits must be clearly defined and intracranial vascular anatomy should be elucidated when known. Any preoperative imaging should be made available. Substantial intracranial arterial stenosis, in particular of the posterior circulation, may be especially perfusion dependent. This can occasionally be shown clinically by the presence of recurrent, sometimes stereotypic transient ischemic attacks. In these situations, blood pressure management intraoperatively for maintenance of adequate cerebral perfusion is paramount. (A) Dropping the mean arterial blood pressure by at least 10 mm Hg from the preoperative baseline increases the risk of stroke by approximately four fold. (B) Aspirin should be continued if possible in patients with cardiovascular disease in the preoperative period. It is considered a stroke prophylaxis agent. (D) Warfarin can be continued if possible.

A skewfoot deformity is the late stage sequela of what preceding condition? A. Talipes equinovarus B. Calcaneovalgus C. Metatarsus adductus D. Calcaneus

Correct answer: (C) Metatarsus adductus. Explanation: The skewfoot deformity is the late stage sequela of untreated or inadequately treated metatarsus adductus. It can occur in older children, adolescents and adults. It is a combination of a rigidly adducted forefoot and partially or fully compensated flatfoot. Talipes equinovarus, or clubfoot, is not the predecessor to the skewfoot deformity. Calcaneovalgus is not the predecessor to the skewfoot deformity. A calcaneus deformity is not the predecessor to the skewfoot deformity.

In the Grand Rapids Arch Collapse Classification System (GRACC), Type 3 affects the: A. Gastrocnemius, with equinus, metatarsalgia, and plantar fasciitis B. Forefoot, with hallux valgus, hammertoes, and medial column incompetence C. Midfoot, with midfoot DJD and transverse arch collapse D. Hindfoot, with peritalar subluxation, hindfoot valgus, and spring ligament pathology

Correct answer: (C) Midfoot, with midfoot DJD and transverse arch collapse. Explanation: The GRACC is a simple and replicable system which provides an algorithm for treatment of forefoot, midfoot, rearfoot, and ankle pathology based upon the presence of equinus and progressive arch collapse. There is also a type 5 involving the ankle, with ankle valgus and DJD, and deltoid ligament affectation.

A primary reason for troughing following z-scarf osteotomy of the first metatarsal is: A. Two screw fixation with rigid ORIF principles B. Inadequate compensation for metatarsus adductus C. Osteotomy is performed in mostly diaphyseal bone D. Inadequate soft tissue dissection leading to osteotomy

Correct answer: (C) Osteotomy is performed in mostly diaphyseal bone. Explanation: Troughing may occur if the osteotomy is performed in mostly diaphyseal bone. This allows the cortical bony edge to displace into the medullary canal, which may cause elevation of the distal capital fragment. Ideally the cut should be performed from metaphyseal to metaphyseal regions so there is adequate cancellous bone to prevent troughing. (A) Two screw fixation is often used to stabilize the capital fragment following z-scarf osteotomy. The screws may be oriented in various positions due to surgeon preference, however correct ORIF principles in healthy bone generally provides adequate fixation. (B) Troughing is often a complication of surgical technique rather than relation to metatarsus adductus. (D) Inadequate soft tissue dissection may result in a shorter osteotomy which may lead to diaphyseal cuts, however this is not a primary reason for troughing. Adequate soft tissue dissection should be performed to allow the full length of the osteotomy desired by the surgeon.

A podiatrist is reviewing surgical anatomy prior to completing a Lapidus bunionectomy on a patient. Which of the following muscles tendons inserts into the base of the 1st metatarsal? A. Peroneus Brevis and Tibialis Anterior B. Extensor hallucis longus and peroneus longus C. Peroneus longus and Tibialis anterior D. Extensor Hallucis brevis and extensor hallucis longus

Correct answer: (C) Peroneus longus and Tibialis anterior. Explanation: The first metatarsal base is an important anatomical landmark when completing a Lapidus fusion. The tibialis anterior muscles tendon inserts into the base of this bone as well as into the medial cuneiform (many texts say it has more of an insertion into the medial cuneiform) The peroneus longus muscles tendon also inserts into the base of the 1st metatarsal bone. The peroneus brevis tendon inserts primarily into the base of the 5th metatarsal. The extensor hallucis longus and brevis do not usually have any insertion into the 1st metatarsal. They both insert into the hallux.

Which muscle has the primary effect on decelerating subtalar joint pronation during contact phase of gait? A. Anterior Tibialis B. Peroneus brevis C. Posterior tibialis D. Extensor Hallucis Longus

Correct answer: (C) Posterior tibialis. Explanation: The posterior tibialis has the primary effect of decelerating and resisting subtalar joint pronation owing to its perpendicular angle relative to the subtalar joint axis combined with its long lever arm. It has nearly twice as much leverage than the soleus muscle does to supinate the STJ. Electromyographic studies indicate the posterior tibialis has two peaks, one during contact phase and the other during propulsion. It also has a substantial effect on decelerating the internal rotation of the lower leg and forward movement of the tibia because of its proximal origin on the leg.

In a chemical matrixectomy, the chemical solution employs what type of injury to the matrix tissue to which it is applied? A. first degree burn B. second degree burn C. third degree burn D. A blistering reaction and elevated and sloughs the matrix

Correct answer: (C) Third degree burn. Explanation: The chemical solution, whether phenol or sodium hydroxide, creates a focused third degree burn of the matrix tissue to which it is applied. The resultant wound heals by secondary intent. It will drain serous exudate for several weeks as the wound rids itself of necrotic tissue. The chemical reaction and tissue destruction is self-limiting due to the layer of necrotic cells created by cellular destruction. The goal is to destroy just enough matrix to prevent nail regrowth. Excessive tissue destruction is to be avoided, as it will result in prolonged drainage and tissue slough. The degree of tissue destruction is dependent upon the underlying vascularity of the area, concentration of the chemical, amount of chemical applied, area of application, and duration of application.

After a tornado, the wounded are transferred to local hospitals. The American Red Cross has also set up a shelter for people displaced by the tornado. Which of the following is consistent with the HIPAA Privacy Rule? A. B. C. D. E. Providers cannot share information to seek payment for healthcare services administered during a disaster. Providers must get patient permission to notify anyone of the patient's location or condition. Providers may share patient information without consent as necessary to locate and notify family members or guardians. Providers must obtain patient consent to share information with organizations like the American Red Cross. Providers must not reveal identifying patient information to the press unless patient consent is obtained.

Correct answer: (C) Providers may share patient information without consent as necessary to locate and notify family members or guardians. Explanation: During a disaster, health care providers may share information as necessary to identify, locate, and notify family members, guardians, or anyone else responsible for the individual's care of the location, general condition, or death of the patient. If possible, the health care provider should get verbal consent. However, if the individual is incapacitated or unavailable, the provider may share information for these purposes without consent, when doing so is in the best interest of the patient. (A) Providers can share patient information to the extent necessary to seek payment for health care services provided during a disaster. (B) Providers can share patient information to provide treatment, to identify, locate, and notify members or anyone responsible for the patient's care of the patient's location, general condition, or death. (D) Providers may share information with disaster relief organizations that are authorized by law or by charter to assist in disaster relief efforts without consent if the information is needed for the organization to respond to the emergency. (E) Covered entities may notify the police, press, or public at large to the extent necessary to help locate, identify, or notify family members and other of the location and condition of their loved ones.

A patient presents ten weeks after sustaining a fracture through the neck of the talus. Radiographic exam was obtained. What radiographic finding pictured below verifies that the vascular supply is intact? A. Absence of degenerative arthritis B. Increased density of the talar body C. Resorption of the subchondral bone of the talar dome D. Increased trabeculation pattern across the fracture

Correct answer: (C) Resorption of the subchondral bone of the talar dome. Explanation: Avascular necrosis of the talus is a common complication after a talar neck fracture. The Hawkins sign is a radiographic indicator of intact vascularity. The Hawkins sign is described as resorption of the subchondral bone of the talar dome. This is evident in the radiograph above. Degenerative arthritis is a long term complication of any articular fracture and independent of vascularity. Increased bone density after a fracture would suggest a sclerotic process. Trabeculation across the fracture site shows healing of the fracture and does not directly reflect vascularity.

Patients with polio demonstrate a characteristically very high calcaneal inclination angle due to triceps surae paralysis while the tibialis anterior, foot intrinsics, and posterior muscle group are selectively preserved. A joint-sparing calcaneal osteotomy which can be utilized, in this scenario is the: A. Grice B. Dwyer C. Samilson D. Silver

Correct answer: (C) Samilson. Explanation: The Grice procedure has been used in poliomyelitis, specifically calcaneocavus. Originally, it was used in younger patients with a rearfoot valgus secondary to flaccid paralytic disorders, very commonly CP. But, it is not joint-sparing, but instead is an extra-articular STJ arthrodesis. Also, the Grice is not recommended for varus deformities, commonly seen in cavus feet. The Dwyer calcaneal osteotomy is a laterally based calcaneal osteotomy with the medial hinge intact. As such, it cannot be repositioned to resolve the high calcaneal inclination angle. It is indicated for a fixed rearfoot/calcaneal varus common with cavus feet, as determined by the Coleman lateral block test. The Silver is a calcaneal osteotomy for the collapsing pes valgo planus deformity. It is an opening calcaneal osteotomy approached from the lateral side, with insertion of bone graft to reduce the rearfoot valgus. Preferably, the medial cortex remains intact, and thus, like the Dwyer, cannot be repositioned to reduce the high calcaneal inclination angle. That leaves the Samilson; a sliding type calcaneal osteotomy approached through a lateral incision. A through-and-through osteotomy is performed, with the posterior fragment displaced dorsally, which decreases the arch and the high calcaneal pitch. A modification includes removal of a wedge of bone laterally to reduce an associated varus component. Fixation can be with screws in the range of 6-8mm thread diameters, or step staples. Another procedure which can accomplish this is the Koutsogiannis.

What tissue is initially primarily affected with malignant hyperthermia? A. cardiac muscle B. smooth muscle C. skeletal muscle D. hypothalamic tissue

Correct answer: (C) Skeletal muscle. Explanation: Malignant hyperthermia is a genetic condition involving the ryanodine receptor in skeletal muscle sarcoplasmic reticulum. The sarcoplasmic reticulum is the calcium-storing organelle within the skeletal muscle cell. Furthermore, the ryanodine receptor modulates the calcium release from the sarcoplasmic reticulum. When an individual with a genetic predisposition for malignant hyperthermia is exposed to a trigger such a volatile inhalation anesthetics or depolarizing muscle relaxants, a massive intracellular release of calcium into the skeletal muscle occurs. This causes severe muscle contractions. As the skeletal muscle attempts to sequester the excess calcium it must use a tremendous amount of adenosine triphosphate (ATP) which generates a great deal of heat which is the classic sign of the disease. From these processes the cell is so damaged that it breaks down leaking potassium, myoglobulin, creatine, phosphate and creatine kinase into the blood stream which in turn creates myriad of negative consequences.

Which non-absorbable suture material has excellent knot security, virtually no tissue reactivity, but poor handling? A. Nylon B. Polypropylene C. Stainless steel wire D. Braided polyester E. Polybutester

Correct answer: (C) Stainless steel wire. Explanation: Stainless steel wire has almost no tissue reactivity, excellent knot security, handles poorly, and is primarily used in orthopedic procedures, hernia repair, abdominal wound closure, and tendon and ligament repair. Nylon has low tissue reactivity, fair knot security, good handling, and is used in ligament and skin. Braided polyester has moderate tissue reactivity, poor knot security, good handling, and is used in ligament and skin. Polypropylene has minimal tissue reactivity, good knot security, good handling, and is used in skin. Polybutester has low tissue reactivity, good knot security, good handling, and is used in skin.

What are the most commonly encountered coalitions in the foot and ankle? A. Calcaneocuboid B. Cubocuneform C. Talonavicular and talocalcalneal D. Calcaneonavicular and talocalcaneal E. Talonavicular and calcaneocuboid

Correct answer: (D) Calcaneonavicular and talocalcaneal. Explanation: The most commonly encountered coalitions are calcaneonavicular and talocalcaneal. The talonavivular, calcaneocuboid, intercuneiform, naviculocuneiform, cubonavicular, and cubocuneiform are rarely identified. Ninety percent of tarsal coalitions involve the talocalcaneal and calcaneonavicular joints.

This is the suture that elicits the minimal acute inflammatory reaction: A. Polybutester B. Nylon C. Stainless steel D. Surgical gut E. Silk

Correct answer: (C) Stainless steel. Explanation: Stainless steel provides the greatest strength and knot security of all available nonabsorbable suture materials. However, its poor handling characteristics may result in kinking, fatigue, fracture, or deformation at points of stress. It elicits a minimal acute inflammatory reaction and is not absorbed. The ends of the wire suture must be handled carefully to avoid puncture of gloves or tissue. Special instrumentation should be used to cut the wire suture. Twisted multistrand steel wire sutures (Flexon) are more flexible and are easier to handle and tie. However, they may be associated with a higher incidence of infection. The Roto Grip needle minimizes twisting and kinking and allows the needle and suture to swivel 360 degrees on one another. In its monofilament form, nylon is superior to polypropylene in strength but inferior in elasticity. Tissue reaction is greater than with polypropylene. Nylon undergoes slow absorption by hydrolysis and loses tensile strength over a long period. Like polypropylene, nylon is not recommended when permanent holding power is desired or required. In addition, it possesses the greatest proclivity for knot slippage. Its handling characteristics and knot-tying properties are otherwise similar to those of polypropylene. Multifilament (braided) nylon offers no clinically significant advantages over the monofilament form. Monofilament nylon is an excellent suture material for skin closure, second only to polypropylene sutures when a nonabsorbable suture is desired. Polybutester is the newest monofilament synthetic, nonabsorbable suture currently available. It is not subject to break-downs and degradation as are nylon and silk. Tissue reaction to this suture is minimal and probably resembles that of polypropylene. Limited data suggest that its properties of stretching and memory may surpass those of polypropylene. Polybutester has excellent handling properties and tensile strength. Its indications for use are similar to those for nylon and polypropylene in extremity surgery. Surgical gut, interwoven collagen fibers that are formed into a single strand. The collagen in these sutures is a protein that elicits a rather severe antigenic response once implanted. Silk is suture with a high capillarity that should not be used in high bacterial load contaminated area.

A 35 year old female patient presents to clinic after slipping on the ice the day before. She states that she was outside walking her dog and slipped and her foot turned in. She went back inside and iced and elevated the area and took NSAID's. The next day she woke up in severe pain with ecchymosis around the 5th metatarsal. X-rays completed at the office today showed a non displaced extra-articular fracture of the 5th metatarsal base at the metaphyseal/diaphyseal junction. Which of the following classifications are correct based on the radiological description? A. Jahss Type 2 B. Stewart Type 2 C. Stewart Type 1 D. Jahss Type 1

Correct answer: (C) Stewart Type 1. Explanation: The classification system that we are looking for based on the radiological description is the Stewart classification of 5th metatarsal base fractures. Stewart type 1 is the Jones fracture which is located at the metaphyseal diaphyseal junction and is therefore extra- articular. Blood supply to this area of bone tends to be poor and frequently this type of injury pattern goes onto non-union. Stewart type 2 is an intra-articular fracture at the base of the 5th metatarsal. Stewart type 3 is extra-articular fracture of the 5th metatarsals styloid process (usually from avulsion of peroneus brevis). The Jahss classification is for dislocations of the 1st MTPJ. Jahss type 1 is dorsal dislocation of the proximal phalanx with intact intersesmoidal ligament and Jahss type 2 is dorsal dislocation with rupture of intersesmoidal ligament, both of which are incorrect for this question.

This congenital deformity of the foot and ankle has 80% of the cases of this deformity are idiopathic and flexible and 20% are associated with additional chromosomal or genetic anomalies, low amniotic fluid, and a breech position. Which of the following is the deformity described? A. Calcaneus valgus B. Pes planus valgus C. Talipes Equinovarus D. Metadductus

Correct answer: (C) Talipes Equinovarus. Explanation: The pathogenesis of the congenital Talipes Equinovarus can be associated with structural abnormalities or chromosomal abnormalities in 20 percent of the cases, 80% are idiopathic flexible and positional. This can also be caused by intrinsic or extrinsic factors to the fetus. This can be due to uterine fibroids, amniotic bands, breech position, too low amniotic fluid. (See picture below)

What coalition does the Harris-Beath view best demonstrate? A. talonavicular coalition B. calcaneonavicular coalition C. talocalcaneal coalition D. calcaneocuboid coalition

Correct answer: (C) Talocalcaneal coalition. Explanation: A talocalcaneal coalition is best seen with a Harris-Beath (calcaneal axial) view. A middle facet talocalcaneal coalition of the subtalar joint will show an obliterated joint space if the coalition if osseous. If the coalition is fibrocatilaginous, the joint space is narrowed with subchondral sclerosis. The middle subtalar joint appears obliquely oriented. A calcaneonavicular bar is best seen on a medial oblique view. A calcaneocuboid and talonavicular coalition can be seen on a medial oblique and lateral views.

It has been established that the once perceived inadequate blood supply to the talus is actually much more extensive. With this being the case, then why is the incidence of avascular necrosis of the talus so high after a talar fracture? A. Although extensive, the vessels of the talus are more susceptible to arteriosclerosis which, in turn, decreases the blood flow. B. In response to the trauma of the talar fracture, the supply vessels undergo a prolonged state of vasoconstriction. C. The rich vascular supply is unable to tolerate the high energy marked displacements that occur in a talus fracture. D. The small diameter vessels of the talus are prone to long spans of vasospasm.

Correct answer: (C) The rich vascular supply is unable to tolerate the high energy marked displacements that occur in a talus fracture. Explanation: The blood supply of the talus although plentiful, is at risk with severe injury. Fractures of the talus are most often the result of high energy or severe injury. The talus has a marginal soft tissue envelope. The rich vascular supply is unable to tolerate the high energy marked displacements that occur in a talus fracture. In contrast, large areas of muscle may remain attached to the shaft in a markedly displaced femoral fracture, whereas in the talus, the extraosseus nutrient circulation is interrupted and there are no broad areas of muscular attachment.

Which of the following best defines the term simple syndactyly? A. There are no nerve or vessel abnormalities. B. The syndactyly is on one foot only. C. There is no bony fusion in the syndactylized pair of toes. D. Only the fourth and fifth toes are involved.

Correct answer: (C) There is no bony fusion in the syndactylized pair of toes. Explanation: In simple syndactyly the bony elements in the adjacent toes remain separate and unfused. Syndactyly results from the failure of the apical ectodermal ridge to stimulate the necessary cell necrosis that leads to the formation of interdigital clefts in the development of the limb bud. It can involve failure of soft tissue separation of the digits, leading to webbing between the toes (simple syndactyly), or failure of separation of the bony and soft tissue elements (complex syndactyly).

A patient in her fourth week of treatment for osteomyelitis of the first metatarsal, using clindamycin 300mg QID, develops unrelenting diarrhea and stomach cramps. She is pregnant. Her albumin is 2.5g/dl. Treatment should be: A. Discontinue clindamycin and observe B. Metronidazole 500mg po TID x 10 days C. Vancomycin 125mg po QID x 10 days D. Fidaxomicin 200mg po BID x 10 days

Correct answer: (C) Vancomycin 125mg po QID x 10 days. Explanation: The clindamycin has caused a C. difficile infection. The antibiotic should be discontinued. Based upon the CDI severity scoring system, the unrelenting diarrhea and stomach cramps in association with an albumin less than 3, classifies this as severe disease, necessitating vancomycin as a first line drug, as opposed to the usual first line drug being metronidazole. The cost difference is quite remarkable; a 10-day course of metronidazole is $22.00. Vancomycin is $680.00. (A) The disease is classified as severe under the CDI severity scoring system. Merely discontinuing the drug and observing will not resolve the disease, and in fact, will cause it to worsen. (B) Metronidazole is usually the first line drug. In this scenario, her disease is classified as severe, as opposed to mild/moderate. Thus, vancomycin is the first choice. Additionally, metronidazole must be avoided in pregnancy and during breast feeding, as the active metabolites are detected in breast milk. During pregnancy, metronidazole can cause facial anomalies following exposure. (D) The FDA granted approval for fidaxomicin in 2011. However, there is limited data to support its superiority over vancomycin. Additionally, a 10-day course is approximately $2800.00.

A patient steps on a nail and transects the lateral plantar nerve component that innervates only the quadrantus plantar muscle. What is the most likely toe deformity that would be seen in midstance to full-forefoot loading in gait? A. Extension deformity of the lesser toes B. Flexion deformities of the toes C. Varus rotation of the lesser toes D. Valgus rotation of the lesser toes

Correct answer: (C) Varus rotation of the lesser toes. Explanation: The quadrants plantae muscle functions to negate/balance the FDL tendon's medially oblique pull to the lesser digits to create a more rectus/sagittal plane motion. If the balancing muscle was lost through the deinnervation, a slight varus rotation of the lesser toes would occur in stance. The long flexor tendon would have an unregulated proximal medial pulling of the digits which as one moved lateral the medial nature of the pull would be greater. In other words that varus deformity of the fifth toe would have a greater degrees of varus deformity than the fourth and so on.

A patient presents to clinic with complaints of hyperkeratosis of the third, fourth, and fifth toes, on both feet with a painful callus on the nail groove in the lateral nail groove of the fifth, bilaterally. The radiographs taken prior to the exam reveal varus rotation of the phalanges, with medial deviation of the distal phalanx on the middle phalanx. What is the diagnosis? A. Mallet toe B. Hammer toes C. Varus toes D. Claw toe

Correct answer: (C) Varus toes. Explanation: Also known as varus toes, most often congenital, they often involve the third, fourth and fifth toes. The deformity represents flexion and varus rotation of the distal phalanx at the interphalangeal joint. Some adduction is likewise present, and in more severe cases the proximal interphalangeal joint is also involved. Varus toes are often evident with flexion, varus, and adduction of the distal aspect of the toe. They are usually bilateral. Symptoms are usually related to hyperkeratoses that develop on the lateral side of the distal or proximal interphalangeal joints. Painful Lister's corn in the lateral nail groove of the fifth toe may develop as a result of transferring weight off the side of the side of the underlapping toe. The radiographs show varus rotation of the phalanges, with medial deviation of the middle phalanx on the proximal phalanx or medial deviation of the distal phalanx on the middle phalanx. Goals of the treatment are to achieve a rectus digit, free of painful clavi, flexible enough to absorb shock and to allow transferred of the weight toe off.

A healthcare provider learns from a mutual acquaintance that a former patient who was treated for opioid addiction 5 years ago has completed medical school and entered a residency program. They last saw the patient 4 years ago, at which time the patient was compliant with treatment. What is the best course of action? Breach confidentiality and report healthcare provider to medical board. Contact healthcare provider assistance program in former patient's state and explain history of drug abuse. Without evidence that this former patient is impaired, do not breach confidentiality. Contact former patient and request evidence of compliance with treatment. Inform relevant authority about former patient's potential impairment due to legal responsibility.

Correct answer: (C) Without evidence that this former patient is impaired, do not breach confidentiality. Explanation: Although impairment may result from the use of psychoactive agents or illness, the presence of a disorder does not necessarily imply impairment. Unless a healthcare provider seems to be impaired now, there is no legal or ethical duty to report the healthcare provider based on history. Unless there is evidence of current impairment or patient danger, a patient who happens to be a healthcare provider has the same rights to confidentiality under the provider- patient relationship. However, when a healthcare provider identifies an impaired healthcare provider, there is a clear ethical duty to protect the patients of the impaired healthcare provider by reporting the situation to an appropriate authority, although the legal responsibility to report an impaired healthcare provider varies from state to state.

Surgical prophylactic antibiotic use is indicated when an elective surgery on an otherwise healthy patient is expected to last longer than? A. 30 minutes B. 60 minutes C. 90 minutes D. 120 minutes

Correct answer: (D) 120 minutes. Explanation: Any surgery lasting more than 2 hours is an indication for the use of prophylactic antibiotics. This extended operative time leaves the patient more vulnerable to exogenous contaminants from the operating room environment, operating room personnel, and the patient's own flora. Other indications for the use of prophylactic antibiotics include operating on an immune-compromised patient, trauma surgery, implant surgery, or when an infection could cause such a devastating risk that any potential risk of antibiotic use is overshadowed.

What is the appropriate metatarsal stump parabola following a transmetatarsal amputation? A. 1 > 2 > 3 > 4 > 5 B. 2 > 1 > 3 > 4 > 5 C. 1 = 2 > 3 > 4 > 5 D. 2 > 1 = 3 > 4 > 5

Correct answer: (D) 2 > 1 = 3 > 4 > 5. Explanation: It is imperative to maintain a normal metatarsal parabola when performing a transmetatarsal amputation. 2 > 1 = 3 > 4 > 5. In other words the second metatarsal is the longest with the first and third shower than the second but equal in length. Finally the fourth and fifth go down like stair steps. If this is not achieved that an uphill battle will ensue of continued skin breakdown from inadequate offloading. (A) The second metatarsal is to be the longest metatarsal. (B) The first and third metatarsals are to be of equal length. (C) The second metatarsal is to be the longest metatarsal and the first and third metatarsals are to be of the same length.

When measuring the metatarsus adductus angle in older children using the bisection of the second metatarsal and the second cuneiform, above what angle is considered abnormal? A. 10 degrees B. 15 degrees C. 20 degrees D. 25 degrees

Correct answer: (D) 25 degrees. Explanation: A metatarsus adductus angle greater than 25 degrees is considered abnormal when measuring the bisection of the second metatarsal and second cuneiform. Engel et al originally described this method of measuring metatarsus adductus. However, this method of measuring the metatarsus adductus angle requires the ossification of the second cuneiform, which is why this method is only used in older children. (A) A metatarsus adductus angle greater than 10 degrees is not considered abnormal when measuring the bisection of the second metatarsal and second cuneiform. However, this is considered the normal angle formed between the bisection of the 2nd metatarsal and the long axis of the calcaneus when measuring the metatarsus adductus angle using this method. Meaning any angle greater than 10 degrees would signify metatarsus adductus. This method is typically used in infants as few other bones are visible radiographically. (B) A metatarsus adductus angle greater than 15 degrees is not considered abnormal when measuring the bisection of the second metatarsal and second cuneiform. (C) A metatarsus adductus angle greater than 20 degrees is not considered abnormal when measuring the bisection of the second metatarsal and second cuneiform.

Screws are available in three basic sets: minifragment, small fragment set, and large fragment set. What outer diameter is found in the small fragment set? A. 2.7 and 3.5 mm B. 1.5 and 2.0 and 2.7 mm C. 2.0 and 3.5 mm D. 3.5 and 4.0 mm E. 4.5 and 6.5 mm

Correct answer: (D) 3.5 and 4.0 mm. Explanation: AO/ASIF screws are available in three basic sets (other sets are available): minifragment, small fragment, and basic or large fragment. The minifragment set contains screws with an outer diameter of 1.5, 2.0, and 2.7 mm. The small fragment set contains screws with an outer diameter of 3.5 and 4.0 mm. The large fragment set contains both 4.5- and 6.5-mm screws. All AO/ASIF screw sets come complete with a full complement of equipment to insert each type of screw. Screws are available in stainless steel or titanium.

What is the toxic dose of lidocaine without epinephrine for a 92 kg male? A. 300 mg B. 368 mg C. 398 mg D. 414 mg

Correct answer: (D) 414 mg. Explanation: Complications from local anesthetic typically occur from toxicity or allergy. Allergy is relatively rare. Toxicity is seen when an excessive dose based on mg/kg of body weight is given, accidental intravascular injection or rapid absorption from a highly vascular tissue. The traditional memorized toxic dose for lidocaine without epinephrine is 300 mg. A critical caveat to this number is that it is based on an individual weighing 70 kg. The more accurate way of thinking about the toxic dose of lidocaine is to calculate toxic does by 4.5 mg/kg. So in the example above a 90 kg male would have a toxic dose of 414 mg. 92 x 4.5 = 414.

A patient has been on 5 mg taper dose of glucocorticoid regimen, did finish her therapy 5 days ago, presents with a paronychia, and you elect to perform a partial nail avulsion. Please pick what applies. A. B. C. D. Give 5 mg of prednisone the day of the nail avulsion. Give 24 mg prednisone taper over 4 days after the nail avulsion. For elective surgery, 25 mg IV hydrocortisone or equivalent in the presence of hyponatremia, hypertension, hyperkalemia. 5 mg or less of prednisone for less than 3 weeks should not result in clinically significant adrenal suppression.

Correct answer: (D) 5 mg or less of prednisone for less than 3 weeks should not result in clinically significant adrenal suppression. Explanation: The hypothalamic-pituitary-adrenal axis (HPA) is central to a patient's ability to generate a surgical stress response. A defect in this system can have dramatic consequences in the perioperative period. Adrenal insufficiency is to be avoided. But a daily therapy with 5 mg or less of prednisone and any dose given for less than 3 weeks should not result in clinically significant adrenal suppression. (A) (B) The dose and duration of exogenous corticosteroids required to produce clinically significant tertiary adrenal insufficiency is highly variable. But general principles can be outlined, a daily therapy with 5 mg or less of prednisone should not result in adrenal suppression. (C) Same as choice (A) and (B), there is no sign of hyponatremia, hypertension, or hyperkalemia, in this case.

What is the minimal time frame that is recommended to wait to perform digital amputation after a distal revascularization procedure? A. Simultaneous B. 24 Hours C. 48 Hours D. 72 Hours

Correct answer: (D) 72 Hours. Explanation: In patients with digital gangrene and an ischemia foot a revascularization is required before digital amputation can be performed. After the procedure, it is best to wait at least 72 hours before digital amputation to allow revascularization of ischemic tissues. (A) Simultaneous digital amputation at the time of the revascularization would likely lead to excessive tissue removal as the tissues in question have not yet had an opportunity to respond to the re-established blood flow. (B) Digital amputation performed 24 hours after revascularization would likely lead to excessive tissue removal as the tissues in question have not yet had an opportunity to respond to the reestablished blood flow. (C) Digital amputation performed 48 hours after revascularization would likely lead to excessive tissue removal as the tissues in question have not yet had an opportunity to respond to the reestablished blood flow.

What is the percentage of protection provided by a lead apron by low level kVp radiographs (below 70kVp)? A. 100% B. 30% C. 75% D. 95%

Correct answer: (D) 95%. Explanation: The 3 main ways to protect oneself or a patient from radiation are time, distance and shielding. The less time one spends in radiation area, the less the exposure. Radiation distance decreases inversely as the distance squared. Any material interposed between the source of radiation and the individual receiving radiation, can protect that person from radiation. A lead apron (0.5mm Pb-equivlant) provides 95% protection from low level xrays (less than 70kVp).

What is the main purpose of the SACH heel modification to shoes? A. Increase pronatory moments to compensate for soft-tissue equinus B. Compensate for limb length discrepancy C. Decrease tension on the Achilles tendon D. Act as an extrinsic shock absorber

Correct answer: (D) Act as an extrinsic shock absorber. Explanation: It is placed in the posterior midsole of the shoe and usually made of a high-density foam rubber wedge.

Which suture has the greatest soft tissue reaction in the body? A. Nylon B. Silk C. Polyes D. Catgut

Correct answer: (D) Catgut. Explanation: The degree and severity of soft tissue response to absorbable sutures varies among the materials but is overall greater than that elicited by nonabsorbable sutures. Multifilament materials and natural materials will elicit more of a response than monofilament and synthetic materials. Plain catgut elicits the most severe inflammatory response among all suture materials followed by chromic catgut then natural fiber materials such as silk.

Which of the following is not an appropriate form of surgical hemostasis? A. Electrocautery B. Manual pressure C. Pneumatic tourniquet D. All of the above are possible and appropriate forms of hemostasis.

Correct answer: (D) All of the above are possible and appropriate forms of hemostasis. Explanation: Maintaining hemostasis is an important principle of surgical intervention. Not only does inadequate hemostasis decreased visibility in the surgical field, it also increases the risk of post-operative complications. There are many different methods to maintain surgical hemostasis, and often multiple methods are used during the same case. A pneumatic tourniquet is frequently used during foot and ankle surgery. A tourniquet can be placed at the level of the ankle, calf, or thigh. Frequent pressures are range from 250 mmHg at the level of the ankle to 350 mmHg at the thigh. Electrocautery is used frequently is cauterize bleeding vessels in a precise fashion. Larger vessels that are not able to be cauterized due to their size are usually ligated by hand. A simple, but often very effective, method of hemostasis is holding manual pressure against bleeding tissues. Thus all of the methods listed above are appropriate.

All of the following are mechanical methods of hemostasis except? A. Tourniquet application B. Acrylates C. Bone wax D. Aluminum chloride hexahydrate

Correct answer: (D) Aluminum chloride hexahydrate. Explanation: Aluminum chloride hexahydrate is an example of a chemical method of hemostasis. Examples of mechanical methods of hemostasis include direct pressure, vessel ligation, bone wax, tourniquet application, compressive dressings, and the use of acrylates. Mechanical methods of hemostasis work by tamponade and have no hemostatic property of their own. Bone wax works by providing a mechanical tamponade. Acrylates adhere to the tissue and provide a mechanical blockage of hemorrhage. The use of a tourniquet is the most common method of mechanical hemostasis among surgeons of the lower extremity. The tourniquet can be applied to the thigh, calf or ankle depending on the duration of the surgery, the location of the surgical procedure, and the concern of vessel damage from the applied cuff.

What radiographic sign is seen on radiographs of patients with an osseous calcaneonavicular coalition? A. Zebra Nose Sign B. Giraffe Neck Sign C. Bucket Handle Sign D. Anteater Nose Sign

Correct answer: (D) Anteater Nose Sign. Explanation: The anteater nose sign is seen on a lateral x-rays of patients with osseous calcaneonavicular coalitions as a tubular elongation of the anterior superior aspect of the calcaneus projecting toward the navicular. While in normal feet the anterior superior calcaneus may lean towards the navicular it will lack the tubular projection seen with the anteater sign. However, it is an oblique view of the foot that best visualizes the calcaneonavicular coalition. With this view, the space between the calcaneus and the navicular will be completely obliterated in osseous coalitions.

Which of the following situations does not require preoperative antibiotic prophylaxis? A. Use of a prosthetic implant B. Surgical procedure > 120 min C. Immunocompromised patient D. Arthroscopic procedure

Correct answer: (D) Arthroscopic procedure. Explanation: Studies have shown that the use of preoperative antibiotic prophylaxis is not indicated in arthroscopic surgical procedures. Prophylactic IV antibiotics are not indicated in clean elective cases where no prosthetic material is being placed unless the patient meets certain high-risk criteria. Any patient undergoing surgery that is expected to last more than 2 hours should receive prophylaxis. Immunocompromised patients, those with uncontrolled diabetes mellitus, receiving long-term immunosuppressive drugs or those with genetic or acquired immune defects, should always be prophylaxed. Prophylaxis is consistently recommended in all cases classified as clean-contaminated, contaminated and dirty

Which of the following statements below is a potential complication of performing a lateral release along with an Austin bunionectomy? A. medial subluxation of the sesamoids B. degenerative arthritis of the sesamoid apparatus C. decreased propulsion power during toe-off of gait D. avascular necrosis of the capital fragment

Correct answer: (D) Avascular necrosis of the capital fragment. Explanation: As the nutrient artery of the first metatarsal enters the proximal third of the bone the osteotomy separates the capital fragment from the medullary flow. The small vessels within the capsular reflections about the metatarsal head often are severed during the dissection needed for a lateral release. This sets up a situation of significant vascular embarrassment and can lead to avascular necrosis. There is no evidence to suggest that subluxation of the sesamoids, arthritis or altered mechanics with the performance of a lateral release.

During a work up of a 25 year old female with the complaint of mild bunion discomfort, an irregularity in the medial cuneiform is noted. The patient does not complain of pain in the area of the medial cuneiform nor is there pain upon palpation to the medial cuneiform. Range of motion of the mid tarsal joint and tarsometatarsal joint is not painful. No swelling, warmth or color changes are noted around the area. She denies trauma or previous surgery. An MRI of the affected foot is ordered to further assess the irregularity. Pictured below is a sagittal image from the MRI study. What is the most likely diagnosis for this irregularity in the medial cuneiform? A. Stress fracture of the medial cuneiform B. Avascular necrosis of the medial cuneiform C. Atrophic nonunion of a cuneiform osteotomy D. Bipartite medial cuneiform

Correct answer: (D) Bipartite medial cuneiform. Explanation: Key points of this question lie in the fact that there are no symptoms related to the irregularity nor is there history of trauma or surgery. This eliminates the possibility of an atrophic nonunion of the medial cuneiform. Likewise if the irregularity was caused by a stress fracture pain and swelling would be present. Avascular necrosis would also cause pain as the bone started to collapse on itself. This is a classic example of a bipartite medial cuneiform which typically presents as a horizontal segmentation. As can also be seen on the MRI articular cartilage is present on the adjacent sides of each segment.

Which of the following techniques addressing lateral ankle instability utilizes the extensor retinaculum? A. Watson-Jones B. Chrisman-Snook C. Lee procedure D. Brostrom with Gould modification

Correct answer: (D) Brostrom with Gould modification. Explanation: The Brostrom with Gould modification directly repairs the lateral ankle ligaments and reinforces using the extensor retinaculum. (A) Watson-Jones utilizes peroneus brevis tendon to reconstruct the ATFL. (B) Chrisman-Snook reconstructs or reinforces the ATFL and CFL using half of the peroneus brevis tendon. (C) The Lee procedures uses peroneus brevis tendon which is passed from a drill hole in the fibula then sutured on itself and peroneus longus tendon.

The Ponseti technique for clubfoot deformity correction addresses the components in a series of casting. What is the correct sequence of deformity correction? A. Varus, Adductus, Cavus, Equinus B. Cavus, Adductus, Equinus, Varus C. Equinus, Cavus, Adductus, Varus D. Cavus, Adductus, Varus, Equinus

Correct answer: (D) Cavus, Adductus, Varus, Equinus. Explanation: A pneumonic to remember the correct sequence is "CAVE" for cavus, adductus, varus, equinus. The Ponseti method is based on the basis that the talar head is the key landmark to obtain safe reduction of the clubfoot deformity. The serial casting is done with gentle manipulation for gradual correction. The first component is correction of the midfoot cavus by supinating and dorsiflexing the first metatarsal to align the forefoot with the hindfoot. Next is abducting the forefoot while holding counter-resistance to the neck of the talus. Then the rearfoot is manipulated out of varus deformity. A percutaneous Achilles tendon release may be necessary in order to correct the equinus component. Surgical soft tissue release may be necessary if the child's foot is resistant to bracing.

Nuclear medicine scans are obtained on a patient with an infected ulceration with questionable osteomyelitis. Both a technetium and a gallium scan were obtained. The technetium scan was negative, but the gallium scan was positive. What is the diagnosis? A. acute osteomyelitis B. chronic osteomyelitis C. infectious periostitis D. cellulitis

Correct answer: (D) Cellulitis. Explanation: When technetium is positive and gallium is negative, chronic osteomyelitis is present. When both technetium and gallium are positive, the diagnosis is either acute osteomyelitis or a septic arthritis. Lastly, when a technetium scan is negative and a gallium scan is positive, the diagnosis is cellulitis. Technetium is a bone-imaging radionuclide while gallium is an inflammatory-imaging radionuclide. Gallium binds to white blood cells, plasma proteins, transferrin, ferritin, lactoferrin and siderophores and travels to areas of inflammation. Technetium binds to hydroxyapatite crystals and travels to area

Anti epilepsy drugs (AED) are known to be toxic. The following should be checked prior to surgery: A. Renal function test B. Liver function test C. Metabolic panel D. Complete blood count, metabolic panel, and liver function

Correct answer: (D) Complete blood count, metabolic panel, and liver function. Explanation: Epilepsy is a broad term encompassing the various seizure disorders. Seizures maybe partial, focal, or generalized. Patients with epilepsy are known to have an increased risk for operative seizure secondary to the choice of anesthetic or physiologic derangement. Most patients are treated with antiepileptic drugs. Most AED at toxic or even at therapeutic levels may cause liver dysfunction and possibly bone marrow suppression. Liver function tests, a complete blood count, and metabolic panel should be performed before surgery.

Which of the following first metatarsal base osteotomies performed for the treatment of hallux abducto valgus has the highest rate of dorsal malunion? A. Proximal chevron osteotomy B. Closing base wedge osteotomy C. Opening base wedge osteotomy D. Crescentic osteotomy E. Juvara osteotomy

Correct answer: (D) Crescentic osteotomy. Explanation: The crescentic osteotomy has the highest incidence of dorsal malunion, ranging from 9-28% of cases. This procedure, however, produces the least metatarsal shortening of any of the procedures listed.

Which hallux valgus procedure results in the least inherent shortening? A. Wilson B. Closing base wedge C. Scarf D. Crescentic

Correct answer: (D) Crescentic. Explanation: The crescentic bunionectomy provides the least inherent shortening. A well accepted rule is that the saw blade used for the osteotomy accounts for 2 mm of bone loss with each pass. With that being said the crescentic bunionectomy utilizes only one pass of the saw so the maximum shortening is 2 mm. (A) The oblique nature of the Wilson bunionectomy, by nature, shortens as it corrects. For every bit of lateral capital translation, a corresponding degree of shortening occurs. The amount of shortening is based on the obliquity of the osteotomy. On average an average of 11 mm of shortening occurs with bunion correction via the Wilson procedure. (B) Again, by nature the closing wedge osteotomy shortened the first metatarsal significantly. In this instance the amount of shortening is dependent on the size of the wedge removed. (C) The scarf osteotomy doesn't directly rely on shortening to provide the hallux vagus correction. Rather the shortening with this procedure comes from the fact that two transverse osteotomies are employed in performing the scarf procedure. This accounts for a minimal total bone loss of 4 mm as compared to the 2 mm of the crescentic procedure.

Your patient had surgery for a non-healing Jones fracture, for which you repaired with a percutaneous 4.5mm malleolar screw. At 90 POD, there is sclerosis at the fracture site; clinically, there is swelling and palpable pain at the fracture site. A 3-phase technetium bone scan demonstrates no uptake in the third phase. The best course of action is: A. Continue with a short leg NWB cast for six more weeks. B. Ultrasound bone stimulator with a removable walking boot. C. Remove screw, and replace with percutaneous insertion of 6.5mm screw. D. Debridement of fracture site, autogenous bone grafting, plate fixation.

Correct answer: (D) Debridement of fracture site, autogenous bone grafting, plate fixation. Explanation: Sclerosis, edema, pain, and no uptake after only 90 POD are all indicative the fracture site is turning cold and a non-union has occurred. An aggressive approach is indicated. The original screw is removed and the bone debrided to bleeding margins. Autogenous bone graft is probably better than allografting; the graft can be taken from the calcaneus, tibia, or hip. Use of osteoinductive bone substitutes can be added to the graft. The plate is placed laterally on the bone. (A) Prolonged casting alone will not allow this fracture to heal, and in fact will result in cast disease. (B) An imaging modality to determine if a bone stimulator will work is the technetium-99 bone scan. If the scan does not show uptake in the third phase, there is no bony activity, and thus no active bone left to stimulate. (C) Actually, James Nunley, MD, in his book, advocates removing the screw if possible and performing the surgery again, percutaneously, without taking down the fracture site. Being a board exam, they are probably looking for a more aggressive answer.

When examining the following first metatarsal osteotomies which one takes the most advantage of the foot's biomechanics? A. Luldloff B. Reverdin C. Cresentic D. Dorsiflexory Wedge

Correct answer: (D) Dorsiflexory Wedge. Explanation: The dorsiflexory wedge osteotomy of the first metatarsal is the most biomechanically favored osteotomy from the list above. This is explained by the fact that every time the forefoot is loaded, the dorsiflexory wedge osteotomy is compressed. The Ludloff osteotomy is a shaft osteotomy angled from dorsal proximal to plantar distal resulting in separation of the osteotomy when the forefoot is loaded. The Reverdin capital closing base wedge osteotomy is likewise unstable with forefoot loading. The crescentic osteotomy likewise also has no intrinsic stability.

Which one of the following is an advantage of external fixation? A. Nerves and vessels are preserved B. Loosening of pins is necessary C. Less morbidity and mortality D. Ease of patient mobilization

Correct answer: (D) Ease of patient mobilization. Explanation: External fixators are a means of reducing bone fragments to the anatomic position using a frame that is outside the skin. The bone segments are secured using pins or wires that pass through the skin. Advantages of external fixators include the accessibility of the frame for manipulation or adjustment after initial fixation, easy access to the injury site, minimal interference with adjacent joints, and ease of patient mobilization.

Out of the biomechanical conditions listed below, which is a major contributor to the deformity pictured below? A. Anterior equinus B. Rearfoot varus C. Forefoot supinatus D. Equinus

Correct answer: (D) Equinus. Explanation: Pictured above is a Charcot midfoot. Of note on the xray is the pronounced talar declination as well and the nearly parallel calcneal inclination. Additionally, a significant midfoot collapse through the tarsometatarsal joints is observed. This is a tremendously strong joint complex which either a significant amount of force is needed to disrupt or a weakening of supportive ligamentous structures predispose the joints complex to such a fate. Equinus has been cited as uniformly being present in cases of midfoot charcot neuroarthropathy and it is significant enough to cause the calcaneal inclination angle.

Which of the following is NOT an appropriate choice for local anesthesia for elective bunion repair utilizing a distal first metatarsal osteotomy? A. Mayo block B. Popliteal and Saphenous nerve block C. Ankle block D. Hallux digital block

Correct answer: (D) Hallux digital block. Explanation: In order for local anesthesia to be effective, it must act upon nerves proximal to the site where anesthesia is desired. Of the listed blocks, all act proximal to the intended surgical site except for the hallux digital block. A Mayo block is a commonly used choice for bunion surgery that utilizes a distal osteotomy. Both an ankle block or a popliteal and saphenous block are proximal to the intended surgical site and would be appropriate choices. A digital hallux block would not properly anesthetize the site of the distal first metatarsal osteotomy and would therefore not be an appropriate choice for the proposed surgery.

What radiological view is demonstrated below and what condition is it showing? A. Long leg calcaneal axial; anterior facet coalition of the talocalcaneal joint joint B. Harris-Beath view; anterior facet coalition of the subtalar joint C. Calcaneal axial view; posterior facet coalition of the subtalar joint D. Harris-Beath view; middle facet coalition of the talocalcaneal joint joint

Correct answer: (D) Harris-Beath view; middle facet coalition of the talocalcaneal joint joint. Explanation: The question above has two aspects that must be addressed. First is the radiological view in the clinical image. The image above is an example of the Harris-Beath view also known as the calcaneal axial view. It is not a long leg calcaneal axial. The other portion of the question is identifying the clinical condition. The clinical condition is a middle facet coalition of the subtalar joint (talocalcaneal joint). The subtalar joint is synonymous with the talocalcaneal joint. The anterior facet cannot be visualized in this view and the posterior facet is clearly visualized and is free of pathology. So the correct answer is a Harris-Beath view demonstrating a middle facet coalition of the talocalcaneal joint.

A 38 year old construction worker suffers an ankle injury while on the job. He is evaluated in the emergency department about 1 hour after his injury. Physical exam shows a medially displaced foot with an approximately 4cm open wound on the lateral side of the ankle with grass and dirt present. There is a roughly 2cm segment of bone protruding through the skin. Neurovascular exam of the foot is normal. X-rays show displaced and comminuted fractures of the medial and lateral malleolus. What is the most important next step in the management of this patient? A. Take deep cultures of the open wound B. Attempted reduction and splinting of the fracture C. Thorough irrigation of the open wound at bedside in the ED D. Initiation of broad spectrum antibiotics E. Emergent irrigation and debridement in the operating room

Correct answer: (D) Initiation of broad spectrum antibiotics. Explanation: The most important first step in the management of this open fracture is the initiation of IV antibiotic therapy. Reports in literature have consistently shown that time to surgical debridement is less crucial when compared to early initiation of antibiotics when it comes to preventing infection in open fractures. Deep cultures of a clearly contaminated wound would not be very beneficial in the treatment of this patient because the culture results would almost certainly indicate a multimicrobial infection that would be treated with broad spectrum antibiotics. In the large majority of instances, reduction and splinting of the extremity should be attempted in the emergency department, but it would not take precedence over the initiation of IV antibiotics. Thorough irrigation of the open wound at bedside in the emergency department to remove the frank contamination should only be undertaken after verification of intact circulation in the foot and initiation of IV antibiotics. Reports in the literature generally support urgent irrigation and debridement in the operating room, but do not classify a stable open fracture as emergent.

A 55 year old male complains of persistent, intense pain sub 2nd metatarsal heads on both feet. Patient reports failure of shoe changes, padding and custom orthotics by another podiatrist. Below is a clinical radiograph from this encounter. What surgical procedure would re-establish weight bearing of the first ray? A. Phalangeal-based hemi-implant arthroplasty B. Metatarsal-based hemi-implant arthroplasty C. Total great toe joint arthroplasty D. Interpositional arthrodesis first metatarsophalngeal joint

Correct answer: (D) Interpositional arthrodesis first metatarsophalngeal joint. Explanation: The first metatarsophalangeal joint interpositional arthrodesis would be the most appropriate procedure to re-establish the weight-bearing potential of the first ray. The Keller- type procedure rendered the first metetarsophalangeal joint non-functional. Reestablishing the length of the ray will result in fuantional ray that will remove much of the excess workload from the second ray. (A) A phalangeal-based hemi-implant would have no legitimate effect of the load bearing properties of the first ray. (B) A metatarsal-based hemi-implant would also not accomplish much in this scenario interns of reestablishing the load bearing property of the first ray. (C) A total great toe implant arthroplasty would restore some of the length of the digit, but would not help restore the load bearing capacity of the first ray.

A patient will undergo a podiatric surgical procedure in 2 days, and the procedure will require an immobilization of more than an hour intraoperative and more postoperatively. The patient is currently on warfarin, with an internationalized normalized ratio of 3.0. Which of the following would be the most appropriate rapid form of reversal of the anticoagulation? A. Fresh frozen plasma B. 5-mg oral vitamin K tablets C. Subcutaneous delivery of vitamin K D. Intravenous vitamin K

Correct answer: (D) Intravenous vitamin K. Explanation: For less urgent situations like this case, in a surgery to be performed within 24 to 96 hours, vitamin K is the treatment of choice. Intravenous vitamin K will provide a more rapid reversal than any other routes of administration. A retrospective study found that 1 mg of IV vitamin K reversed anticoagulation to an INR <1.4 in a median time of 27 hours. The most notable adverse events to IV vitamin K were dyspnea and chest tightness during infusion, which occurred in approximately 2% of patients. In emergent surgical procedure, reversal of anticoagulation is typically achieved with the administration of fresh frozen plasma or vitamin k. FFP has an immediate onset of action and its effects are short-lived. And it does not cause resistance to anticoagulation with warfarin postoperatively. The INR should be monitored after initial administration of FFP and every few hours subsequently to determine if additional treatment is needed to achieve the INR goal. Vitamin K given orally is the most common method of administration and does not provide the same potential side effects as the intravenous vitamin K. Dosages of 1.0 mg to 2.5 mg of oral vitamin K will correct supratherapeutic INR >4.5 to a therapeutic range within 24 to 48 hours. Oral vitamin K is only available in 5-mmg tablets in the United States; these tablets are scored allowing for easy administration of 2.5 mg. Lower doses can be administered by having the patient drink the intravenous preparation. Subcutaneous delivery of vitamin K used to be common place; it has been shown that absorption by this route is not predictable and thus no longer recommended.

A 63-year-old patient presents for outpatient surgery with history of cardiac disease. The patient had a preoperative EKG and was cleared for surgery by his cardiologist and the anesthesiologist. Which of the following inhaled anesthetics would be safest for a patient with cardiac history? A. Halothane B. Nitrous Oxide C. Propofol D. Isoflurane

Correct answer: (D) Isoflurane. Explanation: Isoflurane is the most cardio protective out of all the inhaled anesthetics. Propofol commonly used as an induction agent in general anesthesia, however it is not an inhaled anesthetic. Nitrous Oxide doesn't produce adequate anesthesia when used solely, however can cause cardiac depression. Halothane is more appropriate for pediatric patients and is not generally used in adults.

Which of the following orthotic modifications would help alleviate symptoms caused by over pronation? A. Blake Everted Pour B. Lateral Flange C. First ray cut-out D. Kirby Medial Skive

Correct answer: (D) Kirby Medial Skive. Explanation: Inverted heel cup orthoses modifications (like the Kirby medial Skive and the Blake Inverted Pour) reduce rearfoot pronation and relieve pronation-related symptoms by increasing the orthotic reactive forces on the medial aspect of the plantar heel so that increased STJ supination moments would result and more effectively treat difficult pathologies such as pediatric flatfoot deformity, posterior tibial dysfunction, and sinus tarsi syndrome.

Which radiographic "projection" listed below is most appropriate to order when trying to evaluate for a calcaneo-navicular coalition? A. Lateral foot B. Medial Oblique foot C. Harris & Beath D. Lateral Oblique foot

Correct answer: (D) Lateral Oblique foot. Explanation: When speaking in terms of "projections", the best answer is a lateral oblique foot projection as this highlights the lateral column of the foot. Projection is a term used to describe the positioning technique and refers to the anatomic area that is first exposed to the x-ray beam or closest to the tube. Views, on the other hand, are named based on the body part closest to the film and are the opposite of projections. Therefore, a medial oblique view would be most appropriate for assessing a calcaneo-navicular coalition as the medial side of the foot is closest to the film.

What is an absolute contraindication for the procedure pictured below? A. High intermetatarsal angle B. Hypermobility of the first ray C. High distal articular set angle D. Long first metatarsal

Correct answer: (D) Long first metatarsal. Explanation: Pictured is an opening wedge osteotomy. The absolute indication is the already long first metatarsal. Unlike most first metatarsal osteotomies which all have some degree of shortening involved, in the opening base wedge osteotomy there is lengthening of the first metatatarsal. This is ideal in the case of a shorter metatarsal and with careful attention can be used with a first metatarsal of normal length. The intermetatarsal angle really has no bearing as it can correct a wide range of IM angles. Hypermobility is not a contraindication. In some surgeon's hands a Lapidus procedure would be indicated in the presence of hypermobility, however with the lengthening effect achieved with the opening wedge osteotomy this will in part tighten the plantar aponeurosis enhancing the windlass mechanism. The high distal articular set angle would have minimal bearing on procedure choice for the osteotomy of the first metatarsal.

Using lag screw without neutralization plate for long bone fracture fixation will lead to the following: A. Long term stability of the fracture fragments B. Acceleration of the healing process C. Control of bending forces D. Mechanical failure

Correct answer: (D) Mechanical failure. Explanation: The lag effect using screw is adequate as the only fixation for some metaphyseal or epiphyseal fractures. Particularly in cortical bone, lag screws must be protected with a plate or plate equivalent, such as external fixator. The protective device is referred to as neutralization device/plate because it controls the bending, translational, and torsional forces. Use of lag screws without a neutralization plate for long bone diaphyseal fixation invites mechanical failure.

Which finding is consistent with the diagnosis of clubfoot deformity in a child? A. Lateral subluxation of the navicular B. Rearfoot valgus C. Forefoot abduction D. Medial deviation of the talar neck

Correct answer: (D) Medial deviation of the talar neck. Explanation: Clubfoot deformity is characterized by the medial and plantar deviation of the anterior aspect of the talus, the talar neck. Additional variances may include a smaller talar bone and delayed ossification. (A) The deformity in clubfoot deformity is medial subluxation of the navicular due to the medial deviation of the talar neck. This medial protrusion may be noticeable on clinical examination. Radiographically, in severe cases, the lateral aspect of the anterior talus may not articulate with the navicular. (B) The rearfoot in clubfoot deformity is in a varus and equinus position. The severe equinus often causes a clinical appearance of a posterior fold just superior to the posterior calcaneus. (C) The forefoot is in adduction, not abduction. The relationship of the forefoot to rearfoot causes a classic clinical appearance of a C shaped curvature on the lateral aspect of the foot and a medial fold rather than rectus medial soft tissue alignment.

A patient presents complaining of numbness of the second and third digits of the left foot. There is no complaint of numbness of the hallux or fourth and fifth digits. A previous crossover toe repair with an extensor digitorum longus transfer under the intermetatarsal ligament was performed on the same foot six months prior. The most likely diagnosis is iatrogenic severance of which nerve? A. Saphenous B. Deep peroneal C. Lateral dorsal cutaneous D. Medial dorsal cutaneous

Correct answer: (D) Medial dorsal cutaneous. Explanation: The key to this question is recognition of surgical anatomy. The medial dorsal cutaneous supplies sensory innervation to the hallux, second digit and medial aspect of the third digit with the exception of the adjacent surfaces of the hallux and second toe. The surgical procedure described above would require dissection that could injure the medial dorsal cutaneous nerve. The lateral cutaneous nerve supplies innervation to the lateral aspect of the third digit and the fourth and fifth digits. The innervation to the lateral aspect of the hallux and medial aspect of the second digit is supplied by sensory branches of the deep peroneal nerve. The saphenous nerve provides sensation to the dorsal medial aspect of the foot.

An outlast shoe may be utilized to correct which pediatric deformity? A. Flexible pes planus B. Rigid pes planus C. Equinus D. Metatarsus adductus

Correct answer: (D) Metatarsus adductus. Explanation: The forefoot of an outlast shoe is directed outward to provide correction in the transverse plane. This would be an appropriate treatment for metatarsus adductus of the forefoot. Children generally graduate to straight last shoes after completing treatment. (A) (B) An outlast shoe would not provide medial longitudinal support for a flexible or rigid pes planus deformity. It would not be appropriate to direct the forefoot outward with an outlast shoe. (C) The outlast shoe mainly corrects in the transverse plane. It would not provide correction or accommodation for an equinus deformity.

An 83 year old female with NIDDM and profound neuropathy continually suffers from repeated ulceration on the plantar aspect of the left midfoot. She is a resident of a skilled nursing facility. She ambulates minimally, but it has been stressed by the patient, her family and the facility the importance of maintaining this mobility as long as possible. Pedal pulses are palpable. Complete midfoot collapse with an abducted forefoot is noted. The deformity is rigid and stable. Multiple unsuccessful attempts of offloaded with diabetic shoes have been attempted. AFO offloading is not an option as the facility believes this will make her ambulation too difficult and greatly increase her fall risk. The following are images of the patient described above. When consulted for surgical correction the best surgical option for this patient would is: A. Talonavicular-cuneiform arthrodesis B. Plantar-flexory medial closing wedge osteotomy of the medial cuneiform C. Triple Arthrodesis D. Midfoot Ostectomy

Correct answer: (D) Midfoot Ostectomy. Explanation: The key elements to this question is that our patient is geriatric and minimally ambulatory, but in need of a surgical intervention due to the recurrent pressure related issues. Her advanced age introduces several issue that must be considered. First her skin is likely thin and atrophic as well as her bone stock is likely osteopenic. Additionally, if a period of prolonged nonweightbearing is required then she might not be able to regain her level of mobility. For these reasons extensive procedures such as the talonavicular-cuneiform arthrodesis, cuneiform osteotomy and triple arthrodesis would all pose too many negatives due to bone quality issues, prolonged convalescence and tenuous soft tissue envelope. The midfoot osteotomy is ideal as convalescence is minimal, osteotomy and arthrodesis healing is not needed and the deformity is rigid and stable.

A concerned parent brings a 5 month old infant with flexible metatarsus adductus for a consult. The patient does not seem to have pain, however the patients are concerned with the appearance of the foot. The appropriate treatment plan at this time would be: A. Begin serial casting B. 3 month trial with Denis-Brown bar C. Surgical medial soft tissue release D. Monitor closely

Correct answer: (D) Monitor closely. Explanation: For infants under six months of age, it is appropriate to observe the flexible metatarsus adductus. The flexible deformity generally resolves by the time the child starts walking. (A) Serial casting is performed for rigid metatarsus adductus deformity, generally initiated before the child is 8 months old. Casting is performed until the fifth metatarsal base is no longer prominent, the convex lateral border is corrected, and the rearfoot is rectus. (B) Denis-Brown bars can be utilized after serial casting to maintain the correction of the deformity. However, the use and results are controversial and is not widely accepted. It would not be indicated as first line treatment for this patient. (C) Surgical intervention would not be warranted as first line treatment for this five month old infant with flexible deformity. Only if the patient was older, usually between four to six years old, and had failed conservative treatments including serial casting would a surgical soft tissue release be considered.

Patients with a history of stroke, intracranial hemorrhage, intracranial atherosclerosis or vasculitis, undergoing foot and ankle surgery, should have the following preoperative workup: A. Echocardiogram is sufficient B. An EKG only is sufficient C. Cardiac stress testing is the test of choice D. Must include EKG, echocardiography, and possible cardiac stress test

Correct answer: (D) Must include EKG, echocardiography, and possible cardiac stress test. Explanation: Patient with a history of stroke, intracranial hemorrhage, and intracranial atherosclerosis undergoing elective surgery should undergo a workup including a thorough EKG, echocardiogram and possibly cardiac stress test. This patient population often has a substantial history of risk factors such as hypertension, diabetes, dyslipidemia, and cardiac arrhythmias. They may have vascular disease with significant end organ damage. (A) Echocardiography refers to the evaluation of cardiac structures and function with images and recordings produced by ultrasounds. It helps to assess the cardiac anatomy into a 2D modality performed from either the thorax (TTE) or from within the esophagus, capable of delineating flow and deriving data. This test should be used in conjunction with EKG, and cardiac tress tests, in patient with a history of stroke, intracranial hemorrhage, and intracranial atherosclerosis undergoing elective surgery. (B) EKG has many uses. It can serve as an independent marker of myocardial, ionic, and drug- induced abnormalities of the heart. It can provide information that is essential for the heart, and it can provide information that is essential for proper diagnosis and therapy of many cardiac problems. It is recommended not be used alone in the perioperative assessment of patients with a history of stroke, intracranial hemorrhage, and intracranial atherosclerosis undergoing elective surgery. (C) Stress testing is a non -invasive test used to evaluate the cardiovascular system's response to stress and exercise under carefully controlled conditions. Exercise is the body's most common physiologic stress; it places major demands on the cardiopulmonary system. Thus exercise is to be considered the most practical test cardiac perfusion and function. It can have a high yield of diagnostic information when combined with other modalities.

An 11 year old male accompanied by his mother, presents to the clinic with an infected ingrown left hallux nail. The decision is made for an incision and drainage of the nail border. You explain to the mother in layman terminology the nature of the procedure, all reasonable alternatives to the procedure, the relevant risks benefits and uncertainties related to each alternative. During this discussion you verify the mother's understanding of the information and answer any and all questions that the mother has. After finishing this process the mother signs a consent form for an incision and drainage of her son's left hallux nail border. Was informed consent obtained? Yes, all the generally accepted elements of informed consent were discussed. Yes, the mother, without coercion, signed the consent form. No, the child has the majority and has demonstrated the decision making skills to provide his own informed consent. No, although all the generally accepted elements were included, the mother is not capable of providing informed consent, but rather informed permission for treatment.

Correct answer: (D) No, although all the generally accepted elements were included, the mother is not capable of providing informed consent, but rather informed permission for treatment. Explanation: The foundation of this question lies in the fact that children do not have decision- making ability in order to provide informed consent. Furthermore, since consent is given for an intervention for oneself by oneself, parents cannot, by definition, provide informed consent on behalf on their children. In these scenarios, parents and/or guardians are providing informed permission for treatment. Emancipated minors are viewed differently. Emancipated minors are capable of providing their own informed consent.

Is the arthrodesis site stable and what radiographic indicator verifies the degree of stability? A. Yes; good alignment relative to the second metatarsal in the sagittal plane B. No; dorsal jamming of the first metatarsophalangeal joint C. Yes; intact, unbroken fixation D. No; lucency around the screw heads

Correct answer: (D) No; lucency around the screw heads. Explanation: The photo above shows an atrophic nonunion of a Lapidus procedure. Indicative of this is the absence of bone callus and the sclerotic fusion surfaces. No trabeculations are noted across the fusion site. Furthermore, there is lucency around the screw heads indicative of movement and instability of the arthrodesis site. No dorsal osteophytes of the first metatarsal head are noted that would suggest dorsal jamming.

This accessory ossicle is found adjacent to the navicular tuberosity: A. Os interphalageus B. Os intermetatarseum C. Os vesalianum D. Os tibiale externum E. Os peroneum

Correct answer: (D) Os tibiale externum. Explanation: Present in approximately 10-15% of children and is one of the most common accessory ossicles. The ossicle is found adjacent to the navicular tuberosity and may either represent a sesamoid in the posterior tibial tendon or an accessory ossification center for the navicular tuberosity. The sesamoid characteristically is round, small, and located at a distance from the navicular tuberosity. The accessory ossification center, in contrast, is larger, oval or semicircular, and in close apposition to the tuberosity. Cartilage or fibrocartilage may attach to it, or it may articulate with the tuberosity, containing true synovial tissue.

Calcium phosphate and hydroxyapatite exhibit which principle of bone healing? A. Osteoinduction B. Osteogenesis C. Osteosynthesis D. Osteoconduction

Correct answer: (D) Osteoconduction. Explanation: Hydroxyapatite is converted from coral through a hydrothermal exchange process. It is purely an osteoconductive material. Calcium phosphate has been used as a restorable bone cement for filling defects. The cement acts as a scaffolding through which osteoconduction occurs. Osteoinduction involves induction of bone turnover processes. Osteogenesis is the regeneration of bone. Osteoconduction is the scaffold for new bone to form.

What is the most common risk for patients with epilepsy undergoing a surgical procedure? A. Nuchal rigidity B. Facial drooping C. Malignant hyperkalemia D. Perioperative seizure

Correct answer: (D) Perioperative seizure. Explanation: Perioperative seizure is the most common risk for patients with epilepsy disorder. It is therefore important to obtain a complete history, including type of seizure disorder and its characteristics so that the perioperative team can recognize if and when the patient has a seizure. If there are structural or metabolic abnormalities, they should be corrected if possible. Uremia, hypoglycemia, and drug toxicity or withdrawal should be avoided. (A) Nuchal rigidity may be a sign of more ominous process, among other disorders this could be associated with meningococcemia. This should be correlated with the history and physical, signs and symptoms are of the most critical importance. A complete white blood cell count with differential should then be ordered, and consider cancelling the surgery, until the abnormalities are corrected. (B) Facial drooping may be associated with other disorders such as underlying stroke, if it is sudden surgery should be cancelled, and patient should be diligently directed for appropriate screening of the underlying disorder, as this could be life threatening. (C) Malignant hyperkalemia is mostly associated with another neurologic disorder such as Duchenne's muscular dystrophy.

What structure supports the head of the talus? A. Deep distal slips of the deltoid ligament B. Posterior tibial tendon C. Plantar fascia D. Plantar calcaneonavicular ligament

Correct answer: (D) Plantar calcaneocuboid ligament. Explanation: Although the calcaneus and navicular do not articulate directly, the plantar calcaneonavicular or "spring" ligament passess from the sustentaculum tali to the plantar surface of the navicular. This ligament serves to support the head of the talus. Additional support of the talocalcaneonavicular joint includes the dorsal talonavicular ligament and the calcaneonavicular limb of the bifurcate ligament.

Which one of the following condition will least likely be a contraindication to spinal or epidural anesthesia? A. Cellulitis on the back in area of needle stick B. Shock/Trauma patient C. Patient with INR 3.7 D. Pregnancy

Correct answer: (D) Pregnancy. Explanation: Contraindications include cellulitis in the area of where the needle will be placed, a patient in shock or trauma patient as they tend to be in a state of hypovolemia. A patient on blood thinners (who did not stop) would also be a contraindication to avoid bleeding into the spinal cord. Another contraindication is in patients with multiple sclerosis. Pregnancy is not a contraindication to regional anesthesia (think of the epidural females get for C-Section).

What should the position of the midtarsal joint be in during orthotic casting for a patient with plantar fasciitis? A. Supinated B. Inverted C. Dorsiflexed D. Pronated

Correct answer: (D) Pronated. Explanation: It is essential that the midtarsal joint be casted in pronation when casting a patient with plantar fasciitis. As supination of the midtarsal joint is the pathologic motion that predispose the foot for plantar fasciitis, it is imperative the midtarsal joint is maximally pronated. (A) Holding the midtarsal joint in supination during the casting process should be avoided at all cost. If the cast would be taken with the midtarsal joint supinated the foot would be continually held a pathologic position and a negative clinical outcome would occur. (B) Holding the midtarsal joint inverted during the casting process should be avoided at all cost. If the cast would be taken with the midtarsal joint supinated the foot would be continually held a pathologic position and a negative clinical outcome would occur. (C) Holding the midtarsal joint in a dorsiflexed position would not provide a beneficial effect in the casting process for a patient with plantar fasciitis.

A 34 year old female presents one week after a morton's neurectomy on the right foot. She is complaining of significant pain and throbbing in her surgical foot. She admits to being on the foot more than what she was advised. Once the surgical dressing is removed it was evident she had a post operative hematoma. What is the most reasonable initial treatment for this foot? A. empiric oral antibiotics B. aspiration C. incision and drainage D. reduced ambulation and compressive dressing

Correct answer: (D) Reduced ambulation and compressive dressing. Explanation: The clinical scenario above depicts a dead space hematoma status post a third space neurectomy. This leads to not only increased patient pain and frustration, but can be a virtual smorgasbord for bacteria with resultant infection. Likely contributing to the hematoma is the patient's noncompliance in regard to activity limitations. Initial treatment of hematomas are typically successful and usually consist of reduced ambulation/weight-bearing and compression dressings. If unsuccessful then aspiration and or removal of sutures with full drainage can be performed. Antibiotics are not indicated in this case as this is not infectious in nature.

The transverse axis of motion at the first metatarsophalangeal joint allows movement in what plane? A. Coronal plane B. Coronal and sagittal plane C. Sagittal plane of transverse plane D. Sagittal plane

Correct answer: (D) Sagittal plane. Explanation: The transverse axis of the first metatarsal phalangeal joint is an axis whose position moves with sagittal plane motion of the joint. As arthrodial motion of the joint begins at 20 degrees of dorsiflexion, the transverse axis shifts dorsally and proximally within the metatarsal head.

Which nerve is not affected in a popliteal block and what is the reason it is not affected? Saphenous - It branches from the sciatic nerve proximal to the popliteal fossa. Tibial - It is not a branch of the sciatic nerve and is therefore not located in the popliteal fossa. Sural - It does not have motor branches. Saphenous - It is not a branch of the sciatic nerve and is not located in the popliteal fossa.

Correct answer: (D) Saphenous - It is not a branch of the sciatic nerve and is not located in the popliteal fossa. Explanation: A popliteal block, also known as a popliteal sciatic nerve block, is a commonly used regional block in foot and ankle surgery. The goal of a popliteal block is to block the sciatic nerve in the proximal aspect of the popliteal fossa. The sciatic nerve consists of 2 main branches, the tibial nerve and the common peroneal nerve, which diverge approximately 4-10 cm proximal to the popliteal fossa crease. A properly performed popliteal block will result in anesthesia of the distal 2/3 of the leg with the exception of the skin on the medial aspect of the foot and leg. The skin on the medial aspect of the leg and foot is innervated by the saphenous nerve, which is a branch of the femoral nerve. (A) The saphenous nerve is not a branch of the sciatic nerve. (B) The tibial nerve is a branch of the sciatic and is blocked during a popliteal block. (C) Though a true statement, does not apply to popliteal blocks since the sural nerve comes from both the tibial and peroneal nerves and is therefore blocked in a popliteal block. There are 2 common approaches for a popliteal block, an intertendinous approach or a lateral approach. The end result for both approaches is the same.

Which nerve listed below originates from a proximal nerve other than the sciatic nerve? A. superficial peroneal nerve B. sural nerve C. deep peroneal nerve D. saphenous nerve

Correct answer: (D) Saphenous nerve. Explanation: The sciatic nerve originates in the lower spine, travels down the posterior thigh and into the popliteal fossa where the divisions start to occur. Initially the medial sural nerve and the common peroneal nerve branch while the sciatic nerve continues distally past the ankle and into the foot as the tibial nerve. The common personal nerve further branches around the fibular head into the superficial and deep peroneal nerve. The medial sural nerve receives an anastomotic branch from the common peroneal nerve to form the sural nerve. On the other hand, the saphenous nerve is the terminal sensory branch of the femoral nerve.

A 29 year old sand volleyball player presents to the clinic complaining of pain to the back of his right ankle. States about four days ago when playing sand volleyball when his toes got caught in the sand bending his foot sharply back. He noticed instant pain in the back of his ankle. Also, he reports that he notices increased pain when going down stairs and sometimes it feels as if it is going to give out on the stairs. Physical exam reveals tenderness on palpation to the posterior lateral ankle joint. The pain is reproduced when the foot is plantar flexed and an axial load is applied to the heel. Active plantarflexion and dorsiflexion of the hallux causes pain. Radiographic images are below. What is the diagnosis? A. Symptomatic os trigonum B. Posteromedial OCD talar lesion C. Steida's Process D. Shepherds fracture

Correct answer: (D) Shepherds fracture. Explanation: The key to the correct answer to this question is the recognition of the comparative contralateral radiograph. The mechanism of injury, signs and symptoms described above are appropriate for this type of injury. The importance of the contralateral radiograph lies in the fact that fewer than 1% of patients have a free os trigonum on one side and a fused os trigonum on the other side. On the left ankle a fused os trigonum is readily identifiable, while a fractured lateral talar process is identified on the right.

A 15 y/o patient presents with a painful bunion deformity. She has seen another podiatrist and tried orthotic devices and stretching for equinus but remains dissatisfied. She now presents to you to discuss surgery. Clinically, she has 5 degrees forefoot varus and 5 degrees rearfoot valgus with stance. The Silfverskiold test shows 0 degrees ankle DF with the knee extended and 7 degrees ankle DF with the knee flexed. Gait analysis demonstrates a pronated foot. Radiographs demonstrate: HIA 7 degrees; DASA 6 degrees; PASA 5 degrees; HAA 25 degrees; IMA 15 degrees; MAA 20 degrees; Metatarsal Protrusion Distance +2mm. Based upon the information provided, the BEST combination of surgical procedures are: A. McBride, Reverdin-Laird, Arthroereisis, Gastrocnemius recession. B. Silver, Youngswick, Arthroereisis, TAL. C. Silver, Juvara type A, Heyman, Herndon and Strong (HHS), TAL. D. Silver, Juvara type A, Lepird, Gastrocnemius recession.

Correct answer: (D) Silver, Juvara type A, Lepird, Gastrocnemius recession. Explanation: The patient has a fully compensated forefoot varus deformity, with forefoot varus and rearfoot valgus equal. If there is more ankle DF with the knee flexed versus the knee extended, a gastrocnemius equinus exists, and a recession is indicated. If there was a lack of adequate DF with the knee both extended and flexed, a TAL is necessary. But, how much DF is necessary to prevent pathology? Although classically taught as 10 degrees, many in the profession are trending toward 5 degrees as being adequate. Radiographs demonstrate an abnormal hallux abductus angle, slightly abnormal IM angle, and high metatarsus adductus angle (upper limit 15 degrees). The pronated gait is secondary to compensation for the metatarsus adductus. When evaluating a bunion deformity, look at the metatarsus adductus angle first, and build your choice of procedures from there. Boards want to know if you appreciate the role of metatarsus adductus; remember the formula: IMtrue=IMmeasured+(MAA-15). In this case, the IMtrue is 20 degrees, necessitating a proximal osteotomy. The use of arthroereisis in this case may actually unmask the adductus, leading to an in-toe gait post-operatively. The Youngswick plantarflexes and shortens; the MPD is normal. The H,H+S is a metadductus procedure indicated for a younger age group; the Lepird is an osseous procedure for metadductus. Finally, realize the Silver as originally described did include a lateral release, not just a "bumpectomy."

The most common complication associated with MBA STJ arthroereisis is: A. Infection B. Flattening of the lateral talar process C. Inflammatory reaction caused by polymethylmethacrylate cement D. Sinus tarsalgia (reactive synovitis)

Correct answer: (D) Sinus tarsalgia (reactive synovitis). Explanation: This is more common in adults than children, but can be seen with a heavier child. This can be likened to Chang's description of "lateral impingment syndrome", which can result from insertion of too large an implant, resulting in minimal motion. The impingment results from the implant impinging against bone or soft tissue. It may be seen early or late in the post- operative period. Treatment is a heel lift, steroid injection; if unsuccessful, replacing the implant with a smaller one. (A) The infection rate is no greater than with other surgeries of the foot. Appropriate antibiotic prophylaxis with cephalexin, or clindamycin if allergic, is warranted. (B) Flattening of the lateral talar process is seen with a STA-Peg type of device; an axis-altering device, as opposed to the self-locking wedge type MBA device. In addition, sclerotic changes in the calcaneus were noted. However, neither radiographic change seemed to affect clinical satisfaction. (C) The advantage of the MBA device is that cement is not required to maintain its placement in the sinus tarsi, as it is entirely extra-articular. In fact, using cement with the STA-Peg has become optional and based upon surgeon preference.

A 31 year old female reports for her first post-operative follow-up one week after left 3rd inter- metatarsal neurectomy. She reports that for the first couple days she was doing well and her pain was well controlled. On the the third day her pain noticeably increased. She denies any nausea, fever, vomiting or chills at that time or currently. This increased pain level continued until yesterday. Today she states her pain is much better. Physical exam of the foot shoes intact sutures and well coapted incision. Mild edema is present consistent with post-operative edema. No erythema, drainage or warmth noted. The patient is afebrile. What is the most likely cause of this increased pain on the third post-operative day? A. Post-operative infection B. Post-anesthetic overshoot C. Stump neuroma D. Spontaneous firing of the neuroma

Correct answer: (D) Spontaneous firing of the neuroma. Explanation: The term neuroma is actually a misnomer as it pertains to the inter metatarsal space. A more accurate term is a inter metatarsal perineurial fibrosis. However, when the "neuroma" is resected a true neuroma is formed at the level of the proximal digital nerve stump. Once it is severed it begins spontaneous firing and exhibits two peaks of activity. The first peak of activity occurs on the third postoperative day with the second peak occurring within the third postoperative week. For some patients, these peaks can be more symptomatic than in others and this is believed to be enhanced by sympathetic fibers. There is no evidence of infection or hematoma clinically. (A) There is no evidence of infection or hematoma clinically so post-operative infection is unlikely. Furthermore she stated here pain recited the days before the visit. (B) Post-Anesthetic overshoot pertains to a potential fever developing after surgery. (C) One week after neuroma surgery is far too early to declare the presence of a stump neuroma.

In gait cycle, a period from heel contact of one limb to heel contact of the opposite limb is also known as? A. Cadence B. Stride length C. Step base of gait D. Step

Correct answer: (D) Step. Explanation: Step is defined as the period from heel contact of one limb to heel contact of the opposite limb. Each stride consists of two steps, usually of equal length.

Which of the following is seen with CLOSED chain pronation of the subtalar joint? A. Calcaneus inverts B. Talus abducts and plantarflexes C. Tibia externally rotates D. Talus adducts and plantarflexes

Correct answer: (D) Talus adducts and plantarflexes. Explanation: Closed chain means that the foot is weight-bearing on a supportive surface and is thus loaded. This is different that open chain kinetics which is when the foot is not directly on the weight-bearing surface. With closed chain pronation, the calcaneus everts while the talus adducts and plantarflexes. The tibia internally rotates and this is usually seen in the tibial tuberosity. With closed chain supination, the calcaneus inverts while the talus abducts and dorsiflexes. There is external rotation of the tibia. Thus, looking at the above answer choices, the only choice that fits would be that the talus adducts and plantarflexes in closed chain pronation.

Which of the following statements about the Cole procedure is false? The Cole procedure is not indicated for the skeletally immature foot. The Cole procedure is indicated when the deformity is rigid and the apex of deformity is located in the midfoot. When performing the Cole procedure one must preserve the talonavicular and calcanea-cuboid joints. The Cole procedure is indicated when the deformity is flexible and the apex of deformity is located in the midfoot.

Correct answer: (D) The Cole procedure is indicated when the deformity is flexible and the apex of deformity is located in the midfoot. Explanation: The Cole procedure is described as a closing wedge osteotomy with removal of a dorsally based wedge. The wedge is removed from a distal cut through the cuboid and cuneiforms coupled with a proximal cut through the cuboid and navicular. It is only indicated in the skeletally mature foot. Performing the procedure in the skeletally immature foot can lead to shortening. Additionally the osteotomy does not involve the talonavicular or calcaneal-cuboid joints. These joints are spared in the procedure. The Cole procedure is only indicated in the rigid anterior cavus foot when the apex of deformity is in the midfoot.

While assisting in a surgical correction of a hammer toe deformity, you are questioned on some specific surgical anatomy involved in the etiology of contracted digits. You are asked to name the specific attachment of the extensor apparatus encasing the extensor digitorum longus and brevis, please choose the correct answer: A. The middle phalanx. B. The medial and lateral fibrous extensions. C. The sling mechanism wrapping around the base of the proximal phalanx. D. The extensor apparatus has no specific attachments into the proximal phalanx.

Correct answer: (D) The extensor apparatus has no specific attachments into the proximal phalanx. Explanation: The extensor expansions encircle the proximal phalanx and attach to themselves. The expansion fibers are more oblique as they near the proximal interphalangeal joint. The interossei insert into the plantar foremost aspect of the proximal phalanx above the deep transverse intermetatarsal ligmant and insert into the plantar and deep transverse metatarsal ligament.

The soleus and gastrocnemius are both part of the posterior calf muscles, and are active during contact phase of the gait cycle. What is the function of the gastrocnemius muscle? A. The soleus decelerates internal rotation of the tibia B. Accelerates internal rotation of the tibia C. Decelerates external rotation D. Provides mild flexion of the knee to prevent hyperextension

Correct answer: (D) The gastrocnemius provides mild flexion of the knee to prevent hyperextension. Explanation: 1. The soleus decelerates internal rotation of the tibia. 2. The soleus provides mild deceleration of calcaneal eversion. 3. The soleus decelerates forward movement of the tibia. 4. The gastrocnemius provides mild flexion of the knee to prevent hyperextension. Both muscles initiate activity midway through this phase and therefore do not develop sufficient torque to control subtalar joint function. With the foot on the floor, the soleus exerts a force from the heel to its tibial insertion, resisting forward movement of the tibia due to momentum. The gastrocnemius insertion on the femur creates flexion tension on the knee to resist hyperextension. In addition, the tendon of the gastrocnemius and soleus inserts medial to the subtalar joint axis, thereby introducing a supination effect on the subtalar joint which assists in decelerating its pronation movement.

All of the following statements regarding surgical fixation with titanium are correct except: The alloy Ti-6Al-4V, commonly used for bone fixation, consists of 6% titanium, 4% aluminum, and vanadium. Has a high corrosion resistance. The protective oxide film can reform instantaneously if broken by wear and abrasion. The modulus of elasticity of this titanium alloy is more rigid than that of bone.

Correct answer: (D) The modulus of elasticity of this titanium alloy is more rigid than that of bone. Explanation: The modulus of elasticity of titanium alloy (Ti-6Al-4V) is half that of other surgical metals and nearly matches that of bone. This allows for significantly less stress shielding than with surgical stainless steel. It indeed does have a high corrosion resistance as a result of a titanium oxide film that forms on the implant. If that film would become disrupted as in the case of joint motion or load bearing it would instantaneously reform. This can occur even in minute amounts of oxygen. This phenomenon leads to superior durability and longevity of the implant.

Neuromyodesis is an attempt to prevent stump neuroma formation. When performed: A. The nerve stump is sutured to an extrinsic muscle of the foot. B. The endoneurium of the nerve is sutured to muscle. C. The nerve stump is pulled taut up to the muscle utilized. D. The procedure is performed on a NWB area of the foot.

Correct answer: (D) The procedure is performed on a NWB area of the foot. Explanation: A stump neuroma forms from irritation to the cut nerve end, the irritation coming from movement and weightbearing. Maintaining the neuromyodesis on a NWB area of the foot reduces stump neuroma formation. (A) Stump neuromas forms in areas of movement. An extrinsic muscle has more movement than an intrinsic muscle. Examples of intrinsic foot muscles utilized in neuromyodesis include the interossei, lumbricales, adductor hallucis, or FDB. (B) The outermost layer of the nerve, the epineurium, is sutured to muscle, using small (6-0) absorbable or non-absorbable suture. The endoneurium encircles nerve fibers within a bundle. (C) Principles of neuromyodesis include the use of an intrinsic muscle, off the WB surface, and secured with suture to a muscle under no tension.

A 19 year old male sustained a major inversion injury. Immediately he was unable to bear weight on the foot. Gross deformity of the ankle was present. The distal neuromuscular status was intact. Radiographic exam in the emergency department confirmed medial dislocation of the subtalar joint. Which of the following statement is false regarding medial subtalar joint dislocations? The deep portions of both the deltoid and the calcaneofibular ligaments are disrupted The bifurcate ligament and the spring ligament are the only ligaments not disrupted in this injury The sustentaculum tali acts as a fulcrum for the posterior body of the talus to produce the dislocation The sequence of disruption begins with the subtalar joint first followed by the talonavicular joint

Correct answer: (D) The sequence of disruption begins with the subtalar joint first followed by the talonavicular joint. Explanation: Medial subtalar joint dislocations are a gruesome injury that account for 1% of all dislocations. It involves the subtalar joint itself, the talonavicular joint, and the ankle joint. Studies have shown the deep portions of the deltoid and the calcaneofibular ligaments are involved in both the medial and lateral subtalar dislocations. Regardless of the type of dislocation the bifurcate ligament and the spring ligament are always preserved. In the medial subtalar joint dislocation the sustentaculum tali does act as a fulcrum for the posterior body of the talus. When the dislocation occurs the talonavicular joint must first dislocate before the talus can rotate and dislocate the subtalar joint.

A 33 year old male presents to the emergency department. Approximately an hour earlier he was chopping wood and a large log dropped on his toe. Below is a photographical and radiographic images of his toe on initial presentation. Please choose the most correct statement listed below. A. Open reduction and internal fixation is indicated. B. Treatment should consist of evacuation of the hematoma and immobilization. C. Little concern of osteomyelitis is warranted. D. This should be classified and treated as an open fracture.

Correct answer: (D) This should be classified and treated as an open fracture. Explanation: Any phalangeal fracture with a lacerated nail bed is to be classified and treated as an open fracture. Whenever a subungal hematoma constitutes more than 25% of the nail then as in this case a nail bed laceration might be present. (A) In this contaminated open fracture, with a non displaced fracture, open reduction and internal fixation is not indicated. (B) Evacuation of the hematoma and immobilization would not address the laceration or the open nature of the fracture. (C) It has been well documented that direct extension osteomyelitis has been linked to nail bed lacerations and open fractures.

Which of the following shoe modifications would be beneficial in reinforcing the medial longitudinal arch for flexible pediatric flatfoot? A. External shoe lift B. Forefoot rocker C. Lateral heel wedge D. Thomas heel

Correct answer: (D) Thomas heel. Explanation: The Thomas heel is a shoe modification used to support the medial longitudinal arch which can be beneficial in the case of flexible pes planus. The external modification extends the standard heel a 1⁄2" on the medial side to provide the additional support. (A) The external shoe lift is utilized in the case of pediatric limb length discrepancy. The lift is generally applied to the bottom of the sole in order to accommodate the inequality of the limb length. (B) A forefoot rocker is an additional platform on the bottom of the sole that sharply tapers from the metatarsophalangeal joints to the distal end of the shoe. This can relieve pressure from the MTPJs by facilitating midstance and toe-off. This would not be appropriate in accommodating flexible pes planus. (C) A lateral heel wedge would further influence the hindfoot into valgus position, which would accentuate or worsen the deformity rather than correcting it.

After an extensive work up the decision has been made to do a nail plate and nail matrix excision with a histopathological examination on the patient pictured below. Which nail ablation and matricectomy technique would be most appropriate? A. Total nail avulsion with laser matricectomy B. Total nail avulsion with electrocautery and curettage of the nail matrix C. Total nail avulsion with chemical matricectomy D. Total nail avulsion with scalpel excision of nail matrix

Correct answer: (D) Total nail avulsion with scalpel excision of nail matrix. Explanation: The key point to this question is the need to preserve both the nail plate and the nail matrix for histopathological examination. Only scalpel excision of the nail matrix would preserve the nail matrix in its entirety for histopathological examination. Laser nail matrix excision, chemical matricectomy and electrocautery and curettage would cause too much destruction and not preserve the nail matrix for histopathological examination.

Please refer to the images below. What is the most likely diagnosis out of the list provided? A. Aneurysmal bone cyst B. Intraosseous ganglion C. Giant cell tumor D. Unicameral bone cyst

Correct answer: (D) Unicameral bone cyst. Explanation: In the skeletally mature individual, simple bone cysts are usually seen in the calcaneus and iliac wing. UBCs are among the most common lesions of the talus. They represent a reactive process. On radiographs they appear as a well-defined lytic lesion without central mineralization. MR imaging can confirm the diagnosis and better define the relationship of the lesion to the subtalar joint and calcaneal-cuboid joint. Usually located in the anterior portion of the calcaneus. On MR they show homogenous low to intermediate signal intensity on T1- weighted images and high signal intensity on T2-weighted images. Aneurysmal bone cysts on MR would be lobulated and multi-septated. Intraosseous ganglion on radiograph would have a sclerotic rim and on MR would be multilocular. Giant cell tumors don't often occur in the calcaneus and on MR imaging they show a varied signal pattern due to hemorrhage and cyst formation.

Screws are available in three basic sets: mini fragment, small fragment, and large fragment. What outer diameters are found in the large fragment set? A. 2.7 and 3.5 mm B. 1.5 and 2.0 and 2.7 mm C. 2.7 and 3.5 mm D. 3.5 and 4.0 mm E. 4.5 and 6.5 mm

Correct answer: (E) 4.5 and 6.5 mm. Explanation: AO/ASIF screws are available in three basic sets (other sets are available): minifragment, small fragment, and basic or large fragment. The minifragment set contains screws with an outer diameter of 1.5, 2.0, and 2.7 mm. The small fragment set contains screws with an outer diameter of 3.5 and 4.0 mm. The large fragment set contains both 4.5- and 6.5-mm screws. All AO/ASIF screw sets come complete with a full complement of equipment to insert each type of screw. Screws are available in stainless steel or titanium.

Place in order of density from least radiopaque to most radiopaque: air, bone, surgical pin, fat, connective tissue. A. Fat, air, connective tissue, surgical pin bone B. Connective tissue, bone, surgical pin, air, fat C. Bone, surgical, connective tissue, air, fat D. Surgical pin, bone, connective tissue, fat, air E. Air, fat, connective tissue, bone, surgical pin

Correct answer: (E) Air, fat, connective tissue, bone, surgical pin. Explanation: X-rays interact with matter in different ways, they may be scattered in different directions, or they can be absorbed by an atom. The energy of the photon and the atomic number influence whether the x-ray is absorbed. The difference is known as differential absorption. Objects of greater anatomic absorb x-rays more readily than those with lower anatomic numbers. The thickness of the material also plays a role into absorption of x-rays. The thicker the substance is the more x-rays it absorbs, this is known as attenuation. Radiopaque applies to those substances that absorb x-rays; they appear white on the exposed film. Bone is radiopaque relative to muscle, but radiolucent when compared to a heavy metal such as lead. In the same manner a thicker foot is more radiopaque than the thinner foot, when the views are the same and the radiographic techniques are the same for both feet.

This is also known as fracture or a break in a metal caused by repetitive applications of loads below the yield strength. Stress risers occur due to microfractures that increase in size under cyclic loading. The microfractures will eventually increase in size enough to cause failure: A. Fatigue fracture B. Fatigue life C. Fatigue limit D. Ultimate tensile strength

Correct: A Explanation: Fatigue fracture is a fracture or a break in a metal caused by repetitive applications of loads below the yield strength. Stress risers occur due to microfractures that increase in size under cyclic loading. The microfractures will eventually increase in size enough to cause failure. Fatigue life is the number of cycles a metal can undergo without breaking when placed under a given cyclic load. Fatigue limit or endurance limit is the cyclic load that a metal can endure indefinitely. Ultimate tensile strength is the maximum stress a material can undergo in a single application without breaking.

Closed reduction of lateral subtalar dislocations may be prevented by interposition of the: (Select all that apply) A. Tibialis anterior tendon B. Tibialis posterior tendon C. FHL tendon D. FDL tendon

D - FDL tendon,B - Tibialis posterior tendon. Correct answer: (B) (D) Tibialis posterior tendon and FDL tendon. Explanation: The tibialis posterior tendon is the most common tendon to prevent closed reduction, being snugged up tight against and around the talar head. The FDL is also possible, but more rare.

Anatomically, dorsal subluxation of the first ray is prevented by which of the following components of the tarsometatarsal joint? (Select all that apply) A. First plantar cuneometatarsal ligament. B. Peroneus longus. C. Interosseous ligament between first metatarsal and first cuneiform. D. FHL.

D - FHL.,B - Peroneus longus.,A - First plantar cuneometatarsal ligament.. Correct answer: (A) (B) (D) First plantar cuneometatarsal ligament, peroneus longus and FHL. Explanation: Mizel studied the first plantar cuneometatarsal ligament and found it to be key to preventing dorsal mobility of the first ray. Both the peroneus longus and FHL are dynamic stabilizers of the first TMT articulation. The interosseous ligament is not always present, and if present, could be a small portion of the Lisfranc ligament. It is less than 2mm thick.

You are planning an elective foot and ankle surgery on a patient who has just finished a course of 20 mg/d x 30 days of prednisone; you are planning an open plantar fasciotomy of the left foot. (Select all that apply) The hypothalamic-pituitary-adrenal axis suppression for up to 6-12 months. The hypothalamic-pituitary- adrenal axis suppression the same amount of days as the treatment. Proceed with the surgery without supplementation. If time allows, an ACTH stimulation test should be performed to see if the adrenal gland responds appropriately to supraphysiologic doses of ACTH.

D - If time allows, an ACTH stimulation test should be performed to see if the adrenal gland responds appropriately to supraphysiologic doses of ACTH.,A - The hypothalamic-pituitary- adrenal axis suppression for up to 6-12 months.. Correct answer: (A) (D) Explanation: In general any patient who has received the equivalent of 20 mg/d of prednisone for greater than 5 days is at risk for suppression of the hypothalamic-pituitary- adrenal axis, and if they have been on therapy for approximately a month they may have HPA suppression for up to 6 to 12 months after stopping therapy. Similarly, an equivalent dose of prednisone 5 mg or less for any period of time will usually not significantly suppress the HPA axis. Other modes of steroid administration should be noted preoperatively as topical, inhaled and regional administration of glucocorticoids may cause adrenal suppression. Test to detect perioperative adrenal suppression or, perhaps more importantly, identify patients who will respond to supplemental glucocorticoids have been neither sensitive nor specific. However, the short ACTH stimulation test is able to reliably assess adrenocortical function. If the test is abnormal preoperatively, supplemental perioperative glucocorticoid administration is justified.

Radiographic evaluation of the cavus foot deformity on the lateral foot projection demonstrates a Meary's angle of 16° and a calcaneal inclination angle of 20°. From the options below what would be the best surgical procedure? (Select all that apply) A. Dwyer osteotomy B. Steindler stripping C. Cole procedure D. Japas procedure

D - Japas procedure,C - Cole procedure. Correct answer: (C) (D) Cole procedure and Japas procedure. Explanation: The radiographic parameters above describe an anterior cavus foot deformity. The Cole procedure is indicated for the anterior cavus foot deformity as it elevated the forefoot out of equinus into a more rectus position. The radiographic parameters above describe an anterior cavus foot deformity. The Japas procedure is indicated for the anterior cavus foot deformity as it elevated the forefoot out of equinus into a more rectus position. It eliminates the shortening of the foot that is seen with the Cole procedure. (A) The Dwyer osteotomy is indicated in the rigidly hindfoot varus. The radiographic parameters above describe an anterior cavus foot deformity. Therefore there is not direct indication in the information provided above for the Dwyer osteotomy. (B) The radiographic parameters above describe an anterior cavus foot deformity. While a soft tissue procedure such as a Steindler stripping might be a useful adjunct, the primary procedure should be an osseous procedure.

Which trauma classification system takes into account the amount of energy involved in the injury? (Select all that apply) A. Gustilo B. Byrd C. Tscherne D. Pachuda

D - Pachuda,B - Byrd. Correct answer: (B) (D) Byrd and Pachuda. Explanation: The Byrd classification for open fractures primarily focuses on the amount of energy associated with the injury. It ranges from Type I injuries which are low energy to type IV injuries which are extremely high energy injuries. The Pachuda classification for open fractures primarily focuses on the amount of energy associated with the injury. It ranges from grade 1 injuries which are low energy injuries to grade 4 injuries which are extremely high energy injuries. (A) The Gustilo classification for open fractures primarily focuses on the fracture and its degree of contamination. It is first divided into type I, II, III which signify a clean, moderately contaminated and highly contaminated wounds respectively. Type III is further subdivided into A, B, C which signify adequate soft tissue coverage, inadequate soft tissue coverage and arterial injury respectively. (C) The Tscherne classification of open fractures primarily focuses on the degree of soft tissue injury with the fracture. It ranges from 0 to 3 with 0 being slight soft tissue damage and 3 being extensive soft tissue contusion, crushing, degloving or compartment syndromes.

Patient presents to the emergency department with grade IIIA ankle fracture. After ruling out poly-trauma, and gathering a past medical history, and the return laboratory results, you found that the patient has hyperthyroidism. You suspect that it may have been undiagnosed or poorly controlled. (Select all that apply) A. B. C. D. E. Proceed with the trauma surgical reconstruction of the grade IIIA ankle fracture Beta blockers should be available as they inhibit sympathetic activation activation but also inhibit the peripheral conversion of T4 to T3 Thionamides should be readily available as they are actively transported into the thyroid gland and inhibit further production of hormone Potassium iodide should be readily available as it is usually administrated preoperatively for thyroid surgery as it decreases the vascularity of the gland Anesthetic agents containing epinephrine should be given as epinephrine is beneficial in case of thyroid storm

D - Potassium iodide should be readily available as it is usually administrated preoperatively for thyroid surgery as it decreases the vascularity of the gland,C - Thionamides should be readily available as they are actively transported into the thyroid gland and inhibit further production of hormone,B - Beta blockers should be available as they inhibit sympathetic activation activation but also inhibit the peripheral conversion of T4 to T3,A - Proceed with the trauma surgical reconstruction of the grade IIIA ankle fracture. Correct answer: (A) (B) (C) (D) Explanation: In patients with uncontrolled hyperthyroidism presenting for elective surgery, their surgical procedure should be postponed until they are on a stable medical regimen to reduce the risk of thyroid storm. For those presenting for urgent or emergent surgery, it is incumbent on the anesthesiologist to have ready access to drugs that block the systemic effects of excess thyroid hormone. Such drugs include Beta blockers, antithyroid medications, and iodine. Beta blockers not only directly inhibit sympathetic activation but also inhibit the peripheral conversion of T4 to T3. Thionamides, such as propylthouracil (PTU) and methimazole, are actively transported to the thyroid gland and inhibit further production of hormone. Furthermore, PTU inhibits peripheral conversion of T4 to T3. Finally, inorganic iodide in excess will manifest an antithyroid action known as the Wolff-Chaikoff effect. Potassium iodide can also be given as either Lugol solution or saturated solution of potassium iodide and is usually administered preoperatively for thyroid surgery as it decreases the vascularity of the gland. Anesthetic agents that are vagolytic or sympathomimetic for example: pancuronium, ephedrine, epinephrine, norepinephrine and atropine are best avoided in patients with uncontrolled hyperthyroidism as those agents should be ruled out in patients with thyrotoxicosis.

What surgical steps distinguish the McBride procedure from the Silver procedure? (Select all that apply) A. Lateral capsulotomy B. Fibular sesamoidectomy C. Adductor tendon release D. Transfer of the conjoined adductor tendon

D - Transfer of the conjoined adductor tendon,B - Fibular sesamoidectomy. Correct answer: (B) (D) Fibular sesamoidectomy and transfer of the conjoined adductor tendon. Explanation: The Silver bunionectomy procedure consist of medial capsulorraphy, medial exostectomy, lateral capsule and adductor tendon release. The McBride, Which is a modification of the Silver procedure, adds fibular sesamoidectomy and transfer of the conjoined adductor tendon to the Silver procedure. (A) Both the Silver and the McBride share the lateral capsulotomy. (C) Both the Silver and the McBride share the release of the adductor tendon.

While preparing for a pre-surgical consultation, you notice that the patient has a history of Graves disease. Select all 3 that apply regarding this endocrinopathy's symptoms. A. Cold intolerance B. Tachycardia C. Anorexia D. Goiter E. Ophthalmopathy

E - Ophthalmopathy,D - Goiter,B - Tachycardia. Correct answer: (B) (D) (E) Tachycardia, Goiter and Ophthalmopathy. Explanation: The causes of hyperthyroidism are myriad, the most common cause of hyperthyroidism by far is Graves disease, which is an autoimmune disorder caused by antibody generation directed at thyroid stimulating hormone (TSH) receptors causing increase in thyroid hormone production. The signs and symptoms are as follow: tachycardia, fever, goiter, tremor, ophthalmopathy. Other manifestations may include gastrointestinal symptoms such as diarrhea, nausea and emesis. Not all patients will have the classic symptomatology or lab findings. There are patients with subclinical presentation of hyperthyroidism characterized by normal levels of free thyroid hormone levels with suppressed TSH; this is more common in the elderly population. Cold intolerance and anorexia are most commonly found in the hypothyroidism, which is another common endocrinopathy very important in the perioperative settings. It affects 1% of the population. It is more prevalent in women, it is characterized by low thyroid hormone levels in the face of normal or elevated TSH, most common symptoms include lethargy, anorexia, fatigue, headaches, hoarse voice, depression and cold intolerance.

This area of the AO/ASIF screw has a curved surface; it serves to increase the contact area between the screw and the bone and reduces stress risers. A. The shank B. The screw head C. The tip D. The pitch

Explanation: The screw head is circular and has a hexagonal slot for the driver. The 1.5- and 2.0- mm screws are designed with either a hexagonal or a cruciate slot. The underside of the screw head has a curved contour (instead of a flat surface) extending toward the shaft. The curved surface increases the contact area between the screw head and bone and reduces the chance for stress risers. A countersink is available to remove a ring of bone at the drill hole entrance to ensure accurate fit between the underside of the screw head and the bone. Countersinking should be done completely, but not excessively. Aggressive countersinking can weaken cortical bone; this may allow screw advancement into underlying cancellous bone as the screw is tightened, Should this occur, a washer should be inserted to increase surface area under the screw, to prevent excessive advancement. A hexagonal screwdriver is available in two sizes: one designed to fit the 2.7-, 3.5-, and 4.0-mm screws and a second for the 4.5- and 6.5-mm screws.


Set pelajaran terkait

Chapter 3 Network Protocols and Communications Questions

View Set

chapter 8 conversational messages

View Set

Case Problem Analysis 4.1: Identifying the Facts & Issues

View Set

[BR UI][Unit 9] Vocabulary (Employment)

View Set